Está en la página 1de 180

GEOMETRIA LINEAL

2n Batxillerat
Amb tots els problemes PAU 1998-2020

Gerard Romo Garrido


Toomates Coolección
Los documentos de Toomates son materiales digitales y gratuitos. Son digitales porque están pensados para ser consultados
mediante un ordenador, tablet o móvil. Son gratuitos porque se ofrecen a la comunidad educativa sin coste alguno. Los libros de
texto pueden ser digitales o en papel, gratuitos o en venta, y ninguna de estas opciones es necesariamente mejor o peor que las otras.
Es más: Suele suceder que los mejores docentes son los que piden a sus alumnos la compra de un libro de texto en papel, esto es un
hecho. Lo que no es aceptable, por inmoral y mezquino, es el modelo de las llamadas "licencias digitales" con las que las editoriales
pretenden cobrar a los estudiantes, una y otra vez, por acceder a los mismos contenidos (unos contenidos que, además, son de una
bajísima calidad). Este modelo de negocio es miserable, pues impide el compartir un mismo libro, incluso entre dos hermanos,
pretende convertir a los estudiantes en un mercado cautivo, exige a los estudiantes y a las escuelas costosísimas líneas de Internet,
pretende pervertir el conocimiento, que es algo social, público, convirtiéndolo en un producto de propiedad privada, accesible solo a
aquellos que se lo puedan permitir, y solo de una manera encapsulada, fragmentada, impidiendo el derecho del alumno de poseer
todo el libro, de acceder a todo el libro, de moverse libremente por todo el libro.
Nadie puede pretender ser neutral ante esto: Mirar para otro lado y aceptar el modelo de licencias digitales es admitir un mundo más
injusto, es participar en la denegación del acceso al conocimiento a aquellos que no disponen de medios económicos, en un mundo
en el que las modernas tecnologías actuales permiten, por primera vez en la historia de la Humanidad, poder compartir el
conocimiento sin coste alguno, con algo tan simple como es un archivo "pdf". El conocimiento no es una mercancía.
El proyecto Toomates tiene como objetivo la promoción y difusión entre el profesorado y el colectivo de estudiantes de unos
materiales didácticos libres, gratuitos y de calidad, que fuerce a las editoriales a competir ofreciendo alternativas de pago atractivas
aumentando la calidad de unos libros de texto que actualmente son muy mediocres, y no mediante retorcidas técnicas comerciales.
Este documento se comparte bajo una licencia “Creative Commons”: Se permite, se promueve y se fomenta cualquier uso,
reproducción y edición de todos estos materiales siempre que sea sin ánimo de lucro y se cite su procedencia. Todos los documentos
se ofrecen en dos versiones: En formato “pdf” para una cómoda lectura y en el formato “doc” de MSWord para permitir y facilitar
su edición y generar versiones parcial o totalmente modificadas. Se agradecerá cualquier observación, comentario o colaboración a
toomates@gmail.com
La biblioteca Toomates Coolección consta de los siguientes libros:
Problem-solving:
Geometría Axiomática GA pdf 1 2 ... 23 portada
Problemas de Geometría PG pdf 123456789
Introducción a la Geometría PI pdf doc
Teoría de números AR pdf 123
Trigonometría PT pdf doc
Desigualdades DE pdf doc
Números complejos PZ pdf doc
Álgebra PA pdf doc
Combinatoria PC pdf doc
Probabilidad PR pdf doc
Guía del estudiante de Olimpiadas Matemáticas OM pdf
Libros de texto (en catalán):
Àlgebra AG pdf 1234
Funcions FU pdf doc
Combinatòria i Probabilitat CP pdf doc
Geometria analítica GN pdf 12
Trigonometria TR pdf doc
Nombres complexos CO pdf doc
Àlgebra Lineal 2n batxillerat AL pdf doc
Geometria Lineal 2n batxillerat GL pdf doc
Càlcul Infinitesimal 2n batxillerat CI pdf 12
Programació Lineal 2n batxillerat PL pdf doc
Recopilaciones de pruebas PAU España:
Catalunya TEC ST , Catalunya CCSS SC , Galicia SG
Recopilaciones de pruebas PAU Europa:
Portugal A SP, Portugal B SQ
Recopilaciones de problemas olímpicos y preolímpicos:
IMO SI, OME SE, OMI SD, AIME SA , Cangur SR , Canguro SG , Kangourou SK ,
AMC12 (2008-2020) SM , SMT SF
Versión de este documento: 14/01/2021
Todos estos documentos se actualizan constantemente. ¡No utilices una versión anticuada! Descarga gratis la última versión de los documentos en los enlaces superiores.

www.toomates.net
Índex
1 Punts i vectors. →
1.1 Vectors. Àlgebra vectorial.
1.2 Vectors proporcionals. Punts alineats. Divisió de segments.
1.3 Mòdul d’un vector. Distància entre dos punts.
1.4 Fora de programa: La desigualtat triangular.
1.5 Producte escalar de dos vectors. Angle.
1.6 Producte vectorial de dos vectors. Àrea.
1.7 Producte mixt de tres vectors. Volum. Punts coplanaris.
1.8 Relació entre els tres productes.
1.9 Fora de programa: L’esfera.
1.10 Notes històriques. La història dels vectors.
1.11 Recopilatori d’exercicis.
1.12 Taula resum dels tres productes.

2 Varietats lineals en l’espai: Rectes i plans. →


2.1 Dependència i independència lineal de vectors.
2.2 Rectes. Equacions de la recta.
2.3 Punt d’intersecció entre dues rectes.
2.4 Plans. Equacions del pla.
2.5 Punt d’intersecció entre recta i pla.
2.6 Recta intersecció de dos plans.
2.7 Rectes coplanàries.
2.8 Resolució de problemes mètrics amb rectes parametritzades.

3 Paral·lelisme. →
3.1 Paral·lelisme entre dues rectes.
3.2 Paral·lelisme entre dos plans.
3.3 Paral·lelisme entre recta i pla.
3.4 Pla paral·lel a dues rectes i que passa per un punt.
3.5 Pla que conté una recta i és paral·lel a una altra.
3.6 Recta paral·lela a dos plans.

4 Perpendicularitat. →
4.1 Perpendicularitat entre plans.
4.2 Perpendicularitat entre rectes.
4.3 Perpendicularitat entre recta i pla.
4.4 Projecció ortogonal d’un punt en una recta. Punt simètric respecte d’una recta.
4.5 Projecció ortogonal d’un punt en un pla. Punt simètric respecte d’un pla.
4.6 Recta perpendicular comú a dues rectes.
4.7 Problemes PAU de perpendicularitat i paral·lelisme.

5 Distància. →
5.1 Distància entre punt i recta.
5.2 Distància entre punt i pla.
5.3 Distància entre dos plans.
5.4 Distància entre recta i pla.
5.5 Distància entre dues rectes.
5.6 Problemes PAU amb distància, projecció ortogonal i punt simètric.

6 Posició relativa. →
6.1 Posició relativa entre recta i pla.
6.2 Posició relativa entre dos plans.
6.3 Posició relativa entre dues rectes.
6.4 Posició relativa de tres plans.

7 Angles. →
7.1 Angle entre dues rectes.
7.2 Angle entre dos plans.
7.3 Angle entre recta i pla.

8 Problemes PAU amb àrea i volum. →


8.1 Problemes PAU amb àrees..
8.2 Problemes PAU amb volum.

9 Recopilacions d’exercicis. →
10 Apèndix. →
10.1 Taula Paral·lelisme-Perpendicularitat-Angle.
10.2 Taula d’aplicacions de la projecció ortogonal.
10.3 Algunes demostracions.

Solucions. →
Links d’interès. →
1 Punts i vectors.
1.1 Vectors. Àlgebra vectorial.
Vectors de IR3.

Un vector de IR3 és una terna ordenada de tres nombres reals: v  (v1 , v2 , v3 )
Els nombres v1 , v2 , v3 s’anomenen coordenades o components del vector.
Els vectors es designen, generalment, amb una lletra minúscula coronada amb
  
una petita fletxa al capdamunt.: v , w, t ...
 
Per exemple: v  (2,1,4) , w  (3,0,1) .

Observació. Interpretació gràfica d’un vector.


Un vector en IR3 representa una direcció en l’espai, un moviment en línia recta, i
per tant es representarà per una fletxa.

Definició. Suma de vectors.


Dos vectors es poden sumar sumant els seus components corresponents. La
representació gràfica de la suma de vectors es basa en la llei del paral·lelogram.
 
Exemple: v  (1,2,7) i w  (5,4,1) , llavors
 
v  w  (1  5 , 2  (4) , 7  1)  (4,2,8)

Propietats de la suma de vectors.


     
Associativa: u  v   w  u  v  w
   
Commutativa: u  v  v  u
   
Existència de vector nul: 0  v  v on 0  (0,0,0)
  
Existència de vector oposat: v  ( v )  0 , on si
 
v  (v1 , v2 , v3 )  v  (v1 ,v2 ,v3 )

Observació. Interpretació gràfica de la suma de vectors.


Hem dit abans que un vector representa una direcció en l’espai. La suma de dos
vectors representa la direcció resultant de la combinació de les seves direccions.
Visualment la podem representar dibuixant el segon vector a continuació del
primer:
Definició. Resta de vectors.
 
Per restar dos vectors v  w se suma a al primer el oposat del segon:
   
v  w  v   w . A la pràctica, restem component a component.
 
Exemple: v  (3,2,7) i w  (1,5,2) , llavors
 
v  w  (3  (1),2  5,7  2)  (4,7,5)

Multiplicació d’un vector per un escalar.


Anomenem escalar a qualsevol nombre real. Per a multiplicar un nombre per un
vector s’ha de multiplicar cada component del vector per aquest nombre. És fàcil
comprovar que aquesta operació multiplica la longitud del vector pel nombre. Si
el signe del nombre és negatiu, s’obté un vector de la mateixa longitud però en
sentit contrari.
 
Exemple: v  (3,2,7) i k  3 , llavors k v  3 (3,2,7)  (9,6,21)

Propietats del producte de vectors per escalars.


 
Associativa : a b w  a b  w
  
Distributiva (I): (a  b) w  a w  b w
   
Distributiva (II): a v  w  a v  a w
 
Producte per la unitat : 1w  w
 
Producte per zero: 0 w  0

1.1.1
   
Calcula u  v , on u  (1,4,3) i v  (5,3,2) .

1.1.2
   
Calcula u  v , on u  (3,0,1) i v  (4,2,3) .

1.1.3
  
Determina el vector v  1.3 u, on u = ( 2 , -3 , -5 )

1.1.4
  
Determina el vector v  0.7 u, on u = ( 1 , 4 , -2 )
1.2 Vectors proporcionals. Punts alineats. Divisió de segments.

Definició. Punts.
Un punt de IR3 és la representació matemàtica d’una posició en l’espai.

Analíticament ve representat per una terna ordenada P  ( p1 , p2 , p3 ) , igual que


els vectors. Per anomenar els punts fem servir lletres majúscules: P, Q, R, ...

Per exemple: P  (4,2,1) , Q  (3,2,0) .

Observació. Representació gràfica de punts.


Els punts es representen per posicions en l’espai, marcades amb una petita
“boleta”. Per exemple, el punt P  ( 3 , 1 , 2 ) :

Atenció! Analíticament els punts i els vectors es representen de la mateixa


manera: amb matrius 1x3. Tanmateix no hem de confondre mai aquestes dues
entitats matemàtiques.

No és el mateix anar en la direcció (1,3,-2) que estar a la posició (1,3,-2). La



primera idea es representa amb un vector v  (1,3,-2) , i la segona amb un punt
P  (1,3,-2).

Els punts no admeten operacions: No es poden sumar punts, ni es pot multiplicar


un punt per un escalar.
1.2.1
Representa en l’espai els següents punts:

a)  4,1,1 b)  1,1,3 c) (3,1,1)

1.2.2
Determina les coordenades dels següents punts:
a) b)

c) d)
Definició. Primera operació entre punts i vectors: Punt + Vector = Punt.
 
Donat un punt P i un vector v , definim el punt final Q  P  v com el punt a on

arribem si sortim de P i anem en la direcció de v :

P  ( p1 , p2 , p3 ) 
   Q  P  v  ( p1  v1 , p2  v2 , p3  v3 )
v  (v1 , v2 , v3 ) 

Definició. Segona operació entre punts i vectors: Punt – Punt = Vector.


 
Donats dos punts P i Q, existeix un únic vector v tal que Q  P  v .
Aquest vector es denota per PQ i s’anomena “vector de desplaçament”.
Es calcula restant component a component (sempre final menys inici):

P  ( p1 , p2 , p3 )
  PQ  Q  P  (q1  p1 , q2  p2 , q3  p3 )
Q  (q1 , q2 , q3 ) 

Gràficament serà la direcció que prenem quan anem des del punt inicial P al
punt final Q.

Atenció! L’ordre importa, no és igual PQ que QP , de fet són contraris l’un de


l’altre:
QP   PQ
Vectors proporcionals.
 
Donats dos vectors v  (v1 , v2 , v3 ) i w  ( w1 , w2 , w3 ) , direm que són
 
proporcionals quan existeixi un k  IR tal que v  k w , és a dir, quan podem
passar d’un a l’altre multiplicant per un escalar:

Visualment dos vectors seran proporcionals quan determinin una mateixa


direcció en l’espai, independentment del seu mòdul i sentit.

Exemple.
   
v  (5 ,  2 , 7) i w  (15,  6 , 21) són proporcionals perquè w  3 v .
 
v  (2 , 1 , 3) i w  (8,  4 ,  11) no són proporcionals, perquè 8  (4)(2) ,
 4  (4) 1 però  11  (4)  3 .

Proposició.
 
Dos vectors u i v són proporcionals si i només si les raons de les seves
components són iguals:

u  (u1 , u2 , u3 )   u1 u2 u3
 u  kv   
v  (v1 , v2 , v3 )  v1 v2 v3

Aquesta caracterització pot ser molt útil, però té l’inconvenient de què ens
podem trobar amb zeros als denominadors, cosa que, fa mal als ulls.
 
Per exemple: Els vectors u  (1,0,2) i v  (2,0,4) són proporcionals perquè
  1 0 2
v  2u . Dividint components apareix un 0/0:    0.5
2 0 4
Punts alineats.
Direm que tres punts A  (a1 , a2 , a3 ) , B  (b1 , b2 , b3 ) i C  (c1 , c2 , c3 ) estan
alineats quan els vectors AB i AC siguin proporcionals: AC  k AB .

Exemple resolt.
Determina els valors de a i b per als què els punts A  (1,1,1) , B  (a,2, b) i
C  (1,0,0) estan alineats.

Solució:
Els punts A, B I C estan alineats quan els vectors AB i AC són proporcionals:
AB  (a,2, b)  (1,1,1)  (a  1,1, b  1)
AC  (1,0,0)  (1,1,1)  (0,1,1)
a  1  0

AB  k AC  (a  1,1, b  1)  k (0,1,1)  (0,k ,k )  1  k
b  1  k

 k  1, a  1, b  2

La solució és a  1, b  2 .

1.2.3
Determina si els punts A  (1,2,1) , B  (2,3,0) i C  (1,0,4) estan o no
alineats.

1.2.4
Determina si els punts A  (3,2,4) , B  (6,1,2) i C  (12,3,6) estan o no
alineats.

1.2.5
Donats els punts A  (2,0,1) , B  (3,2,2) i C  ( x,6,4) , determina el valor
de x que fa que estiguin alineats.
1.2.6
Calcula els valors de a i b per als què els punts A  (1,1,1) , B  ( a , 2 , b ) i
C  (1, 0 , 0 ) estiguin alineats.

1.2.7
Demostra que els punts P=(2,4,6), Q=(-2,-2,-2) i R=(6,10,14) estan alineats.

1.2.8
Determina els valors de a i c per als quals el punt (a,1,c) pertanyi a la recta
determinada pels punts (0,2,3) i (2,7,8).

1.2.9
Els punts A  ( a , 3 , 2 ) , B  ( 4 , b ,  1 ) i C  ( 6 ,  1 , a ) estan alineats.
Determineu els valors d’a i b.
Divisió de segments.

Donats dos punts en l’espai P  ( p1 , p2 , p3 ) i Q  (q1 , q2 , q3 ) , podem trobar el


seu punt mig R:
1
R  P  PQ
2

I amb aquest mateix raonament podem dividir qualsevol segment de qualsevol manera,
com es pot veure en el següent exemple:

Exemple resolt.
Determina el punt P que divideix el segment AB en una proporció 5 / 6 , on
A=(-2,0,6) i B=(10,-6,-12).

Solució:
AB  B  A  (10  (2),6  0,12  6)  (12,6,18)
5 5
P  A  AB  (2,0,6)  (12,6,18)  (8,5,9)
6 6

Determinació de la raó de divisió de segments.

Exemple resolt.
Donats A  (4,6,10) , B  (2,4,6) , C  (14,0,2) , comprova que B està entre A
i C i determina la raó AB/AC.

Solució:
AB  B  A  (6,2,4) 
  AC  3 AB
AC  C  A  (18,6,12)

Els vectors de translació són proporcionals, i per tant els tres punts estan
AB 1
alineats, i es compleix 
AC 3
1.2.10
Donats els punts P  (3,2,4) , Q  (5,4,6) , R  (9,8,10) , determina la raó
PQ
PR

1.2.11
Donats els punts A  (2,3,5) i B  (1,4,6) , determina el punt C del segment
AC 2
AB tal que 
AB 3

1.2.12
Determina els dos punts del segment PQ que el divideixen en tres parts iguals,
on P  (4,2,6) i Q  (10,16,6)

1.2.13
Un segment d’origen en el punt A = (–1, 4, –2) i extrem en el punt B està dividit
en cinc parts iguals mitjançant els punts de divisió A1, A2, A3 i A4 (vegeu la
figura). Si sabem que A2 = (1, 0, 2), quines són les coordenades de B?

Solució PAU CAT TEC JUNY 2005 1.4

1.2.14
a) Si A, B i M són tres punts de l'espai que compleixen la relació AB  2 AM
digueu quin serà el valor de r a l'expressió MA  r MB
b) Si la relació anterior entre vectors s'hagués produït al pla i les coordenades de
A i B fossin respectivament (3, –5) i (–5, 7), quines serien les coordenades del
punt M? Justifiqueu la resposta.

PAU CAT TEC SET 1997 1A.2


Punt simètric d’un punt respecte d’un altre.

Donats dos punts en l’espai P  ( p1 , p2 , p3 ) i Q  (q1 , q2 , q3 ) , podem trobar el


punt R que fa que Q sigui el punt mig entre P i R:

P  ( p1 , p2 , p3 )
  R  P  2 PQ
Q  (q1 , q2 , q3 ) 
1.3 Mòdul d’un vector. Distància entre dos punts.

Definició. Mòdul d’un vector.



Definim el mòdul (o “longitud”) d’un vector v  (v1 , v2 , v3 ) per

v  v12  v22  v32

Propietats bàsiques del mòdul d’un vector.



a) v  0
 
b) v  0  v  0
 
c) k v  k v

La desigualtat triangular de Cauchy-Schwarz.


La norma de la suma de vectors és sempre menor o igual que la suma de les
normes de cadascun dels vectors:
   
u v  u  v

Demostració. Vegeu apartat 10.3.

Observació.
La desigualtat triangular de Cauchy-Schwarz garanteix que en tot triangle la
longitud d’un costat és sempre menor que la suma dels seus altres dos costats, o
dit amb paraules senzilles, que la línia recta és sempre el camí més curt entre dos
punts.

Distància entre dos punts.


La distància entre dos punts P  ( p1 , p2 , p3 ) i Q  (q1 , q2 , q3 ) serà el mòdul del
vector que determinen:

PQ  dist ( P, Q)  PQ  (q1  p1 )2  (q2  p2 )2  (q3  p3 )2


Exemple resolt.
Determina la distància PQ entre els punts P  (1,3,4) i Q  (4,1,2)

Solució:
PQ  Q  P  (5,4,2)
PQ  PQ  (5)2  42  (2)2  25  16  4  45  3 5  6.71

1.3.1
Determina la distància entre els punts P = ( 14 , 1 , 17 ) i Q = ( 9 , -4 , 13 ).

1.3.2
Determina la distància entre els punts P = ( 4 , 2 , 7 ) i Q = ( -16 , -10 , 3 )

1.3.3
Els punts A= ( 5 , -1 , 1 ) , B= ( 7 , -4 , 7 ) , C= ( 1 , -6, 10 ) i D= (-1 , -3 , 4 ) són
coplanaris. Demostra que formen un rombe.

1.3.4
a) Demostra que els punts A  (  , 2 ,  ) , B  (2 ,   , 0 ) i C  ( ,0 ,   2 ) són
els vèrtexs d’un triangle isòsceles.
b) Determina el valor de  per al qual el triangle ABC és equilàter.

1.3.5
Determineu el punt de l’eix Y que està a una distància 10 del punt ( 1 , 2 , 3 )

1.3.6
Determina el punt de l’eix X que és equidistant als punts A  ( 3 , 2 , 2 ) i
B  ( 5, 5, 4 )

1.3.7
Determina el punt de l’eix Z que equidista dels punts A=(2,3,-4) i B=(6,-2,-1)
P  (0,0,2)
Equidistància a tres punts.

Exercici resolt.
Determina les coordenades del punt del pla x  0 (és a dir, és de la forma
P  (0, y, z ) ) que equidista dels tres punts A=(2,0,3), B=(0,3,2) i C=(0,0,1)

Solució:
PA  PB  (0  2) 2  ( y  0) 2  ( z  3) 2  (0  0) 2  ( y  3) 2  ( z  2) 2  z  3 y  0
PB  PC  y 2  9  6 y  z 2  4  4 z  y 2  z 2  1  2 z  3 y  z  6
z  3 y  0
  y  1, z  3  P  (0,1,3)
3 y  z  6

1.3.8
Determina les coordenades del punt del pla z  0 (és a dir, és de la forma
P  ( x, y,0) ) que equidista dels tres punts A=(3,2,-5), B=(5,1,7) i C=(4,-3,-6).

Xavi Mates: Vectors 3D


https://youtu.be/3EvOF7v_y_c
1.4 Fora de programa: La desigualtat triangular.
En tot triangle, la longitud d’un costat ha de ser estrictament menor que la suma dels
altres dos costats, i si es compleix la igualtat és que estan alineats (Elements, 1.20)

Exemple resolt.
Demostra que els punts P  (2,3,5) , Q  (1,2,3) , i R  (7,0,1) estan alineats.

Primer mètode: Mitjançant proporcionalitat dels vectors translació:

PQ  Q  P  (3,1,2)
  PR  3PQ  PR // PQ
PR  R  P  (9,3,6) 

Els vectors directors són proporcionals, i per tant els punts estan alineats.

Segon mètode: Mitjançant la desigualtat triangular.

PQ  PQ  3,1,2  32   1   2  9  1  4  14
2 2

QR  QR  6,2,4  62   2   4  6  4  16  56  2 14
2 2

PR  PR  9,3,6  92   3   6  81  9  36  126  3 14
2 2

I observem que PQ  QR  14  2 14  3 14  PR , per tant, estan alineats

1.4.1 Exercici.
Demostra que els punts (-2,3,5), (1,2,3) i (7,0,-1) estan alineats, de dues maneres
diferents: Amb vectors translació proporcionals i aplicant la desigualtat
triangular, com en l’exemple anterior.
Nota històrica.
Aquest teorema s’anomena també “el teorema dels rucs”, perquè ja Proclus, un
comentarista dels Elements d’Euclides del segle IV, comenta que els epicurs
acostumaven a ridiculitzar aquest teorema dient que “era evident fins i tot per a
un ruc, i no requeria prova”. En efecte, si col·loquem un ruc en un vèrtex A
d’un triangle i farratge en B, el ruc anirà directament pel camí més curt, sense
passar pel tercer vèrtex C.
1.5 Producte escalar de dos vectors. Angle.

Producte escalar de dos vectors.


 
Donats dos vectors v  (v1 , v2 , v3 ) i w  ( w1 , w2 , w3 ) , es defineix el seu producte
escalar com:
 
v  w  v1w1  v2 w2  v3 w3

Desigualtat de Schwarz.
   
Sempre es compleix v w  v  w

Demostració. Al final del document.

Angle entre dos vectors.


 
    v w
Sabem que v  w  v  w i per tant  1     1
vw

Aquesta propietat ens permet definir l’angle entre dos vectors com:

 
  v w
  cos(v , w)   
vw

Corol·lari. Vectors perpendiculars mitjançant el producte escalar.


 
Dos vectors v  (v1 , v2 , v3 ) i w  ( w1 , w2 , w3 ) són perpendiculars (o ortogonals)
 
(i escriurem v  w ) quan el angle que determinen és de 90º, és a dir, quan
formin un angle recte.
 
Llavors cos(v , w)  cos(90º )  0 i per tant, aplicant 1.5.2 tenim
 
  v w        
0  cos(v , w)     v  w  0  cos(v , w)  v  w  0  v  w
vw

és a dir, dos vectors (no nuls) seran perpendiculars si i només si el seu producte
escalar és zero:

   
v  wvw0
Exemple resolt.
 
Determina el valor de a de forma que els vectors u  (2, a,1) i v  (1, a,1)
siguin perpendiculars.

Solució:
   
u  v u v  0
 
u  v  2(1)  a  a  1  1  a 2  2  1  0  a 2  1  a  1
Les solucions son a  1 i a  1

Proposició.
  
Com que u cos( ) és la projecció escalar del vector u sobre el vector v , el
producte escalar es pot entendre com el producte d’aquesta projecció per la

longitud de v .

Proposició.
  
v  v v

Demostració. Només cal desenvolupar les definicions:


  
v  v  (v1 , v2 , v3 )  (v1 , v2 , v3 )  v1  v2  v3  v
2 2 2

1.5.1
 
Determineu k per a què els vectors a  (3, k ,2) i b  (k ,9,6) siguin:
a) perpendiculars,
b) paral·lels.

1.5.2

Determina l’angle, en graus, entre els dos vectors v  (1,5,1) i

w  (2,3,4) .

1.5.3

Determina l’angle, en graus, entre els dos vectors v = ( 5 , -2 , 1 ) i

w = ( 4 , -4 , -1 ).

1.5.4
  
Donats els vectors u  (1, 1, 0) i v  (a, 1,  1) , trobeu el valor d’a per al què u i

v formen un angle de 60º.
1.5.5
Sabent que ABCD és un quadrat, A  ( 2, 0, 2 ) , B  (1, 1, 0) i C  (0, y, z ) ,
trobeu raonadament les coordenades y i z de C.

1.5.6
  
Donats els vectors v1  (a  1,2a,1) , v2  (2, a,2a) i v3  (a,2,4a  2) de IR3:
 
a) Calculeu l’angle que formen v1 i v2 quan a= 0.
  
b) Trobeu el valor del paràmetre a perquè els vectors v1 , v2 i v3 siguin
perpendiculars dos a dos.
Solució PAU CAT TEC SET 2009 1.4

1.5.7
Considereu els punts de l’espai A= (0, –2a–1, 4a–2), B= (1, –3, 4), C= (3, –5, 3).
a) Comproveu que el triangle de vèrtexs A, B i C és rectangle en B per a
qualsevol valor de a.
b) Calculeu els valors de a que fan que aquest triangle sigui isòsceles.

Solució PAU CAT TEC SET 2003 3.4

1.5.8
Els punts A(k–3, 2, 4), B(0, k+2, 2) i C(–2, 6, k+1) són tres dels vèrtexs d’un
rombe ABCD (vegeu la figura).

a) Calculeu el valor de k.
b) Demostreu que el rombe és un quadrat.
Solució PAU CAT TEC JUNY 2004 1.4

1.5.9
 
Siguin u i v els dos vectors del pla :
  1
u  (1,1) v  (1  3,1  3 )
2
 
Calculeu l’angle que formen u i v .

PAU CAT TEC SET 1998 5.3


Fora de programa: El Teorema de Pitàgores.
El teorema de Pitàgores estableix que en un triangle rectangle la suma dels
quadrats dels catets (els costats que formen l'angle recte) és igual al quadrat de la
hipotenusa

El recíproc també es compleix, és a dir: en un triangle, si la suma dels quadrats


de les longituds dels costats més curts és igual al quadrat de la longitud del
costat més llarg, llavors l'angle comprès entre els dos costats més curts és un
angle recte.

AB  AC  AB2  AC 2  BC 2

Exemple resolt.
Demostra que els punts A  ( 2 , 1 , 7 ) , B  ( 1 ,  1 , 3 ) , C  ( 4 , 2 , 6 ) són els
vèrtexs d’un triangle rectangle.

Solució.
Primera versió: Mitjançant vectors:
AB  (  1 ,  2 ,  4 )

  AB  AC  (1)  2  (2) 1  (4)(1)  2  2  4  0
AC  ( 2 , 1 ,  1 ) 

Per tant AB  AC i el triangle és rectangle en A.

Segona versió: Aplicant el Teorema de Pitàgores:

AB  (  1 ,  2 ,  4 )  AB  (1) 2  (2) 2  (4) 2  1  4  16  21


AC  ( 2 , 1 ,  1 )  AC  2 2  12  (1) 2  4  1  1  6
BC  ( 3 , 3 , 3 )  BC  32  32  32  9  9  9  27

I observem que BC 2  27  21  6  AB2  AC 2 , és a dir, es compleix el


Teorema de Pitàgores.

1.5.10
Demostra que els punts P  ( 0 , 7 , 10 ) , Q  (  1 , 6 , 6 ) i R  (  4 , 9 , 6 )
formen un triangle rectangle isòsceles.
1.6 Producte vectorial de dos vectors. Àrea.
Producte vectorial.
 
Donats dos vectors v  (v1 , v 2 , v3 ) i w  ( w1 , w2 , w3 ) , es defineix el producte
vectorial dels dos vectors com el vector:

i j k
   v v3 v v3 v1 v2 
v  w  v1 v2 v3   2 , 1 , 

w w3 w1 w3 w1 w2
w1 w2 w3  2 

Propietat fonamental del producte vectorial.


   
El vector u  v és perpendicular als dos vectors u i v , i el seu sentit és el sentit
 
de desplaçament d’un tirabuixó que va de u cap a v per l’angle més petit.

La millor manera d’obtenir un vector perpendicular a altres dos donats és


calculant el seu producte vectorial.

1.6.1
   
Determina el producte vectorial a  b , on a  (1,4,1) , b  (2,4,6) .

1.6.2
   
Determina el producte vectorial a  b , on a  (3,2,5) , b  (1,4,6) .

1.6.3
Comprova amb els vectors (2,2,1) i (1,2,1) si el producte vectorial compleix
la propietat commutativa.

1.6.4
 
Donats els vectors u  (1, 2, 1) i v  (2, 1, 3) , trobeu un vector perpendicular a
tots dos i de mòdul 3.

1.6.5
  
Siguin els vectors u  (1,1,3) , v  (2,2,1) i w  (3,2,5) . Calculeu:
       
a) u  (v  w) b) u  (v  w) c) L’angle que formen u i v .

Altres propietats del producte vectorial.


    
1. u  v  u v sin(u , v )
2. El producte vectorial de dos vectors linealment dependents és nul.
Producte vectorial i àrea.

a) L’àrea del paral·lelogram que determinen dos vectors serà igual al mòdul del
producte vectorial:
 
Àrea  v  w

 
Per exemple, els vectors v  ( 3 , 2 ,1) i w  (1, 4 , 3 ) determinen un
paral·lelogram en l’espai:

La seva àrea serà:

i j k
 
v  w  3 2 1   2 ,  8 , 10 
1 4 3

Àrea   2 ,  8 , 10  
 2 2  (8) 2  10 2  2 42  12.96

b) Dos vectors determinen un triangle, que serà la meitat del paral·lelogram que
determinen, per tant, l’àrea del triangle determinat per dos vectors serà la meitat
del mòdul del producte vectorial dels dos vectors:

1  
Àrea  vw
2

Amb l’exemple anterior:

1   1
Àrea  v  w  2 42  42  6.48
2 2
1.6.6
Determina l’àrea del paral·lelogram definit pels vectors
 
v = ( -2 , -1 , 2 ) i w = ( -1 , 0 , 4 )

1.6.7
Determina l’àrea del paral·lelogram definit pels vectors
 
v = ( 3 , -4 , 1 ) i w = ( 5 , 0 , 2 )

1.6.8
Tenim els punts A  (1,0,0) , B  (2,1,2) i C  (1,2,2) . Si sabem que ABCD és
un paral·lelogram, trobeu el punt D, i el seu perímetre i àrea.

1.6.9
Determina el valor de a de forma que els punts A  (1,0,1) , B  (1,1,1) i
C  (1,6, a) siguin els vèrtexs d’un triangle d’àrea 3 / 2 .

Tres condicions equivalents.


Donats dos vectors en l’espai, són equivalents:
a) Els dos vectors són linealment dependents.
b) Els dos vectors generen un paral·lelogram d’àrea 0.
c) El producte vectorial dels dos vectors és 0.

L’apartat 8.1 està dedicat als problemes PAU relacionats amb l'àrea.

Xavi Mates: Producte vectorial


https://youtu.be/QxGSICD-OPk

MateFacil:
https://youtu.be/8_L_LhFYN40

Pablo Samsó Aparici:


https://youtu.be/W1HFLJhK_YQ
1.7 Producte mixt de tres vectors. Volum. Punts coplanaris.

Producte mixt de tres vectors.


  
Donats tres vectors u  (u1 , u 2 , u 3 ) v  (v1 , v 2 , v3 ) i w  ( w1 , w2 , w3 ) , el
producte mixt dels tres vectors és el determinant de la matriu que hi generen:

u1 u2 u3
  
det(u , v , w)  v1 v2 v3
w1 w2 w3

Producte mixt i volum.


1. El valor absolut del producte mixt de tres vectors és igual al volum del
paral·lepípede que determinen.

  
Volum | det(u , v , w) |

(observa que la fórmula porta valor absolut)

2. El volum del tetràedre desenvolupat per tres vectors és igual a un sisè del
valor absolut del producte mixt dels tres vectors (ja que el paral·lepípede
correspon a 6 tetràedres).

1   
Volum  | det(u , v , w) |
6

1.7.1
  
Determina el volum del paral·lelepípede generat pels vectors u, v i w.
  
u = ( 1 , -6 , -2 ), v = ( 3 , -4 , -3 ), w = ( -5 , -2 , 2 )

1.7.2
  
Determina el volum del paral·lelepípede generat pels vectors u, v i w.
  
u = ( 3 , 1 , -1 ), v = ( -1 , -2 , 3 ) , w = ( 3 , -5 , -3 )
Quatre condicions equivalents.
  
Donats tres vectors u  (u1 , u2 , u3 ) v  (v1 , v2 , v3 ) i w  ( w1 , w2 , w3 ) , és
equivalent:
a) Són coplanaris.
b) Són linealment dependents.
c) El paral·lepípede que generen té volum 0, o dit amb paraules senzilles,
quan generen en l’espai una “capsa plana”.
  
d) det(u , v , w)  0

Quatre punts coplanaris.


Direm que quatre punts A, B, C i D són coplanaris quan els vectors AB , AC i
AD siguin coplanaris, és a dir, quan tinguin determinant zero:

A, B, C i D són coplanaris  det( AB, AC, AD)  0

1.7.3
Calcula el volum del prisma amb arestes definides per els vectors (4,1,1) , (2,12)
i (3,1,1) .

1.7.4
Si ABCDEFGH és un paral·lepípede, i sabent que A  (0, 1, 1) , B  (2, 1, 1) ,
C  (1, 1, 3) i E  (2, 0, 1) , trobeu la resta de vèrtexs i el seu volum.

1.7.5
Determina si els següents punts són o no coplanaris:
a) A  (1,0,1) , B  (3,2,0) , C  (1,1,2) , D  (3,2,4)
b) A  (0,1,5) , B  (4,6,1) , C  (2,2,1) , D  (2,1,0)
1.7.6
Determina x de forma que els següents quatre punts siguin coplanaris:
a) A  (0,0,1) , B  (0,1,2) , C  (2,1,3) , D  ( x, x  1, 2)
b) A  (x,0,1) , B  (0,1,2) , C  (1,2,3) , D  (7, 2, 1)

1.7.7
Les coordenades dels vèrtexs consecutius d’un paral·lelogram són
A  ( 1 , 0 , 0 ) i B  ( 0 , 1 , 0 ) . Les coordenades del centre M són
M  ( 0 , 0 , 1 ) . Trobeu les coordenades dels vèrtexs C i D.

1.7.8

Determineu el volum del paral·lelepípede format pels vectors u  ( 3 ,  2 , 5 ) ,
 
v  ( 2 , 2 , 1 ) i w  (  4 , 3 , 2 ) .

1.7.9
Determineu el volum del tetraedre de vèrtexs A=(3, 2, 1), B=(1, 2, 4) ,
C=(4, 0, 3) i D=(1, 1, 7).

1.7.10
Donats els punts A=(0,5,3), B=(0,6,4), C=(2,4,2) i D=(2,3,1), comproveu que els
quatre punts són coplanaris i que el polígon ABCD és un paral·lelogram.

1.7.11
Donats els punts A=(0,0,0), B=(1,0,−1), C=(0,1,−2) i D= (1,2,0), es demana
demostrar que els quatre punts no són coplanaris i calcular el volum del tetraedre
que formen.

1.7.12
  
Considerem els vectors u  ( 2 , 3 , 4 ) , v  (  1 ,  1 ,  1 ) i w  (  1 ,  ,  5 ) .
a) Determina els valors de  per als què el paral·lelepípede que formen els tres
vectors tingui volum igual a 6 unitats cúbiques.
b) Determina els valors de  per als què són tres vectors linealment dependents
(és a dir, coplanaris).

1.7.13
Demostra que els quatre punts A=(1,0,-1), B=(2,1,0), C=(0,0,-1) i D=(-1,1,1) no
són coplanaris.

1.7.14
Determina els valors d’a i b per als què els punts A=(1,0,0), B=(a,b,0), C=(a,0,b)
i D=(0,a,b) siguin coplanaris.
1.7.15
Calcula el volum del tetraedre de vèrtexs A(1,1,1), B=(0, −2,2), C=(−1,0,2) i
D=(2, −1,2).

1.7.16
Donats els punts A=(1, 3, −1), B=(a, 2, 0), C=(1, 5, 4) y D=(2, 0, 2),
a) Trobeu el valor de a per al què els quatre punts siguin coplanaris.
b) Trobeu els valors de a per als quals el tetraedre de vèrtexs A,B, C i D tingui
volum igual a 7.
PAU Madrid 2012

L’apartat 8.2 està dedicat als problemes PAU relacionats amb el volum.

Xavi Mates: Producte mixt


https://youtu.be/5C0PbQ9IDPE

Unicoos: Producto mixto de vectores ... volumen paralelepipedo...


https://youtu.be/SeroDDEq7SI

Xavi Mates: 4 punts coplanaris


https://youtu.be/SeodTqmmNto
1.8 Relació entre els tres productes.
Els tres productes que hem estudiat: Producte escalar, producte vectorial i
producte mixt estan relacionats amb la següent igualtat:
     
det(u , v , w)  u  v  w
Demostració.
i j k
   v v3 v1 v3 v1 v2 
u  v  w  (u1 , u2 , u3 )  v1 v2 v3  (u1 , u2 , u3 )   2 , , 
 w2 w3 w1 w3 w1 w2 
w1 w2 w3
u1 u2 u3
v v3 v v3 v v2   
 u1 2  u2 1  u3 1  v1 v2 v3  det(u , v , w)
w2 w3 w1 w3 w1 w2
w1 w2 w3

1.8.1
          
Donats els vectors u , v i w , calcula v  w , u  v  w i det(u , v , w) i comprova
     
que es compleix la propietat u  v  w  det(u , v , w) .
  
a) u  ( 3,1 ,  4 ) , v  (  2 , 0 , 7 ) , w  ( 5 ,  3 ,  1 ) .
  
b) u  (  2 , 0 ,1 ) , v  ( 0 , 7 , 3 ) , w  ( 5 , 1 ,  4 ) .
  
c) u  ( 3 ,  1, 5 ) , v  ( 5 ,  6 , 0 ) , w  ( 0 ,  2 , 3 ) .
1.9 Fora de programa: L’esfera.

Definició. Esfera.
Donat un punt C  (c1 , c2 , c3 ) i un radi r  0 , l’esfera de centre C i radi r serà el
lloc geomètric de tots els punts P  ( x, y, z ) tals que la seva distància al centre C
és igual a r, és a dir,
dist ( P, C )  r  PC  r

Naturalment, l’esfera en l’espai és l’equivalent a la circumferència en el pla.

Proposició. Equació de l’esfera.


Donat el centre C  (c1 , c2 , c3 ) i el radi r  0 , el punt P  ( x, y, z ) pertany a
l’esfera si i només si
dist ( P, C )  r  PC  r  PC  r  PC  r

CP  P  C  ( x  c1 , y  c2 , z  c3 )
CP  ( x  c1 ) 2  ( y  c2 ) 2  ( z  c3 ) 2
Per tant :
PC  r  ( x  c1 ) 2  ( y  c2 ) 2  ( z  c3 ) 2  r
( x  c1 ) 2  ( y  c2 ) 2  ( z  c3 ) 2  r 2

La part de l’esquerra d’aquesta equació la podem desenvolupar en forma de


polinomi en x, y i z per arribar a una equació de la forma

x 2  y 2  z 2  Ax  By  Cz  D  0

Un cop més, recorda que NO has de memoritzar cap d’aquestes fórmules. Has
d’estudiar, aprendre i entendre com passar d’una a l’altra sense haver de mirar
els apunts.
Problema resolt.
Donats dos punts de l’espai A  (0,0,1) i B  (0,2,3) , determineu el lloc
geomètric de tots els punts P tals que AP  BP .

Solució:
En primer lloc anem a fer un estudi geomètric del problema:

Si ens limitem a un pla, els punts P tals que AP  BP són els punts de la
semicircumferència de diàmetre AB . Aquest resultat és la proposició 31 del
Llibre III dels Elements d’Euclides. (Vegeu
www.toomates.net/biblioteca/AlgebraLineal.pdf Apartat 9.1.4)

Com que estem en l’espai, hi ha moltes més solucions: Aquesta


semicircumferència la podem girar pel seu eix per obtenir una circumferència de
diàmetre AB, formada per punts que també satisfaran la condició de
perpendicularitat:

El lloc geomètric solució del problema serà, doncs, la circumferència de


diàmetre AB, és a dir, aquella que té centre el seu punt mig M i radi AB / 2 .
Anem ara a resoldre aquest mateix problema amb coordenades :

A  (0,0,1) 
  AP  P  A  ( x, y, z  1)
B  (0,2,3)   

P  ( x, y, z )  BP  P  B  ( x, y  2, z  3)

AP  BP  AP  BP  0  ( x, y, z  1)  ( x, y  2, z  3)  0 
x 2  y ( y  2)  ( z  1)( z  3)  0 
x 2  y 2  2 y  z 2  3z  z  3  0 
x2  y2  2 y  z 2  4z  3  0

Comprovem que es tracta de la circumferència de diàmetre AB:

Punt mig del segment AB:


1 1
AB  B  A  (0,2,2)  AB  (0,1,1)  M  A  AB  (0,0,1)  (0,1,1)  (0,1,2)
2 2

Diàmetre de la circumferència:
d  AB  02  22  22  8  2 2

Radi de la circumferència :
d 2 2
r   2
2 2

Equació de la circumferència:
MP  P  M  ( x, y, z )  (0,1,2)  ( x, y  1, z  2)
d ( M , P)  MP  x 2  ( y  1) 2  ( z  2) 2

2  d ( P, M )  2  x 2  ( y  1) 2  ( z  2) 2 
2  x 2  ( y  1) 2  ( z  2) 2 
2  x2  y 2  2 y  1  z 2  4z  4 
0  x2  y 2  2 y  1  z 2  4z  4  2 
0  x2  y 2  2 y  z 2  4z  3

I efectivament arribem a la mateixa equació.


1.10 Notes històriques. La història dels vectors.
La llei del paral·lelogram per a l’addició de vectors és tan intuïtiva que el seu
origen és desconegut. Podria haver aparegut en un treball ara perdut d’Aristòtil
(384-322 aC), i es troba en la Mecànica de Hieró d’Alexandria (segle I dC). Va
ser, també, un dels primers resultats del Principia Mathematica (1687) d’Isaac
Newton (1642-1727). En els Principia, Newton va tractar de manera extensa el
que ara es consideren les entitats vectorials (per exemple, velocitat, força), però
mai el concepte de vector. L’estudi i l’ús de vectors no es va sistematitzar fins
als segles XIX i XX.

Els vectors van sorgir a les primeres dues dècades del segle XIX amb les
representacions geomètriques de nombres complexos. Caspar Wessel (1745-
1810), Jean Robert Argand (1768-1822) i Carl Friedrich Gauss (1777-1855)
van concebre nombres complexos com a punts en el pla de dues dimensions, és a
dir, com a vectors de dues dimensions. En 1837, William Rowan Hamilton
(1805-1865) va demostrar que els nombres complexos es podrien considerar
parells de nombres (a, b). Aquesta idea era una part de la campanya de molts
matemàtics, incloent-hi el mateix Hamilton, per a buscar una manera d’ampliar
els "nombres de dues dimensions" a tres dimensions.
En 1827, August Ferdinand Möbius va publicar un llibre curt, Càlcul
baricèntric, en el qual va introduir el segment dirigit que va denotar amb les
lletres de l’alfabet; ja eren vectors, encara que no tenien aquest nom. En el seu
estudi de centre de gravetat i la geometria descriptiva, Möbius va desenvolupar
el càlcul amb aquests segments dirigits; els va sumar i va demostrar com es
multiplicaven per un nombre.

William Rowan Hamilton (1805-1865)

Finalment, el mateix Hamilton va introduir en 1843 en concepte de vector,


precisament com un segment orientat de l’espai.
El desenvolupament de l’àlgebra de vectors i de l’anàlisi de vectors tal com el
coneixem avui va ser fet per primera vegada per J. Willard Gibbs (1839-1903)
en les classes per als seus estudiants en la Universitat de Yale. Gibbs va intuir
que els vectors proporcionarien una eina més eficient per al seu treball en la
física. Així, doncs, començant el 1881, Gibbs va imprimir en privat notes sobre
anàlisi dels vectors per als seus estudiants, que van ser distribuïts extensament
entre els erudits dels Estats Units i d’Europa.
1.11 Recopilatori d’exercicis.

1.11.1
Donats els punts A  (1,1,0) , B  (1, b,4) , C  (4,10, c) , D  (1,2,3) i
E  (0,5, e) , trobeu les coordenades que falten en B, C i D sabent que A, B i C
estan alineats i A, B, D i E són coplanaris.

1.11.2
Determineu els punts que divideixen el segment AB en 5 parts iguals, suposant
que A  (2,3,6) i B  (13,6,6)

1.11.3
Donats els quatre punts A  (1,2,3) , B  (b,1,3) , C  (2, c,4) i D  (5,2, d )
a) Trobeu b, c i d sabent que ABCD és un paral·lelogram.
b) Trobeu el centre del paral·lelogram.
c) Trobeu els punts que divideixen el segment CD en 3 parts iguals.

1.11.4
Els punts A  (a,3,2) , B  (4, b,1) i C  (6,1, a) estan alineats. Trobeu els
valors d’A i B.

1.11.5
Trobeu k per a què els 4 punts següents siguin coplanaris:
P  (1,3,1) , Q  (2,0,1) , R  (0,0,1) i S  (k ,1,2)

1.11.6
Trobeu k per a què els 4 punts següents siguin coplanaris:
P  (3,1,0) , Q  (2,2,1) , R  (0,3,1) i S  (1,2, k )

1.11.7
Si sabem que ABCD és un quadrat, A  (2,0, 2 ) , B  (1,1,0) , C  (0, y, z ) ,
trobeu les coordenades que falten de C.

1.11.8
 
Donats els vectors u  (1,2,1) i v  (2,1,3) , trobeu un vector perpendicular a u

i v de mòdul 3.
1.11.9 
 
Donats els vectors a  (1,1,3) , b  (2,0,3) i c  (7,2, z) , trobeu:
 
a) els mòduls de a i b .
 
b) L’angle que formen a i b .
  
c) La coordenada que falta en c , sabent que c  a .

1.11.10
  
Donats els vectors a  (2,1,3) , b  (2,3,1) i c  (0,1,5) , trobeu:
 
a) L’àrea el paral·lelogram determinat per a i b .
  
b) El volum del paral·lepípede determinat per a , b i c .

1.11.11
Trobeu l’àrea del triangle ABC , on A  (1,2,3) , B  (2,0,4) i C  (10,5,0) .

1.11.12
a) Si A  (3,0,1) , B  (6,4,5) i C  (5,3, z) , trobeu z si sabem que el triangle
ABC és rectangle, on A  90º .
b) Trobeu la seva àrea.

1.11.13
 
Donats u  (3,4,12) i v  (5,2,6) , calcula:
 
a) u  v
 
b) u i v.
 
c) L’angle entre u i v .
 
d) Quant ha de valer y perquè w  (7, y,2) sigui perpendicular a u ?

1.11.14

Troba els vectors de IR3 que són perpendiculars a v  (1,0,1) i formen un angle
 1 2 1
de 60º amb v   , ,  .
2 2 2

1.11.15
  
Troba el valor de m perquè els vectors u  (3,5,1) , v  (2,1,1) i w  (1,4, m)
determinin un paral·lelepípede de volum 11.

1.11.16
Donats els punts A  (2,1,1) , B  (0,0,1) i C  ( ,0,0) , troba el valor de  tal
que l’àrea del triangle ABC sigui 2.
1.11.17
  
Donats els vectors u  (a, b,1) , v  (3,4,1) i w  (1,2, c) , determina els valor
 
dels paràmetres a, b, c  IR de manera que els vectors v i w siguin
    
perpendiculars i que, a més, u  w  v . Quin angle formen u i v en aquest cas?

1.11.18
Demostreu que els tres punts (1,1,1) , (2,0,1) i ( 0,2,1) són sobre una mateixa
recta.

1.11.19
  
Considerem els vectors u  (1,1,1) , v  (2,2, a) i w  (2,0,0) .
  
a) Determina els valors de a per als què els vectors u , v i w son coplanaris.
   
b) Determina els valors de a per als què els vectors u  v i u  v son
perpendiculars.

1.11.20
Siguin els punts A  (1,2,1) , B  (2,3,1) , C  (0,5,3) i D  (1,4,3) ,
a) Demostreu que els quatre punts són coplanaris.
b) Demostreu que ABCD és un rectangle.
c) Calculeu l’àrea d’aquest rectangle.

1.11.21
Calculeu un vector de mòdul 1 que sigui perpendicular als vectors (1,0,2) i
(2,1,0) .

1.11.22
  
Donats els vectors u  (a,1  a,2a) , v  (a,1, a) i w  (1, a,1) , trobeu els valors
de a per als què els tres vectors siguin coplanaris.

1.11.23
Se consideren els punts A  (1, a,0) , B  (1,1, a  2) i C  (1,1, a) .
a) Comproveu que no estan alineats, sigui quin sigui el valor del paràmetre a.
b) Trobeu l’àrea del triangle que determinen aquests punts.

1.11.24
Considerem el tetraedre de vèrtexs A=(1, 0, 0), B=(1, 1, 1), C=(-2, 1, 0) y D=(0,
1, 3).
a) Trobeu l’àrea del triangle ABC.
b) Trobeu el volum del tetraedre ABCD.
1.11.25
Els punts A  (1,1,1) , B  (2,2,2) i C  (1,3,3) són tres vèrtexs consecutius d’un
paral·lelogram.
a) Trobeu les coordenades del quart vèrtex D.
b) Calculeu el seu àrea.
c) Comproveu que es tracta d’un rombe i no d’un rectangle.

1.11.26
Donats els punts A  (2,0,2) , B  (3,4,1) , C  (5,4,3) y D  (0,1,4) ,
a) Calculeu l’àrea del triangle de vèrtexs A, B i C.
b) Calculeu el volum del tetraedre (piràmide) ABCD.

1.11.27
Donats els punts A  (1,3,1) , B  (a,2,0) , C  (1,5,4) i D  (2,0,2) , es
demana:
a) Trobeu el valor de a per a què els quatre punts siguin coplanaris.
b) Trobeu els valors de a per a què el tetraedre (piràmide) amb vèrtexs A,B,C,D
tingui un volum igual a 7.

1.11.28
Donats els punts A  (1,3,0) , B  (3,1,2) , C  (7,2,3) , D  (5,2,5) .
a) Demostreu que els punts A, B, C i D són coplanaris.
b) Demostreu que el polígon ABCD és un paral·lelogram i calculeu la seva àrea.

1.11.29
Donats els punts A  ( ,2,  ) , B  (2, ,0) i C  (,0,   2) , existeix algun
valor de  per al què els punts A, B i C estiguin alineats? Comproveu que els
punts sempre formen un triangle isòsceles.

Xavi Mates: Preguntes sobre un paral·lelogram


https://youtu.be/R1at4Rme858

Unicoos: Producto ESCALAR y VECTORIAL de dos vectores...


https://youtu.be/6ABR37--Yr4
1.12 Taula resum dels tres productes.

Interpretació
Fórmula Equació igual a zero
geomètrica

Angle:
Producte escalar

 
  v w
cos(v , w)    Perpendicularitat:
  vw
v  w  v1w1  v2 w2  v3 w3
   
vw0 v  w

 
v  w és perpendicular a
 
v i w:
Producte vectorial

Vectors proporcionals:
i j k
     
v  w  v1 v2 v3 vw  0  v  k w
w1 w2 w3
(no s’utilitza habitualment)

L’àrea del paral·lelogram


 
és igual a v w

El volum del
paral·lelepípede és igual a
Producte mixt

u, v, w 
Vectors coplanaris:
u1 u2 u3
  
u , v , w  v1 v2 v3
w1 w2 w3 u, v, w   0  u, v, w coplanaris
(és a dir, linealment dependents)
2 Varietats lineals en l’espai: Rectes i plans.
2.1 Combinacions lineals de vectors. Subespais vectorials.
Combinació lineal de vectors. Subespai vectorial.
 
Una combinació lineal dels vectors v1 ,, vn  és qualsevol vector de la forma

  
v  1v1    n vn per a certs escalars 1 ,, n  IR

En particular, el vector 0 sempre és combinació lineal de qualsevol conjunt de
vectors:
  
0  0v1    0vn
 
Anomenem subespai vectorial determinat pels vectors v1 ,, vn  al conjunt de
totes les possibles combinacions lineals de vectors
  
v  1v1    n vn

Combinacions lineals
 d’un vector. Rectes vectorials.
Direm que t és combinació lineal del vector u quan existeix un nombre real a
tal que
 
t  au

El conjunt de totes les combinacions lineals d’un vector u  0 forma el que
s’anomena una recta vectorial:
 
u   a u , a  IR 

En particular, el vector zero sempre és combinació lineal de qualsevol vector,


perquè
 
0  (0,0,0)  0u
2.1.1
  
Donat u  (2,3,4) , calcula 2u i  3u .

2.1.2 
 
Donat u  (4 , 2, 3) , escriu t  (12 , 6, 9) com a combinació lineal de u .
Combinacions lineals de dos vectors. Plans vectorials.
  
Direm que t és combinació lineal dels vectors u i v quan existeixen nombres
reals a i b tals que
  
t  au  bv
 
El conjunt de totes les combinacions lineals dels vectors u i v formen el que
s’anomena un pla vectorial:
   
u , v   au  bv , a, b  IR 

 
Però atenció! No sempre! Només si els vectors u i v no són proporcionals, és a
 
dir, quan u  k v .
 
Direm que els vectors u i v són linealment independents quan formin un pla
vectorial, és a dir, quan no siguin proporcionals. Si dos vectors són
proporcionals formen una recta vectorial. Podem dir que “sobra” un d’ells.
En particular, el vector zero sempre és combinació lineal de qualsevol parella de
vectors, perquè
  
0  (0,0,0)  0u  0v

Exercici resolt.

Escriu el vector (12,11,13) com a combinació lineal dels vectors u  (2,1,3) i

v  (3,4,2) .

Solució:
(12,11,13)  a(2,1,3)  b(3,4,2)  (2a,1a,3a)  (3b,4b,2b) 
12  2a  3b

(12,11,13)  (2a  3b,1a  4b,3a  2b)   11  a  4b
13  3a  2b

Arribem a un sistema d’equacions sobredeterminat (3 equacions i 2 incògnites),
que hem de (intentar) resoldre:
12  2a  3b

 11  a  4b  a  4b  11  12  2(4b  11)  3b 
13  3a  2b

 12  5b  22  b  2, a  3
Comprovem que la solució a  3, b  2 satisfà les tres equacions, per tant el
sistema és compatible determinat i la resposta és:
(12,11,13)  3(2,1,3)  2(3,4,2)
Exercici resolt.
Intenta escriure el vector ( 3 ,  4 , 2 ) com a combinació lineal dels vectors
 
u  (3,4,1) i v  (2,5,1) .

Solució:
(3,4, 2)  a(3,4,1)  b(2,5,1) 
(3,4,2)  (3a,4a, a)  (2b,5b,1b) 
(3,4,2)  (3a  2b,4a  5b, a  b) 
3  3a  2b

 4  4a  5b
2  a  b

Arribem a un sistema d’equacions sobredeterminat (3 equacions i 2 incògnites),


que hem de (intentar) resoldre:
3  3a  2b

 4  4a  5b  a  2  b  3  3(2  b)  2b 
2  a  b

 3  6  b  b  3, a  1

Però la solució b  3, a  1 no satisfà la segona equació:


 4(1)  5(3)  4  15  19  4

Per tant el sistema és incompatible, és a dir, no es pot resoldre, i per tant el



vector ( 3 ,  4 , 2 ) no es pot escriure com a combinació lineal de u  (3,4,1) i

v  (2,5,1) . El problema no té solució.

Combinacions lineals de tres


 vectors. L’espai vectorial.   
Direm que el vector t és combinació lineal dels vectors u , v i w quan
existeixen nombres reals a , b i c tals que
   
t  a u  b v  cw
  
El conjunt de totes les combinacions lineals dels vectors u , v i w formen tot
l’espai de vectors:
     
u , v , w   au  bv  c w , a, b, c  IR   IR3

  
Però només quan el seu determinant és diferent de zero: det u , v , w  0
  
i en aquest cas direm que els vectors u , v i w són linealment independents, o
que formen una base de IR3 .
Observació. L’espai IR3 té tres dimensions.
Aquest procés no pot créixer més, s’acaba aquí: És impossible trobar quatre
vectors linealment independents en IR3. Per això diem que l’espai IR3 té tres
dimensions.
Anomenem base de IR3 a qualsevol conjunt de tres vectors linealment
   
independents, i donada una base u , v , w , qualsevol vector t de IR3 sempre es
pot escriure com a combinació lineal d’aquests tres vectors, i a més a més de
forma única:
   
t  a u  bv  c w

Exercici resolt.
  
Donats els vectors u  (2,3,1) , v  (4,1,3) i w  (1,3,2) , demostra que

formen una base de IR3 i escriu el vector t  (1,10 , 5 ) com a combinació lineal
de tots tres.

Solució:
Formen una base perquè el seu determinant no és zero:
2 4 1
 3  1 3  22  0
1 3 2

Volem trobar els nombres a, b i c de forma que


(1,10 , 5 )  a(2,3,1)  b(4,1,3)  c(1,3,2)

És a dir:
( 1,10 , 5 )  (2a,3a, a)  (4b,b,3b)  (c,3c,2c) 
( 1,10 , 5 )  (2a  4b  c,3a  b  3c, a  3b  2c) 
1  2a  4b  c

10  3a  b  3c
5  a  3b  2c

Un sistema lineal 3x3 que sabem que és SCD perquè el seu determinant és
diferent de zero. El resolem per Gauss:
1  2a  4b  c  2 4 1 1   1 3 2 5
    
10  3a  b  3c    3  1 3 10     3  1 3 10  
5  a  3b  2c  1 3 2 5   2 4  1 1 
  
1 3 2 5  1 3 2 5  c  1
    
0 8 9 25    0 8 9 25   b  2
 0  2  5  9  0 0  11  11 a  3
    

Per tant (1,10 , 5 )  (3)(2,3,1)  2(4,1,3)  1(1,3,2)


Objecte geomètric que es
Nombre de vectors linealment
Condició genera
independents

1 vector 
v 0

Recta vectorial

 
2 vectors u kv

Pla vectorial

3 vectors    Tot l’espai de vectors


det(u , v , w)  0
IR3

2.1.3
   
Donat u  (5,1,2) , determina 2u ,  3u i 0.5u

2.1.4
   1
Donat u  (2,7,5) , determina 4u ,  u i u
3

2.1.5
Comprova si els vectors (-2,1,0), (-1,3,1) i (3,3,0) són linealment independents.

2.1.6
Determina k per a que els vectors (-3,k,2), (4,1,0) i (-1,2,-1) siguin coplanaris.

2.1.7
 
Escriviu el vector v  (1,2,4) com a combinació lineal dels vectors a  (1,0,1) ,
 
b  (1,1,0) i c  (0,1,1) .
2.1.8
Sigui V= {(–1, 1, 1), (–2, –1, 0), (1, 2,a)} un conjunt de vectors de IR3.
a) Trobeu el valor o els valors de a perquè V sigui linealment dependent.

b) Quan a= 4, expresseu el vector v  (3, 9, 14) com a combinació lineal dels
vectors de V.
Solució PAU CAT TEC SET 2013 1.1

2.1.9
  
Siguin u1  (-1, 3, 2) , u 2  (2,-1, 4) i u 3  (a  1, a - 1, 4a  2) tres vectors de
l’espai vectorial IR3.

a) Trobeu el valor del paràmetre a per al qual el vector u 3 és combinació lineal
 
dels vectors u1 i u 2
  
b) Comproveu que per a a= 0 el conjunt  u1 , u2 , u3 és linealment independent.

Solució PAU CAT TEC JUNY 2010 4.6

2.1.10
Considereu els vectors de IR3:
  
v1  (-1, 3, 4) , v2  (2, - 1, - 3) i v3  (1, 2k  1, k  3) .
a) Trobeu l’únic valor de k per al qual aquests vectors no són una base de IR3.
b) Per a un valor de k diferent del que heu trobat en l’apartat a), quins són els
      
components del vector w v 1 v 2 v 3 en la base v1 , v 2 , v 3  ?

Solució PAU CAT TEC SET 2005 3.2

2.1.11
 
Donats els vectors u  (1, 2) i v  (-3,1) :
 
a) Comproveu que u i v formen una base de l’espai vectorial dels vectors del
pla;
 
b) Trobeu els components del vector = (–1, 5) en la base  u , v 

Solució PAU CAT TEC SET 2004 5.4

2.1.12
  
Donats els vectors u  (1,-1, 4) , v  (2, 1, 3) i w  (1, 0, 0) ,
a) Determineu si són vectors linealment dependents o independents.
b) Calculeu la relació que hi ha d'haver entre els valors de a i b per tal que el
 
vector (a, 1, b) sigui combinació lineal de u i v .

Solució PAU CAT TEC SET 2000 2.3

2.1.13
Calculeu el valor de k per tal que els vectors ( 1 , 2 , -1, 1 ) , ( 1 , -1 , 1 , 1 ) i
( 2 , 5k , -3k, 2 ) siguin linealment dependents.

PAU COU
2.1.14
  
Donats els vectors u  (a,1  a,2a) , v  (a,1, a) i w  (1, a,1) , es demana:
  
a) Determineu els valors de a per als què els vectors u , v , w siguin linealment
dependents.
   
b) Estudieu si el vector c  (3,3,0) depèn linealment dels vectors u , v , w per al
cas a  2 . En cas afirmatiu, determineu la combinació lineal associada.

PAU MADRID TEC 2000 Opció A

Definició general de dependència i independència lineal.


 
Direm que un conjunt de vectors v1 ,, vn  són linealment dependents quan
podem trobar un conjunt d’escalars 1 ,  , n , no tots nuls, tal que

  
1v1    nvn  0

Equivalentment, direm que són linealment dependents quan algun d’aquests


vectors es pot escriure com a combinació lineal dels altres. En cas contrari direm
que són linealment independents.

Exemple resolt.
  
Demostrar que els vectors u  (1,1,0) , v  (1,3,2) , w  (4,9,5) són linealment
dependents.

Aplicant el determinant:
1 1 0
1 3 2  1  3  5  1  2  4  0  1  9  4  3  0  9  2  1  5  1  1  15  8  18  5  0
4 9 5
Per tant són tres vectors linealment depenents.

Aplicant la definició anterior:


Volem trobar tres escalars a, b, c de forma que
   
au  bv  cw  0

És a dir:
(0,0,0)  a(1,1,0)  b(1,3,2)  c(4,9,5)  (a, a,0)  (b,3b,2b)  (4c,9c,5c) 
0  a  b  4c

 (a  b  4c, a  3b  9c,2b  5c)  0  a  3b  9c
0  2b  5c

Resolem aquest sistema i veiem que és un sistema compatible indeterminat:


b  5a / 3

c  2a / 3
a lliure

Només cal prendre, per exemple: a  3  b  5, c  2 .

Comprovació:
3(1,1,0)  5(1,3,2)  2(4,9,5)  (3,3,0)  (5,15,10)  (8,18,10) 
(3  5  8,3  15  18,0  10  10)  (0,0,0)

Xavi Mates: Base d'un espai vectorial


https://youtu.be/D5Rq5AMt27k
2.2 Rectes. Equacions de la recta.

Equacions de la recta.
Una recta r en l’espai queda determinada per un punt P  ( p1 , p2 , p3 ) (“punt

base”) i un vector v  (v1 , v2 , v3 ) (“vector director”)


Conjunt de punts: r : Q  P  k v , k  IR 

Equació vectorial: ( x, y, z )  ( p1 , p2 , p3 )  k (v1 , v2 , v3 )

 x  p1  k v1

Equació paramètrica:  y  p 2  k v2
z  p  k v
 3 3

x  p1 y  p2 z  p3
Equació contínua:  
v1 v2 v3

a1 x b1 y c1 z d 1


Equació general o cartesiana: 
a 2 x b 2 y c 2 z d 2

on (a1 , b1 , c1 ) i (a2 , b2 , c2 ) no són proporcionals.

Més endavant veurem que l’equació cartesiana d’una recta és,


en realitat, la interpretació de la recta com a intersecció de
dos plans.

De l'equació contínua podem agafar un vector director només mirant els


denomiandors, però no oblidis que les lletres han d'anar amb signe positiu!
x  3 5  y z 1 
   v  (2,3,4)
2 3 4
x  3 5  y z 1 x  3 y  5 z 1 
      v  (2,3,4)
2 3 4 2 3 4
Recta que passa per dos punts.
Donats dos punts P i Q, existeix una única recta que passa per tots dos.

Agafem com a punt base qualsevol dels dos i com a vector director el vector que

determinen: v  PQ .

2.2.1
Determina l’equació contínua de la recta que passa per P = ( -11 , -10 , -6 ) i
Q = ( -8 , 17 , -17 )

2.2.2
Determina l’equació contínua de la recta que passa per P =( 17 , 20 , -13 ) i
Q =( -6 , -1 , -2 )

2.2.3
Considereu els punts de l’espai P  (1, a  1,3) , Q  (0, a  2,1  a) i
R  (2,1,6  6a)
a) Trobeu el valor de a per al qual els tres punts estan alineats.
b) Quan els tres punts estan alineats, quina és l’equació de la recta que els conté?

Solució PAU CAT TEC JUNY 2009 3.6

2.2.6
Trobeu l’equació de la recta continguda en el pla  : x  2y  6z - 2  0 , que talla
els eixos OY i OZ.
Solució PAU CAT TEC JUNY 2007 1.4

2.2.7
Considereu els punts de l’espai P=(–1, a–1, 3), Q=(0, a–2, 1–a) i
R=(2, –1, 6 – 6a).
a) Trobeu el valor de a per al qual els tres punts estan alineats.
b) Quan els tres punts estan alineats, quina és l’equació de la recta que els conté?

Solució PAU CAT TEC JUNY 2007 1.2

2.2.8
Considereu els punts de l’espai A=(1, 1, 2), B=(0, 1, 1) i C=(k, 1, 5).
a) Trobeu l’equació de la recta que passa per A i B.
b) Per a quins valors de k els punts A, B i C formen un triangle?
Solució PAU CAT TEC JUNY 2004 4.1
De l’equació general a l’equació paramètrica.

Exemple resolt.
Determina les equacions paramètriques associades a la recta
2x - y - 5  0
r:
x  z - 2  0

Solució:
Primer mètode. Resolem el sistema compatible indeterminat:

 2  1 0 5  F 2 F  F  2  1 0 5 
2x - y - 5  0   2 2 1 
r:   1 0 1 2    0 1 2  1
x  z - 2  0 0 0 0 0 0 0 0 0 
   

Mètode de la z lliure:
y  2 z  1  y  2 z  1
2 x  y  5  2 x  (2 z  1)  5  2 x  2 z  1  5  2 x  2 z  5  1
 2x  2z  4  x  z  2  x  z  2

x  z  2 x  z  2 x  2  z x  2  
   
 y  2 z  1   y  2 z  1   y  1  2 z   y  1  2
 z lliure z  z z  0  z z  0  
   

I ja tenim la forma paramètrica.



Observem que un punt base és P  (2,1,0) i un vector director és v  (1, 2 ,1) .

Segon mètode. Producte vectorial + Tempteig:


Podem determinar un vector director mitjançant el producte vectorial:
i j k
a1 x b1 y c1 z d 1 
  v  a1 , b1 , c1   a2 , b2 , c2   a1 b1 c1
a 2 x b 2 y c 2 z d 2 a2 b2 c2
En el nostre cas:
i j k
2x - y - 5  0   1 0 2 0 2 1 
r:  v  2  1 0   , ,    1,2,1
x  z - 2  0  0 1 1 1 1 0 
1 0 1

Observem que el vector director obtingut és equivalent al vector director


obtingut amb el primer mètode:  1,2,1  1,2,1
I podem determinar un punt base per tempteig. En el nostre cas concret,
imposant z  0 , tenim
z  0  x  0 - 2  0  x  2  2  2 - y - 5  0  4 - 5  y  -1  y

i arribem al punt base P  (2,-1,0), que coincideix amb el punt trobat amb el
primer mètode.
2.3 Punt d’intersecció entre dues rectes.
La millor forma de determinar el (possible) punt d’intersecció entre dues rectes és
passar-les a forma paramètrica i resoldre el sistema de dues equacions i dues incògnites
que obtenim a l’igualar les tres coordenades.

Exemple resolt.
Determina el punt d’intersecció entre les rectes r : x  1  y  1  z i
 x  2 y  1
s:
y  z 1

En primer lloc passem les dues rectes a forma paramètrica:


 x  1  1
x  1 y  0 z  1  P  (1,0,1) 
r : x 1  y  1 z        y  0  1
1 1 1 v  (1,1,1)  z  1  1

 x  2 y  1
s:  y  1  z  x  1  2 y  1  2(1  z )  1  2 z 
y  z  1
x  1  2z  x  1  2
 
 y  1 z  y  1 
z  z z  
 

Un cop hem passat les dues equacions a forma paramètrica, igualem les tres
coordenades:
1  1  1  2

0  1  1  
1  1  

És un sistema sobredimensionat: Tres equacions i dues incògnites. Serà


compatible determinat si i només si les dues rectes es tallen en un punt.

Resolem el sistema de les dues primeres equacions:


1  1  1  2
  1  1    1  2  2    1  2
0  1  1      1  
 2  1    2  1      1  (1)  2

Comprovem que la solució obtinguda satisfà la tercera equació: 1  1  2  1

Així doncs, el sistema és compatible determinat amb solució   2,   1 .


El punt de tall el trobem substituint un d’aquests dos valors a les equacions
paramètriques corresponents:
 x  1  1  1  1  2  3

 y  0  1  0  1  2  2  P  ( 3 , 2 ,  1)
 z  1  1  1  1  2  1

2.3.1
Determina el punt P d’intersecció entre les rectes r i s.

y 1 6  z x 1  4  y
r : x  12   s:   z2
3 4 4 3

2.3.2
Determina el punt P d’intersecció entre les rectes r i s.

x6 z 3 y  10
r:  3  y  s: x 3  z  10
3 2 5
2.4 Plans. Equacions del pla.

Equacions del pla.


Un pla  en l’espai queda determinat per un punt P (“punt base”) i dos vectors
 
directors  v , w linealment independents (“vectors directors”).

 
Conjunt de punts:  :  Q  P  a v  bw | a, b  IR 

Equació vectorial: ( x, y, z )  ( p1 , p2 , p3 )  a(v1 , v2 , v3 )  b( w1 , w2 , w3 )

 x  p1  a v1  b w1

Equació paramètrica:  y  p2  a v2  b w2
z  p  a v  b w
 3 3 3

Equació cartesiana: Ax  By  Cz  D

El vector n  ( A, B, C ) s’anomena vector normal
del pla i té unes propietats geomètriques molt
interessants. (Vegeu Apartat 2.3.7)

Primer mètode per a obtenir l’equació cartesiana d’un pla.


 
Un punt Q  ( x, y, z) pertany al pla si i només si PQ , v i w son linealment
dependents

i per tant, el determinant


d’aquests tres vectors ha de ser
zero:

x  p1 y  p2 z  p3
v1 v2 v3  0
w1 w2 w3
Pla que passa per tres punts.
Donats tres punts no alineats de l’espai
A  (a1 , a2 , a3 ), B  (b1 , b2 , b3 ), C  (c1 , c2 , c3 )

Existeix un únic pla que passa per aquests tres punts.

 
Agafem com a punt base el punt A, com a vectors directors v  AB , w  AC , i
apliquem 2.3.2.

Exercici resolt.
Determina el pla que passa pels següents punts:

A  (3,4,3) , B  (0,1,4) , C  (1,5,1)

Solució:
x  (3) y  (4) z 3 x3 y4 z 3 x 3 y  4 z 3
0  0  (3) 1  (4) 4  3  0  3 1 4 43  3 5 1 =
1  (3)  5  (4)  1  3 1  3  5  4  1  3 4 1 4
 ( x  3)5(4)  ( y  4)  1  4  ( z  3)3(1)  4  5( z  3)  (1)1( x  3)  (4)3( y  4) 
 20( x  3)  4( y  4)  3( z  3)  20( z  3)  ( x  3)  12( y  4)
 20 x  60  4 y  16  3z  9  20 z  60  x  3  12 y  48
 19 x  16 y  23 z  76

Per tant, l’equació és  19 x  16 y  23z  76  0  19 x  23z  76  16 y

Efectivament, aquesta equació es satisfà amb els punts A, B i C de l’enunciat:


19(3)  23(3)  76  16(4)
19(0)  23(4)  76  16(1)
19(1)  23(1)  76  16(5)

Xavi Mates : Equacions del pla a partir de 3 punts


https://youtu.be/Aahxyb7BxN8

Montero Espinosa: Ecuacion del plano que pasa por tres puntos
https://youtu.be/aivmNQtsNAM
()
2.4.1
Determina l’equació general del pla que passa pels següents tres punts:
a) P=(1, -2, 0), Q=(3, 1, 4), R=(0, -1, 2)
b) P=(1, 1, 1), Q=(1, 2, 0), R=(-1, 2, 1)
c) P=(2, 1, -1), Q=(0, -2, 0), R=(1, -1, 2)
d) P=(3, 2, 0), Q=(1, 3, -1), R=(0, -2, 3)

2.4.2
Determineu raonadament l’equació del pla que passa pels punts A  (0,0,0) ,
B  (6,3,0) i C  (3,0,1)

2.4.3
Determina el pla que passa pel punt P  (2,1,7) i té com a vectors directors
 
v  (2,1,0) i w  (1,0,2) .

2.4.4
Determina el pla que passa pel punt P  (5,0,3) i té com a vectors directors
 
v  (1, 2, 4) i w  (3, 1, 0) .

2.4.5
Donats els punts P=(1, 0, 0), Q=(0, 2, 0), R=(0, 0, 3) i S=(1, 2, 3),
a) Calculeu l’equació cartesiana (és a dir, de la forma Ax+By+Cz+D=0) del pla
que conté els punts P, Q i R.
b) Comproveu si els quatre punts són coplanaris (és a dir, si els quatre estan
continguts en un mateix pla).
Solució PAU CAT TEC JUNY 2012 3.6

2.4.6
Considereu els punts de l’espai A=(0, 0, 1), B=(1, 1, 2) i C=(0, –1, –1).
a) Trobeu l’equació del pla ABC.
b) Si D és el punt de coordenades (k, 0, 0), quant ha de valer k per tal que els
quatre punts A, B, C i D siguin coplanaris?
Solució PAU CAT TEC JUNY 2004 1.2

2.4.7
Un quadrat de l'espai té tres dels seus vèrtexs consecutius situats en els punts de
coordenades enteres P = (3, –2, 4), Q = (a, –1, a + 1) i R = (2, –3, 0).
a) Tenint en compte que els vectors QP i QR han de ser perpendiculars,
calculeu el valor del nombre enter a.
b) Calculeu l'equació del pla que conté aquest quadrat.
c) Calculeu el quart vèrtex d'aquest quadrat.
d) Calculeu l'àrea d'aquest quadrat.
Solució PAU CAT TEC JUNY 2000 3.6
Pla que conté una recta i passa per un punt.

Donada una recta r : P  v i un punt Q r , existeix un únic pla que conté la
recta r i passa per Q.

Agafem com a punt base el punt Q i com a vectors directors el vector director de
 
la recta v i el vector w  PQ .

2.4.8
y
Determineu l'equació del pla que conté a la recta x 1   z  1 i passa per
2
l'origen de coordenades.
PAU CAT TEC JUNY 2003 5.2

2.4.9
x 3 z 1
Considereu la recta r de l'espai d'equacions y
2 2
Trobeu l'equació cartesiana del pla que conté r i que passa pel punt P = (1, 1, 1)
(equació cartesiana vol dir la de la forma ax+by+cz=d).

PAU CAT TEC SET 1999 2.2


Pla que conté dues rectes paral·leles.
 
Donades dues rectes paral·leles r : P  v i s : Q  w , existeix un únic pla que
conté a totes dues.


Agafem com a punt base el punt P i com a vectors directors el vector director v

d’una recta r i el vector w  PQ .

2.4.10
x  5 y 1 z  2 2 x  y  2 z  5  0
Donades les rectes r1: :   i r2: : 
3 2 4 2 x  y  z  11  0
a) Comproveu que són paral·leles.
b) Trobeu l’equació general (és a dir, de la forma Ax+By+Cz+D= 0) del pla que
les conté.
Solució PAU CAT TEC JUNY 2010 1.4

2.4.11
x 1 x  2z  5
Considereu les rectes r :  y  z2 i s:
2  x  2 y  11
Comproveu que aquestes dues rectes són paral·leles i calculeu l'equació del pla
que les conté.
PAU CAT TEC JUNY 1999 6.4
Pla que conté dues rectes que es tallen.
 
Donades dues rectes r : P  v i s : Q  w que es tallen, existeix un únic pla
que conté a totes dues.

Agafem com a punt base qualsevol punt d’una de les rectes i com a vectors
directors els vectors directors de les rectes.

Important: El punt base del pla no necessàriament ha de ser el punt de tall de


les rectes. No l’has de calcular, una feina menys!.

Exercici resolt.
x  y  2  0 x  2
Les rectes r :  i s: són secants. Determina el pla que
 z  1 y  z  5  0
les conté.

i j k

Solució: Un vector director de la recta r és v  1  1 0  (1,1,0)  (1,1,0)
0 0 1
i j k

Un vector director de s és w  2 0 0  (0,2,2)  ( 0 , 1 , 1 )
0 1 1
Clarament són vectors no proporcionals (1,1,0)  k (0,1,1) , per tant són rectes no
paral·leles. Com a punt base del pla podem agafar qualsevol punt de les rectes,
per exemple, prenent la recta r, i fent x  0 obtenim el punt
0  y  2  0  y  2  P  (0,2,1)
Per tant, el pla que volem té com a punt base P  (0,2,1) i vectors directors
 
v  (1,1,0) i w  ( 0 , 1 , 1 )
x0 1 0
0  y  2 1 1  x  z  y  3  x  y  z  3
z  (1) 0 1

2.4.12
Siguin r i s dues rectes d’equacions
y2 za
r : ( x, y, z)  (4,3,4)  t (2,1,1) , s : x 1  
1 3
a) Trobeu el valor del paràmetre a perquè aquestes rectes es tallin.
b) En el cas en què es tallen, trobeu l’equació general (és a dir, de la forma
Ax+By+Cz+D=0) del pla que les conté.

Solució PAU CAT TEC JUNY 2010 5.5


Vector normal del pla.
De l’equació cartesiana Ax  By  Cz  D definim el vector normal al pla com

n  ( A, B, C )

Aquest vector és molt important geomètricament perquè és perpendicular al pla.

Segon mètode per a obtenir l’equació cartesiana d’un pla.

Obtenim un vector normal del pla amb el producte vectorial dels vectors
directors:
  
n  vw

El valor D s’obté substituint el punt base P a l’equació.


Exemple resolt.
Determineu l’equació del pla que passa pel punt (-1, 2 , 3) i és paral·lel als
 
vectors v   1, 2 , 3  i w   1, 3 , 5  .

Solució: Calculem un vector normal al pla amb el producte vectorial:

i j k
    2 3 1 3 1 2 
n  v  w  1 2 3   , ,    1,  2 ,1 

1 3 5 
3 5 1 5 1 3 

Per tant, el nostre pla té per equació 1x  2 y  1z  D

I la D la trobem avaluant al punt donat:


1(1)  2  2  1 3  D  2  D

Per tant, el pla que busquem té per equació x  2 y  z  2


2.5 Punt d’intersecció entre recta i pla.
En l’apartat 6.1 veurem les diferents posicions relatives entre una recta i un pla. Aquí,
de moment, suposarem que la recta i el pla es tallen en un punt i estudiarem com
determinar aquest punt de tall.

Exercici resolt.
Determina el punt de tall entre el pla  : x  y  2 z  5 i la recta
x  y  z  0
r:
2 x  y  z  10

Solució:
x  y  2z  5

Un mètode és resoldre el sistema 3x3  x  y  z  0 per Gauss.
2 x  y  z  10

Si no volem fer-lo, un mètode alternatiu és passar la recta a forma paramètrica:
i j k
Vector director de la recta: v  1 1 1  (2,1,3)
2 1 1
y  z  0
Punt base de la recta (fent x  0 )   y  5, z  5  P  (0,5,5)
 y  z  10
 x  2

Les equacions paramètriques de la recta r són, doncs:  y  5  
 z  5  3

Substituïm el punt genèric de la recta a l’equació del pla, obtenint així una
equació de primer grau en  :

 : x  y  2 z  5  2  (5   )  2(5  3 )  5    2

x  2  2

I per tant el punt de tall és  y  5  2  (4,3,1)
z  5  3  2

La Prof Lina M3: Intersección de una recta y un plano en R3


https://youtu.be/tR-EI8TBJjk

Ronny Online: Intersección entre una recta y un plano en el espacio #1


https://youtu.be/UnG5mQUQmsI
2.5.1
Un avió es desplaça des d’un punt A = (0, 3, 1) cap a una plataforma plana
d’equació π: x – 2y + z = 1 seguint una recta r paraŀlela al vector v = (1, –1, 0).
a) Calculeu les coordenades del punt de contacte B de l’avió amb el pla i la
distància recorreguda.
b) Calculeu l’equació general del pla perpendicular a la plataforma i que conté la
recta r seguida per l’avió des del punt A.
PAU CAT TEC SET 2020 #2

2.5.2
Determina el punt d’intersecció entre el pla  : 2 x  3 y  z  0 i la recta
 x  1  2t

r :  y  4  3t
 z  t

2.5.3
x 1 y  1 z  2
Donats el pla π: 3x –2y +5z = 6 i la recta r :   , busqueu el
2 1 3
punt de tall, si existeix.

Solució PAU CAT TEC JUNY 2008 5.4

2.5.4
Donades les rectes
x  2 y 1 z x 1 y  7 z  5
r:   i s:  
1 2 1 1 2 3

i el punt P= (1, 1, –1), volem trobar l’equació de la recta que passa per P i que
talla r i s. Per aconseguir-ho:
a) Trobeu l’equació general o cartesiana (és a dir, l’equació de la forma
Ax+By+Cz+D=0) del pla π que conté la recta r i el punt P.
b) Trobeu el punt M calculant el punt d’intersecció del pla π amb la recta s.
c) Trobeu l’equació de la recta que passa pels punts P i M.
d) Comproveu que la recta trobada en l’apartat anterior és la que busquem.
PAU CAT TEC JUNY 2008 2.6 (Problema)

2.5.5
Una recta r és paral·lela a la recta s : x  1  y  1  z  1 , talla en un punt A la
x 1 y x  2 y 1 z
recta t :   z  1 , i en un punt B la recta l :   .
3 2 2 2 3

a) Trobeu l’equació del pla determinat per les rectes r i t.


b) Trobeu el punt B calculant el punt d’intersecció del pla anterior amb la recta l.
c) Trobeu l’equació de la recta r.
d) Trobeu el punt A.
Solució CAT TEC SET 2007 3.5
2.5.6
Considerem el cub de vèrtexs A, B, C, D, E, F, G, H que té l'aresta de longitud 4
dm.

a) Determineu l'equació del pla inclinat EHBC si prenem com a origen de


coordenades el vèrtex D i com a eixos de coordenades DA, DC i DH en aquest
ordre, tenint en compte que el sentit positiu de cada un d'ells és el que sortint de
l'origen D va cap a A, C i H, respectivament.
b) Calculeu les equacions de les diagonals CE i AG i utilitzeu-les per calcular les
coordenades del seu punt d'intersecció.

Solució PAU CAT TEC SET 2002 1.6 (Problema)


2.6 Recta intersecció de dos plans.
Dos plans no paral·lels es tallen en una recta.

Les equacions cartesianes dels


dos plans donen directament
l’equació cartesiana de la recta,
ja que l’equació cartesiana d’una
recta no és res més que la
interpretació de la recta com a
intersecció de dos plans.

Propietat important.
El vector director de la recta intersecció és perpendicular comú als dos vectors
normals dels plans, per tant:

Per determinar el vector director


de la recta intersecció podem
calcular el producte vectorial
dels dos vectors normals:

i j k
  
v  n  m  n1 n2 n3
m1 m2 m3

Unicoos: Vector director y punto de una recta dada como intersección de dos planos
https://youtu.be/vGgCMAKAnLo
2.7 Rectes coplanàries.
Pensem en carreteres. Si ens mantenim “a peu de terra”, només tenim dues
opcions: O fem carreteres paral·leles, o hem de fer encreuaments. Dues rectes
diferents en el pla, o bé són paral·leles o bé es tallen en un punt.

Però ara estem en l’espai, hem passat de dos dimensions a tres, i per tant tenim
una dimensió més per on ens hi podem escapar. Ara podem fer rectes que es
creuen, és a dir, que no estan al mateix pla:

Direm que dues rectes són coplanàries quan estiguin contingudes en un mateix
pla.
Direm que dues rectes es creuen en cas contrari, és a dir, quan no siguin
coplanàries.
Dues rectes paral·leles sempre seran coplanàries.

Aquí, l’expressió “creuar-se” no té el sentit habitual de “creuar-se amb una


persona pel carrer” que trobem al diccionari:

encreuar
- intr. pron. [LC] Dues vies, trobar-se en un punt del seu recorregut i travessar l’una l’altra. Els dos
camins s’encreuen a can Gomis.
- intr. pron. [LC] Un automòbil, un tren, un missatger, etc., trobar-se en un punt determinat del
recorregut amb un altre que va en sentit contrari. El Talgo de París i l’exprés de Barcelona
s’encreuaran a l’estació de Girona. Encreuar-se dos correus.
Com determinar si dues rectes són coplanàries.

Donada una recta r amb punt base P i vector director v , i una recta s amb punt

base Q i vector director w , les dues rectes seran coplanàries si i només si
 
PQ, v , w són vectors coplanaris, és a dir, el seu producte mixt (és a dir, el
determinant) és zero:

 
r i s coplanàries  det( PQ, v , w)  0

2.7.1
Determina si les rectes r i s són o no coplanàries:
x 1
r:  y  z 1 s : x   y  z
2

2.7.2
Determina si les rectes r i s són o no coplanàries:
x  y  z  3  0 x 1 y 1 z 1
r: s:  
2 x  y  z  2  0 2 1 3

2.7.3
x  1  
y 1 z 1 
Donades les rectes r : x  2   i s : y  2  
k 2  z  2

Trobeu k per a què r i s siguin coplanàries.

2.7.4
Quina condició han de complir p i q per tal que les dues rectes r1 i r2 de l’espai
estiguin contingudes en un pla ?

x  2 y  p x  z  1
r1 :  r2 : 
y  z  3  y  2z  q
PAU COU
2.7.5
Considereu les dues rectes de l’espai donades per les equacions següents:

x  y  2  x  y  3
 
3x  y  z  4 x  2 y  z  a

Determineu el paràmetre a per tal que siguin coplanàries.


PAU COU

2.7.6
Considereu les rectes r1 i r2 de l’espai donades per les equacions següents:

y 1 x 1 y  1 z
r1 : x  z, r2 :  
2 a b 2

a) Calculeu a i b per tal que r1 i r2 siguin paral·leles.


b) Calculeu la relació que hi ha d’haver entre a i b per tal que r1 i r2 pertanyin al
mateix pla.
PAU COU
2.8 Resolució de problemes mètrics amb rectes parametritzades.
Molts dels problemes de determinació de punts sobre una recta es resolen
seguint el següent model: Prenem un punt genèric de la recta (és a dir, fem
servir la seva equació paramètrica), imposem sobre aquest punt la condició
indicada a l’enunciat, i simplifiquem fins a obtenir una equació sobre el
paràmetre de la recta. Substituint les solucions obtingudes a l’equació
paramètrica obtenim els punts desitjats. Els problemes resolts següents són
exemples concrets que mostren com aplicar aquesta tècnica.

Determinació dels punts d’una recta a una distància fixa d’un punt.

Exercici resolt.
Determineu els punts de la recta
x  2 y 1
  3  z
3 7

Situats a una distància de 5 unitats del punt P  (3,4,1) .

Solució:
Passem la recta a forma paramètrica:
 x  2  3k
x  2 y 1 x  2 y 1 z  3 
  3  z      y  1  7k
3 7 3 7 1  z  3  k

Un punt qualsevol de la recta serà Q  (2  3k , 1  7k ,  3  k ) . Per tant:


Dist (Q, P)  QP  (3  (2  3k ))2  (4  (1  7k ))2  (1  (3  k ))2  *

(3  (2  3k ))2  (3  2  3k ) 2  (1  3k ) 2  9k 2  6k  1
(4  (1  7k ))2  (4  1  7k ) 2  (5  7k ) 2  49k 2  70k  25
(1  (3  k ))2  (1  3  k ) 2  (4  k ) 2  k 2  8k  16

*  59k 2  72k  42  5  59k 2  72k  42  52  59k 2  72k  42  25  0


 59k 2  72k  17  0
 72  72 2  4  59  17  72  1172  72  2 293 k  0.90
k   
2  59 118 118 k  0.32
Per tant els punts són:
 x  2  3(0,90)  0.7

Q1   y  1  7(0,90)  5.3  Q1  (0.7,5.3,2.1)
 z  3  (0,90)  2.1

 x  2  3(0,32)  1.04

Q2   y  1  7(0,32)  1.24  Q2  (1.04,1.24,2.68)
 z  3  (0,32)  2.68

2.8.1
x  4 y 1
Considereu la recta r :   z 1.
2 1

a) Trobeu els dos punts, A i B, de la recta r que estan situats a una distància
d=√6 del punt P=(–1, 1, 2).
b) Trobeu l’àrea del triangle de vèrtexs A, B i P.
Solució PAU CAT TEC SET 2010 2.6

2.8.2
Considereu la recta r d’equació
 x  3  2t

 y  5  2t
z  3  t

i el punt M (2, 3, 7).
a) Trobeu, en funció de t, la distància de M a un punt qualsevol de la recta r.
b) Trobeu les coordenades dels punts A i B de r situats a distància 3 2 del punt
M.
c) El triangle ΔAMB, és rectangle en M?
d) Els punts A i B formen part d’un paral·lelogram de vèrtexs ABCD que té el
centre de simetria en el punt M. Calculeu les coordenades de C i D.

Solució PAU CAT TEC JUNY 2004 4.6 (Problema)

2.8.3
y 5 z 7
Considereu la recta r d'equacions: x  1   . Calculeu els punts
3 4
d'aquesta recta situats a una distància 3 del punt A = (1, 0, 1).
Solució PAU CAT TEC JUNY 2002 3.4

2.8.4
x  2 y  2z  3
Considereu la recta de IR3 d’equacions 
x  y  4z  3
Trobeu tots els punts de la recta donada, tals que la seva distància a l’origen de
coordenades és 14 .
PAU COU
Projecció ortogonal d’un punt en una recta. Distància entre punt i recta.

Volem determinar el punt P de la recta r que és més proper a un punt A donat.


Aquest punt s’anomena projecció ortogonal de A en la recta i verifica una
propietat molt interessant: El punt de la recta amb distància mínima determina
un angle recte:

 
PA  v  0 on v és el vector director de r

Amb aquesta condició podem determinar P, i amb aquest punt, la distància entre
A i P:

dist ( A, r )  dist ( A, P)

Nota: A l’apartat 4.4 resoldrem aquest mateix problema amb tècniques


diferents.

Exercici resolt.
Determineu la distància entre el punt A  (1,2,1) i la recta definida per les
equacions

x  y  2

2 x  y  z  7

Solució:
La recta tindrà com a vector director
i j k
1 0 1 0 1 1
v  1 1 0   , ,   1,1,1  (1,1,1)
 1 1 2 1 2 1 
2 1 1

Com a punt base fem z  0 i per tant obtenim el sistema


x  y  2
  x  5, y  3  Punt base  (5,-3,0)
2 x  y  7

Passem la recta a forma paramètrica:


x  5  

r :  y  3  
z  

Per tant, un punt genèric de la recta serà P  (5   ,3   ,  )
AP  P  A  (5    1,3    2,   1)  (4  ,1  ,   1)

Imposem la condició de perpendicularitat:


AP  v  (4   ,1   ,   1)  (1,1,1)  0  (1)(4   )  1      1  0
 6  3  0    2

La projecció ortogonal de A en la recta és el punt P  (5  2,3  2,2)  (3,1,2)


i la distància del punt A a la recta r serà la distància entre A i P:

dis( A, r )  dist ( A, P)  AP  3  1,1  2,2  1  22  12  12  6

2.8.5
x  y  2 z  1
Determineu el punt de la recta  més proper al punt A  (2,3,1) ,
x  2 y  4 z  1
i la distància entre el punt i la recta.
Punt simètric respecte d’una recta.
Donat un punt A i una recta r, volem determinar el punt A’, simètric de A
respecte de r.

La projecció ortogonal C de A en r és el punt mig del segment AA’, i per tant

A'  A  2 AC

Exercici resolt.
Determineu el punt simètric de A  (3,1,7) respecte de la recta

y  3 z 1
r : x 1  
2 2

Solució:
Passem la recta a equació paramètrica:

Vector director: v  (1,2,2)
Punt base: P  (1,3,1)
 x  1  

r :  y  3  2  C  (1   ,3  2 ,1  2 )  AC  (   2 , 2  2 , 2  6 )
 z  1  2

Imposem la condició de perpendicularitat:


AC  r  AC  v 
(   2 , 2  2 , 2  6 )  (1,2,2)  0 
  2  2( 2  2)  2( 2  6 )  0  9  18  0    2  C  (3,1,5)

Finalment:
A'  A  2 AC  (3,1,7)  2(0,4,4)  (3,3,3)
Determinació del punt d’una recta que equidista respecte de dos punts.

Exercici resolt.
Donats els punts A  (2,1,1) i B  (1,0,1) , i la recta r d’equació
z2
r : x 5  y  ,
2
trobeu el punt C de r que equidista de A i B.
PAU VALÈNCIA TEC JUNY 2008

Solució:

En primer lloc passem la recta a forma paramètrica:


x  5  
z2 Punt base : P  (5,0,-2) 
r : x5 y     y  0  
2 Vector director : v  (1,1,2)  z  2  2

Per tant, un punt genèric de la recta serà P  ( 5   ,  ,  2  2 )

Imposem sobre aquest punt la condició de l’enunciat:


dist ( A, P)  dist ( B, P)  AP  BP

AP  P  A  3   ,   1,3  2 
 AP  3   2    12   3  2 2  62  16  19

BP  P  B  4   ,  ,1  2 
 AP  4   2   2   1  2 2  62  12  17

AP  BP  62  16  19  62  12  17 


62  16  19  62  12  17  16  19  12  17 
1  9 1 
  C   ,  , 1 
2  2 2 

2.8.6
Donats els punts P= (1, 0, –1) i Q= (–1, 2, 3), trobeu un punt R de la recta
x 3 y  4 z 3
r:   que compleixi que el triangle de vèrtexs P, Q i R és
2 3 1
isòsceles, en què PR i QR són els costats iguals del triangle.

Solució PAU CAT TEC JUNY 2013 4.5


Perpendicular comú a dues rectes.
Donades dues rectes r i s, volem determinar els únics punts P de r i Q de s tals
que la recta PQ és perpendicular a r i a s al mateix temps.

Prenem un punt genèric R de la recta r i prenem un punt genèric S de s i


imposem la condició de la doble perpendicularitat:
 
RS  v
 

RS  w

Amb aquestes dues condicions obtenim els punts P i Q. La recta que busquem
serà la recta que passa per aquests dos punts.

Nota: A l’apartat 4.6 resoldrem aquest mateix problema amb tècniques diferents.
Exercici resolt.
Determineu l’equació de la perpendicular comú a les rectes:
x  4 y 1 z  2 y  2 z 8
r:   i s : x 1  
2 1 3 2 2

Solució:
Les rectes venen donades per:
 A  (4,1,2) B  (1,2,8)
r :  i s : 
u  (2,1,3) v  (1,2,2)

Agafem un punt genèric R de la recta r i un punt genèric S de la recta s. Hem de


fer servir la equació paramètrica de cada recta, i hem de fer servir dues lletres
diferents per als paràmetres:
 x  4  2

r :  y  1    R  4  2 , 1   ,  2  3 
 z  2  3

x  1  

s :  y  2  2  S  1   ,  2  2  , 8  2  
 z  8  2

Per tant: RS  (  3  2   ,  3    2 , 10  3  2 )

Ara imposem les condicions de perpendicularitat sobre aquest vector:


 
RS  u  RS  u  0  2(  3  2   )  1(3    2 )  3(10  3  2 )  0
 
RS  v  RS  v  0  1(  3  2   )  2(3    2 )  2(10  3  2 )  0

Simplifiquem el sistema de dues equacions amb dues incògnites anterior:


 6  4  2  3    2  30  9  6  0  27  14  10  0 14  10  27
  
 3  2    6  2  4  20  6  4  0 23  10  9  0  10  9  23 

1
Que té com a solució:     2
2
 1 1
Substituïm per obtenir els punts P i Q: P   5 , ,  Q   1 , 2 , 4
 2 2 

Un vector director de la recta perpendicular serà:


 3 9
PQ    6 , ,    12 , 3 , 9   4 , 1 , 3
 2 2
x 1 y  2 z  4
i per tant la seva equació contínua serà: t :  
4 1 3
Exercici resolt.
Determineu la perpendicular comú a les rectes

x  1 x  
 
r : y  1 i s :  y   1
z    2  z  1
 

PAU ANDALUCÍA JUNY 2009 4A

Solució:
Un punt genèric de r és R  (1,1,   2) i un punt genèric de s és
S  ( ,   1,1) .

Per tant :
RS  (  1,   2,  1)

Imposem les condicions de perpendicularitat :


RS  u  0    1  0

RS  v  0    1    2  0

Resolem el sistema que hem obtingut:


3
 1 ,  
2

Per tant la recta demanada serà la recta que passa pels punts P  (1,1,1) i
3 1 
Q   , , 1  .
2 2 
 1 1 
El seu vector director serà PQ   , , 0   (1,1,0)
2 2 

I per tant tindrà equació:


x 1 y 1
 , z  1
1 1
Determinació dels punts d’una recta equidistants a dos plans.

2.8.7
Sigui r la recta que passa pels punts A = (0, 1, 1) i B = (1, 1, –1).
a) Trobeu l’equació paramètrica de la recta r.
b) Calculeu tots els punts de la recta r que estan a la mateixa distància dels plans
π1: x + y = –2 i π2: x – z = 1.

Nota: Podeu calcular la distància d’un punt de coordenades (x0, y0, z0) al pla
Ax0  Ay0  Cz 0  D
d’equació Ax + By + Cz + D = 0 amb l’expressió
A2  B 2  C 2

Solució PAU CAT TEC JUNY 2018 1.2

2.8.8
Considereu a IR3 la recta que té per equació r: (x, y, z) = ( –4+2λ , –2 , 1–λ ) i els
plans π1 i π2 d’equacions π1: x + 2y + 2z = –1 i π2: x – 2y + 2z = –3,
respectivament.
a) Determineu la posició relativa de π1 i π2.
b) Comproveu que tots els punts de la recta r estan situats a la mateixa distància
dels plans π1 i π2.

Nota: Podeu calcular la distància d’un punt de coordenades (x0, y0, z0) al pla
d’equació
Ax0  By0  Cz 0  D
Ax + By + Cz + D = 0 amb l’expressió
A2  B 2  C 2
Solució PAU CAT TEC JUNY 2015 2.4

2.8.9
A l’espai tridimensional considereu la recta r: (x, y, z) = (3+2α , –α , 3–α ) i els
plans
π1: x + y + z = –1 i π2: (x, y, z) = (2 + λ, 1 – λ + μ, μ).

a) Calculeu l’equació cartesiana (és a dir, que té la forma Ax + By + Cz = D) del


pla π2.
b) Trobeu els dos punts de la recta r que equidisten dels plans π1 i π2.

Nota: Podeu calcular la distància d’un punt de coordenades (x0, y0, z0) al pla
Ax0  By0  Cz 0  D
d’equació Ax + By + Cz + D = 0 amb l’expressió
A2  B 2  C 2

Solució PAU CAT TEC SET 2015 5.2


2.8.10
x2 z 1
Siguin r i s les rectes de IR3 d’equacions r : y i
3 4
s : ( x, y, z)  (1  2 ,3   ,4  3 ) , amb   IR .
a) Comproveu que els punts mitjans dels segments que tenen un extrem situat
sobre la recta r i l’altre extrem situat sobre la recta s formen un pla.
b) Trobeu l’equació general (és a dir, que té la forma Ax+By+Cz=D) del pla de
l’apartat
anterior.

Solució PAU CAT TEC JUNY 2014 3.5

2.8.11
Trobeu els punts de la recta r: x–1 = y+2 = z que equidisten dels plans
 1 : 4x - 3z - 1  0 i  2 : 3x  4y - 1  0.

Solució PAU CAT TEC JUNY 2007 2.4

2.8.12
Determineu els extrems d’un segment AB sabent que el punt A pertany al pla
x 1 y  2 z
2 x  y  z  0 , el punt B pertany a la recta   i el punt mitjà del
2 2 3
segment és (0,0,0).

Solució PAU CAT TEC SET 2006 4.4

2.8.13
Trobeu els punts situats sobre la recta d’equacions paramètriques següents:
x  
 6
 y  2  1 i que disten del pla x  2 y  z  5 .
 z    1 3

2.8.14
Siguin la recta r: (x, y, z) = (5 + k, k, –2 – 2k) i els punts P = (1, 0, –1) i
Q = (2, 1, 1).
a) Calculeu l’equació paramètrica de la recta que passa pel punt Q i és
perpendicular al pla determinat per la recta r i el punt P.
b) Calculeu el punt de la recta r que equidista dels punts P i Q.

Solució PAU CAT TEC SET 2016 1.1


3 Paral·lelisme.
3.1 Paral·lelisme entre dues rectes.
Concepte de paral·lelisme en l’espai.
En el pla, diem que dues rectes són paral·leles quan no tenen cap punt en comú.
Es pot demostrar que, en el pla, dues rectes són paral·leles quan són diferents i
els seus vectors directors són proporcionals.

En l’espai, direm que dues rectes són paral·leles quan els seus vectors directors
siguin proporcionals.

En l’espai, dues rectes paral·leles o bé no tenen cap punt de tall o bé són la


mateixa recta, i sempre seran dues rectes coplanàries.

Per tant, en l’espai ens podem trobar amb tres situacions possibles:

1. Dues rectes coincidents: 3. Dues rectes paral·leles i no diferents


(i que, per tant, no tenen cap punt en
comú)

2. Dues rectes que es tallen en 4. Dues rectes que es creuen, és a dir,


un únic punt: que no siguin coplanàries:
Rectes paral·leles.
Dues rectes són paral·leles, i escriurem r // s , si els seus vectors directors són
proporcionals, és a dir, determinen la mateixa direcció.

r : ( x, y, z )  ( p1 , p2 , p3 )  a (v1 , v2 , v3 )
s : ( x, y, z )  (q1 , q2 , q3 )  b ( w1 , w2 , w3 )
   
r // s  v // w  v  k w

Proposició.
Si les rectes venen donades per les equacions cartesianes o implícites:

Ax  By  Cz  D  A' ' x  B' ' y  C ' ' z  D' ' 


r:  s: 
A' x  B' y  C ' z  D' A' ' ' x  B' ' ' y  C ' ' ' z  D' ' '

 A B C 
  A B C A B C
 A' B' C ' 
r // s  Rang   2  A' B' C '  0 i A' B' C '  0
A' ' B' ' C ' ' 
  A' ' B' ' C ' ' A' ' ' B' ' ' C ' ' '
 A' ' ' B' ' ' C ' ' ' 
 

Proposició. Recta paral·lela a una altra i que passa per un punt.



Donada una recta r : Q  k v i un punt P r , existeix una única recta s paral·lela
a r i que passa per P.

Agafem com a punt base el punt


P i com a vector director el
vector director de la recta r.

profesor10demates: ecuaciones de la recta en el espacio (con paralelismo)


https://youtu.be/2bAZpIYBe6A
3.1.1
Determina l’equació contínua de la recta paral·lela a la recta
x 3 y  2 5 z
  que passa pel punt P  (1, 5,  3) .
2 3 4

3.1.2
Donats els punts P  (3, 2,  4) , Q  (5,  1, 2) y R  (1, 4, 3) , determina la
recta paral·lela a la recta PQ que passa pel punt R.

3.1.3
Donats els punts P  (2, 3, 7) , Q  (4, 1, 3) y R  (3,  5, 4) ,
a) Determina la recta paral·lela a la recta PQ que passa pel punt R
b) Determina punt d’intersecció entre aquesta recta i el pla z  0 .

3.1.4
Donats els punts P  (3,  1,  2) , Q  (2, 3, 1) y R  (1,  5,  3) ,
a) Determina la recta paral·lela a la recta PQ que passa pel punt R.
b) Determina punt d’intersecció entre aquesta recta i el pla z  0 .

3.1.5
2 x  y  3z  2
Donada la recta r : 
x  z  1  0

a) Trobeu-ne un vector director.


b) Calculeu l’equació contínua de la recta que és paral·lela a r i que passa pel
punt P  (1,0,1)

Solució PAU CAT TEC JUNY 2011 1.1

3.1.6
x  y  z  0
Calculeu l’equació de la recta paral·lela a la recta r : 
2 x  y  z  1
que passa pel punt (0,1,0).

Solució PAU CAT TEC SET 2006 4.3

3.1.7
 x  y  3z  1
Considereu la recta de IR3 d’equacions 
x  3 y  z  5

Digueu si el punt (6,2,2) es troba o no sobre la recta paral·lela a l’anterior que


passa per l’origen. Raoneu la resposta.
PAU COU
3.1.8
x y k x  y  2
Donades les rectes r :   z i s: , determineu el valor de k per
k 1 x  z  1
tal que les rectes siguin paral·leles.
PAU COU

3.1.9
Considereu les dues rectes r i s de IR3 donades per les equacions següents:

x  y  0 x  a y  1
r: i s:
a y  z  0 y  z  a

Digueu si hi ha algun valor d’a per al qual r i s es tallin, i si hi ha algun valor d’a
per al qual siguin paral·leles. Justifiqueu la resposta.

PAU COU

3.1.10
Considereu les dues rectes r i s de IR3 donades per les equacions següents:

x  y  a z  1 3b x  2 y  3z  5
r: i s:
2 x  a y  2 z  2 x  z  2

Determineu els valors d’a i b sabent que r i s són paral·leles.


PAU COU
3.2 Paral·lelisme entre plans.

Dos plans paral·lels.


Direm que dos plans  1 i  2 són paral·lels quan són el mateix pla (  1   2 ) o no
tenen cap punt en comú:

1   2  

Dos plans  1 : Ax  By  Cz  D i  2 : A' x  B' y  C ' z  D' seran paral·lels quan


els seus vectors normals siguin proporcionals:

 1 //  2 
( A, B, C ) // ( A' , B' , C ' ) 
( A, B, C )  k ( A' , B' , C ' ) 
A B C
 
A' B' C '

Proposició.
Si els plans els tenim en forma vectorial:

 1 : ( x, y, z )  ( p1 , p2 , p3 )  a(u1 , u2 , u3 )  b(v1 , v2 , v3 )
 2 : ( x, y, z )  (q1 , q2 , q3 )  c( w1 , w2 , w3 )  d (t1 , t 2 , t3 )
 u1 v1 w1 t1 
 
 1 //  2  Rang  u2 v2 w2 t2   2
u t3 
 3 v3 w3

Pla paral·lel a un altre i que passa per un punt.


Donat un pla Ax  By  Cz  D i un punt P que no pertany al pla, existeix un
únic pla paral·lel a aquest i que passa per P.

Agafarem com a vector normal


el vector normal del pla i
deduirem el valor D amb el punt
P.
3.2.1
Determina el pla que passa per Q  (4,3,2) i és paral·lel al pla
 : x  26  y  4 z

3.2.2
Determina el pla que passa per Q  (1,2,3) i és paral·lel al pla
 : 2x  5 y  4z  5

3.2.3
Un segment d’extrems A= (5, 3, 1) i B= (4, 2, –1) es divideix en tres parts iguals
mitjançant dos plans perpendiculars a aquest segment. Calculeu les equacions
dels dos plans i la distància entre ells.
Solució PAU CAT TEC SET 2003 3.6

3.2.4
Quines condicions han de complir els coeficients de les equacions cartesianes de
dos plans de l’espai per tal que siguin paral·lels? Justifiqueu la resposta. Com a
exemple, determineu els valors de a que fan que els dos plans d’equacions

(a  2) x  a y  a z  5 i a x  y  z  3

siguin paral·lels. Per a cada un d’aquests valors de a calculeu la distància entre


els dos plans paral·lels.

PAU COU
3.3 Paral·lelisme entre recta i pla.

Direm que la recta r és paral·lela al pla  quan no tinguin cap punt en comú:

r   

Un pla serà paral·lel a una recta


quan el vector director de la
recta i el vector normal del pla
siguin perpendiculars.

r : P  k v  ( p1 , p2 , p3 )  a (v1 , v2 , v3 )
 : Ax  By  Cz  D

r //   v  ( A, B, C ) 
(v1 , v2 , v3 )  ( A, B, C )  0

Observació: A més a més, hem


de comprovar que el punt P no
pertanyi al pla.

3.3.1
Determineu per a quins valors del paràmetre a el pla π : ax +2y +z = a és
paral·lel a la recta r :
 x  ay  z  1

ax  z  a  1

Solució PAU CAT TEC SET 2001 4.1

3.3.2
Digueu si existeix cap valor de  per al qual la recta següent:
2 x  y  3z  12

x  4 y  5z  3
és paral·lela al pla  x  12 y  13z  5 . Expliqueu bé el perquè de la vostra
resposta.

PAU COU
3.4 Pla paral·lel a dues rectes i que passa per un punt.
 
Donades dues rectes r : P  v i s : Q  w , que es creuen, i un punt R, existeix
un únic pla que passa per R i és paral·lel a les dues rectes.

El vector normal del pla serà


perpendicular als vectors
directors de les rectes, per tant,
prendrem el seu producte
vectorial:
i j k
  
n  v  w  v1 v2 v3
w1 w2 w3

I com a punt base del pla


prendrem el punt R.

3.4.1
4 x  y  z  0 y
Donades les rectes r1 :  i r2 : x  z
2 x  y  2 z  1  0 3

Calculeu l'equació del pla paral·lel a les dues rectes que passa per l'origen.

PAU CAT TEC SET 1999 5.2


3.5 Pla que conté una recta i és paral·lel a una altra.
 
Donades dues rectes r : P  v i s : Q  w , que es creuen, existeix un únic pla
que conté a r i és paral·lel a s.

El vector normal del pla serà


perpendicular als vectors
directors de les rectes, per tant,
prendrem el seu producte
vectorial:
i j k
  
n  v  w  v1 v2 v3
w1 w2 w3

I com a punt base del pla


prendrem el punt base de la
recta.

3.5.1
Trobeu l’equació general (és a dir, de la forma Ax  By  Cz  D  0 ) del pla
x 1 x  y  z  0
que conté la recta r1 :  y  2  z i és paral·lel a la recta r2 : 
2 x  2 y  z  0

Solució PAU CAT TEC JUNY 2010 1.1

3.5.2
Determineu el valor de a per al qual el pla  de IR3 d’equació x  y  z  1 és
3x  2 y  a z  5
paral·lel a la recta  . Escriviu després l’equació del pla que
2 x  y  z  3
conté aquesta recta i és paral·lel al pla  .

PAU COU

3.5.3
Trobeu l’equació cartesiana del pla de l’espai que conté la recta d’equacions
2y 2 x  y  3z  6
x  2( z  1) i és paral·lel a la recta 
3 x  2 y  z  0

PAU COU
3.6 Recta paral·lela a dos plans.

Direm que la recta r és paral·lela a dos plans  1 i  2 quan no tingui cap punt en
comú amb ells:
r  1   , r   2  

Donat un punt P que no pertany als plans  1 i  2 , existeix una única recta r
paral·lela a tots dos plans i que passa per P.

El vector director v de la recta
és perpendicular als dos vectors
normals dels plans, per tant
podem agafar com a vector
director de la recta el producte
vectorial dels vectors normals
dels plans:

i j k
  
v  n  m  n1 n2 ne
m1 m2 m3

3.6.1
Trobeu les equacions de la recta que és paral·lela als plans x  y  0 i x  z  0
i passa pel punt (2,0,0) .
4 Perpendicularitat.
4.1 Perpendicularitat entre plans.
Perpendicularitat entre dos plans.
Dos plans són perpendiculars si els vectors normals a cada pla són
perpendiculars entre ells.

 1 : Ax  By  Cz  D
 2 : A' x  B' y  C ' z  D'
 1   2  ( A, B, C )  ( A' , B' , C ' )
 ( A, B, C )  ( A' , B' , C ' )  0

Exemple resolt.
Determineu el valor de A per al què els plans  1 : x  2 y  z  2 i
 2 : Ax  y  5z  8 siguin perpendiculars.

Solució:
1 : x  2 y  z  2  n1  (1,2,1)
 2 : Ax  y  5z  8  n2  ( A,1,5)
 1   2  n1  n2  n1  n2  0  (1,2,1)  ( A,1,5)  0
 1A  (2)(1)  1 5  0
 A  2  5  0  A  7

4.1.1 Pla perpendicular a dos plans donats.


Responeu a les qüestions següents:
a) Calculeu l’equació cartesiana (és a dir, que té la forma Ax + By + Cz = D) del
pla que passa pel punt de coordenades (0, 0, 1) i és perpendicular als plans
3x + y – z = 1 i x + y + 2z = 5.
b) Suposeu que un pla π1 és perpendicular a un segon pla π2 i que el pla π2 és a la
vegada perpendicular a un tercer pla π3. Expliqueu raonadament si
necessàriament els plans π1 i π3 han de ser perpendiculars entre ells.

Solució PAU CAT TEC SET 2016 1.6


4.1.3
Siguin π1el pla 2x +3y –z=4 i π2 el pla x– 2y– 4z= 10.
a) Comproveu que els plans π1 i π2 són perpendiculars.
b) Trobeu l’equació contínua de la recta paral·lela als plans π1 i π2 i que passa
pel punt P= (–1, 3, 2).

Solució PAU CAT TEC JUNY 2013 5.1


4.2 Perpendicularitat entre dues rectes.

Perpendicularitat entre dues rectes.


Dues rectes són perpendiculars si els seus vectors directors són perpendiculars.

r : ( x, y, z )  ( p1 , p2 , p3 )  a (v1 , v2 , v3 )
s : ( x, y, z )  (q1 , q2 , q3 )  b ( w1 , w2 , w3 )
   
r  s  v  w vw0

4.2.1
y  3 1 z x3 z 1
Siguin r1: : x  2   i r2 :  y 1  .
2 2 2 2

a) Comproveu que r1 i r2 són perpendiculars.


b) Comproveu que es tallen mitjançant la determinació del punt de tall.

Solució PAU CAT TEC JUNY 2011 1.5

4.2.2
Considereu en l’espai IR3 les rectes r i s, les equacions respectives de les quals
són:

x  2 y  m z  0
r : ( x, y, z)  (4,1,0)   (m,1,1) , s:
x  y  z  1

en què m és un paràmetre real. Estudieu si hi ha cap valor d’aquest paràmetre


per al qual les rectes siguin perpendiculars i es tallin.

Solució PAU CAT TEC SET 2009 1.2


4.3 Perpendicularitat entre recta i pla.

Perpendicularitat entre recta i pla.


Una recta és perpendicular a un pla quan el seu vector director és proporcional al
vector normal al pla.

r : P  k v  ( p1 , p2 , p3 )  a (v1 , v2 , v3 )
 : Ax  By  Cz  D

r    v //( A, B, C ) 
(v1 , v2 , v3 )  k ( A, B, C )

Recta que passa per un punt i és perpendicular a un pla.


Donat un punt P  ( p1 , p2 , p3 ) i un pla  : Ax  By  Cz  D , volem una recta r
que passi pel punt i sigui perpendicular al pla.
Agafarem com a vector director de la recta el vector normal del pla i com a punt
base de la recta el punt P.

Pla que passa per un punt i és perpendicular a una recta.


Donat un punt P i una recta r, volem un pla que passi pel punt i sigui
perpendicular a la recta.
Agafarem com a vector normal del pla el vector director de la recta, i com a punt
base del pla el punt P.

4.3.1
a) Calculeu l’equació general del pla π que passa pel punt (8, 8, 8) i té com a
vectors directors u = (1, 2, –3) i v = (–1, 0, 3).
b) Determineu el valor del paràmetre a perquè el punt (1, –5, a ) pertanyi al pla
π i calculeu l’equació paramètrica de la recta que passa per aquest punt i és
perpendicular al pla π.

PAU CAT TEC JUNY 2020 #3

4.3.2
x 1 y  3
Sigui r la recta de l’espai que té per equació r :   z i sigui P el punt
2 1
de coordenades (6, 0, –1).
a) Trobeu l’equació cartesiana (és a dir, que té la forma Ax + By + Cz = D) del
pla que passa pel punt P i talla perpendicularment la recta r.
b) Trobeu l’equació paramètrica del pla que passa pel punt P i conté la recta r.

Solució PAU CAT TEC JUNY 2015 2.2


4.3.3
Sigui π: 3x –2y +z = 10.
a) Trobeu l’equació contínua de la recta r perpendicular a π que passa pel punt
P= (–1, 3, 2).
b) Trobeu també l’equació cartesiana (és a dir, de la forma Ax+By+Cz+D= 0)
del pla π1 paral·lel a π que passa pel mateix punt P.
Solució PAU CAT TEC JUNY 2013 3.1

4.3.4
Considereu els punts A= (–1, 2, 4) i B= (3, 0, –2).
a) Trobeu l’equació del pla format per tots els punts que equidisten de A i B.
b) Donat un punt C = (x,y,z), dividim el segment AC en tres parts iguals i
obtenim els punts A, A1, B i C. Trobeu el punt C.
Solució PAU CAT TEC JUNY 2013 5.5

4.3.5
Considereu les rectes de l’espai següents:
x 1 z 1 x  4 y 1 z  2
r:  y 1  s:  
2 1 3 1 2
a) Comproveu que són secants.
b) Calculeu l’equació contínua de la recta que les talla i que és perpendicular a
totes dues.
Solució PAU CAT TEC SET 2012 4.5

4.3.6
2 x  y  3 z  2
Donada la recta  , calculeu l’equació general (és a dir, de la
x  z 1  0 
forma Ax+By+Cz+D=0) del pla perpendicular a la recta que passa pel punt
P=(1, 0, –1).

Solució PAU CAT TEC SET 2011 2.2

4.3.7
Donats el pla π: x +2y +3z –4 = 0 i els punts P= (3, 1, –2) i Q= (0, 1, 2):
a) Calculeu l’equació contínua de la recta perpendicular al pla π que passa pel
punt P.
b) Calculeu l’equació general (és a dir, de la forma Ax+By+Cz+D= 0) del pla
perpendicular a π que passa pels punts P i Q.

Solució PAU CAT TEC JUNY 2010 4.1

4.3.8
x 1 y  2 z  5
Donats el punt P= (1, 2, 3) i la recta r :   :
2 3 1
a) Trobeu l’equació cartesiana (és a dir, de la forma Ax+By+Cz+D=0) del pla π
que passa per P i és perpendicular a la recta r.
b) Trobeu el punt de tall entre la recta r i el pla π.

Solució PAU CAT TEC JUNY 2009 4.1


4.3.9
Trobeu l’equació de la recta perpendicular al pla π: 2x –y +z +3 = 0, que passa
pel punt (–1, 3,a) del pla.
Solució PAU CAT TEC JUNY 2008 2.4

4.3.10
x  y  z  1
Trobeu l’equació del pla perpendicular a la recta r :  que passa per
2 x  y  3
l’origen de coordenades.
Solució PAU CAT TEC JUNY 2007 2.1

4.3.11
Donats els punts de l'espai A = (2, 0, 0), B = (0, 1, 0) i C = (0, 0, 3).
a) Determineu l'equació del pla π que els conté.
b) Calculeu l'equació de la recta r perpendicular al pla π i que passa per l'origen.

Solució PAU CAT TEC JUNY 2001 2.3


4.4 Projecció ortogonal d’un punt en una recta. Punt simètric.

Projecció ortogonal d’un punt en una recta.

La projecció ortogonal d’un punt


P sobre una recta r és un altre
punt Q que pertany a la recta, i
de tal manera que el vector PQ
és perpendicular al vector
director de la recta.

Mètode de determinació de la projecció ortogonal d’un punt en una recta.

Primer pas: Trobem l’equació


del pla perpendicular a la recta r
que passa pel punt P.

Segon pas: Calculem el punt Q


tall de la recta amb el pla, i
aquest punt serà la projecció
ortogonal.

Nota: A l’apartat 2.7 vam resoldre aquest mateix problema mitjançant rectes
parametritzades.

4.4.1
Determina la projecció del punt P  (3, 3,  5) sobre la recta
x2 y2
r:   z 5
3 3

4.4.2
Determina la projecció del punt P  (0,2,3) sobre la recta
4 x 5 y
r:   z2
4 5

4.4.3
Determina la projecció del punt P  (5,2,3) sobre la recta
x4 2 y  z
r:  
3 3 2
Punt simètric respecte d’una recta.

Primer pas: Determinem la


projecció ortogonal Q de P en r.

Segon pas: Fem S  P  2PQ

4.4.4
Determina el punt simètric del punt P  (0, 5,  2) respecte de la recta
4 y z 3
r : 3  x   .
2 4

4.4.5
Determina el punt simètric del punt P  (2, 1, 2) respecte de la recta
x2 z2
r:  y 5  .
3 2

4.4.6
x  y  z  3  0
Considera el punt P  (3,2,0) i la recta r :  . Determina les
x  2z  1  0
coordenades del punt Q simètric de P respecte de la recta r.

4.4.7
x 5 y 3 z 3
Donats el punt P= (1, 0, –2) i la recta r :   :
2 2 3
a) Trobeu l’equació contínua de la recta que passa pel punt P i talla
perpendicularment la recta r.
b) Calculeu la distància del punt P a la recta r.

Solució PAU CAT TEC JUNY 2010 5.2

4.4.8
x 1 y 1 z
Considereu el punt P = (5, –2, 9) i la recta r :  
2 3 6
a) Calculeu l’equació de la recta s que talla perpendicularment r i que passa per
P.
b) Calculeu el punt de tall T entre les rectes r i s.

Solució PAU CAT TEC JUNY 2003 2.3


4.4.9
Calculeu el peu de la recta perpendicular a la recta

(x, y, z) = (1, –1, 1) + λ(0, 1, 1)

traçada des del punt (1, 0, –1).

Solució CAT TEC SET 2000 6.3

4.4.10
3x  3 3 y  3 3  3x
Considereu la recta r de IR3 d’equacions   .
3 3 6
Calculeu les coordenades del peu de la perpendicular traçada des de l’origen de
coordenades a la recta r i calculeu després la distància de l’origen a aquesta
recta.
PAU COU

4.4.11
Preneu tres eixos perpendiculars de coordenades a l’espai. Trobeu el peu de la
perpendicular traçada des de l’origen fins a la recta que passa pel punt (-1,7,1) i
que té (1,-1,2) com a vector director.

PAU COU

4.4.12
2 x  2 y  z  3
Considereu la recta r de IR3 d’equacions: 
2 x  y  2
Trobeu el peu de la perpendicular traçada des del punt P=(5,5,3) a la recta r, i
calculeu la distància entre P i la recta r.
4.5 Projecció ortogonal d’un punt en un pla. Punt simètric respecte
d’un pla.
Projecció ortogonal d’un punt en un pla.

La projecció ortogonal d’un punt


P sobre un pla  és un punt Q
que pertany al pla, i tal que la
recta PQ és perpendicular al pla.

Mètode de determinació de la projecció ortogonal d’un punt en un pla.

Primer pas: Trobem l’equació de


la recta r perpendicular al pla 
que passa per P

Segon pas: Calculem el punt de


tall del pla amb la recta r, i
aquest punt serà la projecció
ortogonal.

4.5.1
Determina la projecció ortogonal del punt P  (0,4,3) en el pla
 : x  5  4 y  2z .

4.5.2
Determina la projecció ortogonal del punt P  (4,  2, 3) en el pla
 : x  2 y  3z  6 .

4.5.3
Determina la projecció ortogonal del punt P  (3,  3,  2) en el pla
 : 5x  3 y  5  2 z .

4.5.4
Determina la projecció ortogonal del punt P  (4,  5, 3) en el pla
 : x z  y7.
Punt simètric respecte d’un pla.

Primer pas: Trobem la projecció


ortogonal Q del punt P

Segon pas: Fem R  P  2PQ

4.5.5
Determina el punt simètric del punt P  (4,1,0) respecte del pla
 : 5x  5 y  2 z  5

4.5.6
Determina el punt simètric del punt P  (2,4,2) respecte del pla
 : 4 x  y  3z  11 .

4.5.7
Determina el punt simètric del punt P  (2,0,4) respecte del pla
 : 5x  y  2 z  14 .

4.5.8
Un dron es troba en el punt P = (2, –3, 1) i volem dirigir-lo en línia recta fins al
punt més proper del pla d’equació π : 3x + 4z + 15 = 0.
a) Calculeu l’equació de la recta, en forma paramètrica, que ha de seguir el dron.
Quina distància ha de recórrer fins a arribar al pla?
b) Trobeu les coordenades del punt del pla on arribarà el dron.

Nota: Podeu calcular la distància que hi ha d’un punt de coordenades (x0, y0, z0)
al pla d’equació Ax + By + Cz + D = 0 amb l’expressió
Ax0  By0  Cz 0  D
A2  B 2  C 2

PAU CAT TEC JUNY 2019 1.3 (Solució: "Compendium Tec", Pàg. 532)
4.5.9
x  z  2
A IR3, siguin la recta  i el punt P = (0, 1, –1).
2 y  z  4
a) Calculeu l’equació general (és a dir, la que té la forma Ax + By + Cz = D) del
pla π perpendicular a la recta r i que passa pel punt P.
b) Calculeu el punt simètric del punt P respecte del pla x + y + z = –3.

Solució PAU CAT TEC SET 2017 2.5

4.5.10
Considereu el punt A=(1, 2, 3).
a) Calculeu el punt simètric del punt A respecte de la recta d’equació
r : (x, y, z)  ( 3   , 1 , 3 -  ).
b) Calculeu el punt simètric del punt A respecte del pla que té per equació
 : x  y  z  3.
Solució PAU CAT TEC JUNY 2014 3.2

4.5.11
Donats el pla π: 2x –y +3z –8 = 0 i el punt P=(6, –3, 7),
a) Trobeu l’equació contínua de la recta que passa per P i és perpendicular a π.
b) Trobeu el punt del pla π que està més proper al punt P.
Solució PAU CAT TEC SET 2013 1.5

4.5.12
Donats el pla π: x +2y –z = 0 i el punt P= (3, 2, 1):
a) Calculeu l’equació contínua de la recta r que passa per P i és perpendicular a
π.
b) Calculeu el punt simètric del punt P respecte del pla π.

Solució PAU CAT TEC JUNY 2009 3.4


4.6 Recta perpendicular comú a dues rectes.

Determinació de la perpendicular comú amb intersecció de plans.

Donades la recta r amb vector director


 
v , i la recta s amb vector director w ,
volem obtenir la única recta de l’espai
que talla r i s i és perpendicular a totes
dues.

Pas 1: Busquem un vector que sigui


perpendicular a les dues rectes, per
tant, calculem el producte vectorial dels
dos vectors directors:
  
u  vw

Pas 2: Determinem el pla 1 que



conté a r i la direcció u .

Pas 3: El punt Q serà el punt de tall


entre aquest pla  1 i la recta s.

Pas 4: Determinem el pla 2 que



conté a s i la direcció u .

Pas 5: El punt P serà el punt de tall


entre aquest pla  2 i la recta r.

La perpendicular comú serà la


intersecció dels dos plans anteriors:
1   2

O també és la recta que passa per P i Q

Nota: A l’apartat 2.7 vam resoldre aquest mateix problema mitjançant rectes
parametritzades.
Exercici resolt.
Considereu la recta r que passa pel punt (1,2,0) i que té (1,1,1) com a vector
director, i la recta s que passa per (0,0,1) i que té (1,2,1) com a vector director.
Escriviu les equacions paramètriques de la recta que talla les dues anteriors i que
és perpendicular a cadascuna.

Solució:
Pas 1. Vector perpendicular als dos vectors
i j k
  
directors. u  v  w  1 1 1  (1,0,1)
1 2 1

Pas 2. Pla que conté la recta r i la direcció u .
Punt base: A  (1,2,0)
 i j k
v  (1,1,1)   
Vectors directors:    n1  v  u  1 1 1  (1,2,1)
u  (1,0,1) 1 0 1
1x  2 y  z  D  1(1)  2(2)  0  D  3  D  x  2 y  z  3

Pas 3. El punt Q és el punt de tall entre el pla anterior i la recta s.


 x  0  1k x  0  1 2  2
 
s :  y  0  2k  1k  2(2k )  (1  k )  3  2k  3  1  k  2   y  0  2  2  4
z  1  k z  1  2  3
 
Q  (2,4,3)

Pas 4. Pla que conté la recta s i la direcció u .
Punt base: B  (0,0,1)
Vectors directors:
 i j k
w  (1,2,1)   
  n2  w  u  1 2 1  (2,2,2)  (1,1,1)
u  (1,0,1) 1 0 1

1x  1y  1z  D  0  0  1  D  1  D  x  y  z  1

Pas 5. El punt P és el punt de tall entre el pla anterior i la recta r.


 x  1  1k x  1  2  3
 
r :  y  2  1k  1  k  (2  k )  1k  1  k  1  2  1  k  2   y  2  2  4
 z  0  1k z  0  1 2  2
 
P  (3,4,2)
Pas 6. La perpendicular comú és la recta que passa per P i Q
Punt base: P  (3,4,2)
x  3  k

Vector director: PQ  Q  P  (1,0,1)   y  4
z  2  k

Exercici resolt.
x  y  z  1
Determineu la perpendicular comú de les rectes r :  i
 x  2 z  1
y  2 z 1
s:x  
1 2

Solució:

Passem les dues rectes a forma vectorial:


Recta r : Recta s:

i j k w  (1,1,2)

v  1 1  1  (2,1,1)  (2,1,1)
Punt base: B  (0,2,1)
1 0 2
Punt base A  (1,2,0)
(fent z=0)

i j k
    
1. Vector perpendicular a v i w : u  v  w  2  1 1  (1,3,1)  (1,3,1)
1 1 2


2. Pla  1 que conté la recta r i la direcció u :
Punt base del pla: A  (1,2,0)
Vectors directors del pla:
 i j k
v  (2,1,1)   
   n1  v  u  2  1 1  (4,1,7)  (4,1,7)
u  (1,3,1) 
1 3 1
El pla té per equació 4 x  y  7 z  D , i substituint amb A obtenim la D:
4(1)  2  7  0  D  D  2   1 : 4 x  y  7 z  2

3. El punt Q serà la intersecció entre  1 i la recta s:


x  0  

s :  y  2    4  (2   )  7(1  2 )  2  4  2    7  14  2 
 z  1  2

7 7 7  7    7  15  3 
  Q ,2  ,1  2  , , 
11  11 11 11   11 11 11 

4. Pla  2 que conté la recta s i la direcció u :
Punt base del pla: B  (0,2,1)
Vectors directors del pla:
 i j k
w  (1,1,2)   
   n2  w  u  1  1 2  (7,1,4)
u  (1,3,1) 
1 3 1
El pla té per equació  7 x  y  4 z  D2 , i substituint amb B obtenim la
D2 :
 7  0  (2)  4  1  D2  D2  2   2 : 7 x  y  4 z  2

5. El punt P serà la intersecció entre  2 i la recta r:


 x  1  2

s :  y  2  1  7(1  2 )  (2   )  4  2  7  14  2    4  2 
z  0  

7  7 7 7   3 15 7 
   P    1  2 ,2  ,    , , 
11  11 11 11   11 11 11 

Més endavant, a l’apartat 5.5.2, veurem que els punts P i Q que acabem de trobar
són molt útils per calcular la distància entre les dues rectes, perquè la distància
entre les dues rectes serà la distància entre P i Q:

  7 3  15 15  3 7    10  30  10 
PQ  Q  P    ,  ,   , , 
 11 11 11 11 11 11   11 11 11 
  10   30    10 
2 2 2

dist (r , s)  dist ( P, Q)  PQ        
 11   11   11 
100 10 10 11
    3.02
11 11 11
Exercici resolt.
Determineu l’equació de la perpendicular comú a les rectes:

x  4 y 1 z  2 y  2 z 8
r:   i s : x 1  
2 1 3 2 2

Solució:
 A  (4,1,2) B  (1,2,8)
Les rectes venen donades per: r :   i s:
v  (2,1,3) w  (1,2,2)
i j k
    1 3 2 3 2 1 
u  v  w  2  1 3   , ,   4,1,3
 2 2 1 2 1  2 
1 2 2

Pla 1 :

 A  (4,1,2) x  4 y 1 z2

 1 : v  (2,1,3)  2 1 3 0

u  (4,1,3)
 4 1 3
1 3 2 3 2 1
( x  4)  ( y  1)  ( z  2)  0 
1  3 4 3 4 1
6( x  4)  18( y  1)  2( z  2)  0  6 x  24  18 y  2 z  4  0 
6 x  18 y  2 z  38  0  3x  9 y  z  19  0

Pla  2 :

 B  (1,2,8) x 1 y  2 z  8

 2 : w  (1,2,2)  1 2 2 0
u  (4,1,3) 1 3
 4
2 2 1 2 1 2
( x  1)  ( y  2)  ( z  8)  0 
1  3 4 3 4 1
8( x  1)  11( y  2)  7( z  8)  0  8 x  8  11 y  22  7 z  56  0 
8 x  11 y  7 z  42  0

Per tant, la perpendicular comú serà la intersecció dels dos plans anteriors, i per
tant la seva equació cartesiana serà:

3x  9 y  z  19  0
t:
8x  11y  7 z  42  0
Font: I.E.S. “Miguel de Cervantes” (Granada)
4.6.1
A cada apartat, determineu la perpendicular comú de les dues rectes:

a) r : x 1  y   z , s : x  y  z .

x  3 y  9 z  8 x  3 y  2 z 1
b)   ,  
3 2 2 2 1 2

x  y  2  0 x  y  0
c) r :  , s:
z  2  0 4 y  z  6  0

x  1  t x  1 x  
x  z  1   
d) r :  , s : y  t e)  y  1 ,  y   1
 y  z  1 z  3  z    2  z  1
  

4.6.2
Doneu les equacions paramètriques de la recta que talla perpendicularment les
rectes
 x  1  
 x4 y4
r : y  3   s:   z2
z  1   2 4

i doneu també els punts d’intersecció d’aquesta recta amb r i s.
PAU COU

4.6.3
Considereu la recta r de IR3 que passa pel punt (1,1,1) i té (2,1,0) com a vector
director, i la recta r’ que passa per (2,2,2) i té (0,1,1) com a vector director.
Calculeu el punt P de r i el punt P’ de r’ que compleixen la condició que la recta
PP' és perpendicular a r i a r’.
PAU COU

4.6.4
Considereu la recta que passa pel punt (1,2,0) i que té (1,1,1) com a vector
director, i la recta que passa per (0,0,1) i que té (1,2,1) com a vector director.
Escriviu les equacions paramètriques de la recta que talla les dues anteriors i que
és perpendicular a cadascuna.
PAU COU
4.6.5
x  3  

Considereu la recta r de IR d’equacions paramètriques  y  3   i la recta r’
3

z  6  

y 1 z 1
d’equacions x  1   .
2 2
Doneu les coordenades del punt A de r i del punt B de r’ que compleixen la
condició que la recta AB és perpendicular a r i a r’.
PAU COU
4.7 Problemes PAU de perpendicularitat i paral·lelisme.

4.7.1
y2
Siguin les rectes r1 : x  1   z  5 i r2: (x, y, z) = (2 – 3λ, –1 + λ, 2).
1
a) Trobeu l’equació cartesiana (és a dir, que té la forma Ax + By + Cz = D) del
pla que conté la recta r1 i és paral·lel a la recta r2.
b) Digueu quina condició s’ha de complir perquè existeixi un pla que contingui
la recta r1 i sigui perpendicular a la recta r2. Amb les rectes r1 i r2 de l’enunciat,
comproveu si existeix un pla que contingui la recta r1 i sigui perpendicular a la
recta r2.

Solució PAU CAT TEC JUNY 2018 5.5

4.7.2
Considereu els punts P = (3, –2, 1), Q = (5, 0, 3), R = (1, 2, 3) i la recta
x  y  1  0
r:
2 y  3z  5  0

a) Determineu l’equació general (és a dir, que té la forma Ax + By + Cz = D) del


pla que passa per P i Q i és paral·lel a la recta r.
b) Donats el pla x + 2y + m · z = 7 i el pla que passa per P, Q i R, trobeu m
perquè siguin paral·lels i no coincidents.

Solució PAU CAT TEC JUNY 2018 1.4

4.7.3
x  y  z  0
Donats el pla  : x  y  2 z  5  0 i la recta r : 
2 x  y  z  10
a) Calculeu el punt d’intersecció entre el pla i la recta.
b) Trobeu l’equació contínua de la recta s continguda en el pla π, que és
perpendicular a la recta r i talla la recta r.

Solució PAU CAT TEC JUNY 2012 3.3

4.7.4
Considereu les rectes r i s amb les equacions següents:
 1
x  y  3  0 y   0
r: s: 3
2 x  z  2  0  x  2 z  3  0

a) Calculeu, de cada una de les rectes, un punt i un vector director.


b) Determineu si existeix cada un dels objectes següents i en cas afirmatiu
calculeu la seva equació:
i) El pla paral·lel a la recta s que conté la recta r.
ii) El pla perpendicular a la recta s que conté la recta r.
iii) La recta perpendicular a les rectes r i s que passa per (0, 0, 0).

Solució PAU CAT TEC JUNY 2002 2.5 (Problema)


4.7.5
Donats el pla  d'equació x + 2y + 3z – 1 = 0, la recta r d'equacions
x  2z  3
 i el punt P = (2, 1, 1), calculeu:
 y  z  4

a) Unes equacions de la recta que passa per P i és perpendicular a  .


b) L'equació del pla que passa per P i és perpendicular a la recta r.
c) Unes equacions de la recta que passa per P i talla perpendicularment r.
d) Unes equacions de la recta que passa per P, és paral·lela al pla  i tal que el
seu vector director és perpendicular al de r.

Solució PAU CAT TEC JUNY 2000 1.6 (Problema)

4.7.6
Donat el tetràedre de vèrtexs A = (0, 0, 0), B = (1, 1, 1), C = (3, 0, 0) i
D = (0, 3, 0)
a) Calculeu l'equació del pla que conté la cara BCD i la del pla que conté la cara
ACD.
b) Calculeu les equacions de dues de les altures del tetràedre, la que passa pel
vèrtex A i la que passa pel vèrtex B, respectivament. (Nota: altura d'un tetràedre
és la recta que passa per un vèrtex i és perpendicular al pla que determina la cara
oposada.)
c) Comproveu que les dues altures anteriors es tallen en un punt P.
d) Comproveu si la recta que uneix qualsevol vèrtex del tetràedre amb P és
perpendicular a la cara oposada (i és, per tant, una altura del tetràedre).

Solució PAU CAT TEC JUNY 1999 1.6 (Problema)


5 Distància.
5.1 Distància entre punt i recta.
Distància entre punt i recta.
La distància d’un punt P a una recta r serà la mínima distància entre el punt P i
qualsevol punt de la recta.

Distància entre punt i recta amb el mètode geomètric

Primer Pas: Trobarem la


projecció ortogonal Q de P en r
(Vegeu 4.4.1)

Segon Pas: La distància entre el


punt i la recta serà la distància
entre P i Q.

Fórmula de la distància entre punt i recta.

Distància de P  ( p1 , p2 , p3 ) a la recta r : ( x, y, z )  (q1 , q2 , q3 )  k (v1 , v2 , v3 )


PQ  v
d ( P, r )  
v
Exercici resolt.

Calcula la distància de P  (1,0,1) a la recta r : (2 ,1   , )

Solució:
- Pla  que passa per P i és perpendicular a r:

El seu vector normal és el vector director de la recta: n  (2,1,1) .
La seva equació és 2( x  1)  y  1( z  1)  0  2 x  y  z  3  0
Intersecció Q de  i r:
2 4 1 2
2(2 )  (1   )    3  0  6  4  0     Q   , , 
3 3 3 3 

- La distància demanada és la distància entre Q i P:


2 2 2
 4  1  2 3
dist ( P, r )  dist ( P, Q)  1     0      1     0.58
 3  3  3 3

5.1.1
Trobeu la distància de l’origen de coordenades a la recta següent:
x  y  1

x  z  2
PAU COU
5.2 Distància entre punt i pla.

Distància entre un punt i un pla.


La distància entre un punt i un pla és la mínima distància que hi ha entre el punt
i qualsevol punt del pla.

Mètode geomètric per trobar la distància entre un punt i un pla.


La distancia d’un punt P a un pla  és la distancia entre el punt P i la seva
projecció sobre el pla  . Per tant:
a) Trobem la projecció ortogonal
Q de P en  (Vegeu 4.5)

b) Calculem la distància entre P i


Q.

Fórmula de la distància d’un punt a un pla.

Distància d’un punt P  ( p1 , p2 , p3 ) a un pla  : ax  by  cx  d  0

a p1  b p2  c p3  d
d ( P,  ) 
a 2  b2  c2
Atenció!: Observa que l’equació del pla està igualada a zero, és a dir, el valor d està a l’esquerra de la igualtat.

5.2.1
Determina la distància entre el punt P= ( -5 , -1 , 2 ) i el pla  : 5x  y  z  11

5.2.2
Determina la distància entre el punt P= ( -2 , -1 , -3 ) i el pla
 : 4 x  17  3 y  5z

5.2.3
Considereu el pla que té com a vectors directors u = (–1, 3, 2) i v = (2, 1, 0) i que
passa pel punt A = (1, 0, 3).
a) Calculeu l’equació de la recta que és perpendicular al pla i passa pel punt A.
b) Calculeu la distància del punt P = (1, 5, 0) al pla.

Nota: Podeu calcular la distància d’un punt de coordenades (x0, y0, z0) al pla
Ax0  Ay0  Cz 0  D
d’equació Ax + By + Cz + D = 0 amb l’expressió
A2  B 2  C 2

Solució PAU CAT TEC SET 2018 3.3


5.2.4
Siguin el pla d’equació π: x + y – z = 0 i el punt P = (2, 3, 2).
a) Calculeu el punt simètric del punt P respecte del pla π.
b) Calculeu l’equació cartesiana (és a dir, que té la forma Ax + By + Cz = D)
dels dos plans paral·lels a π que estan a una distància 3 del punt P.

Nota: Podeu calcular la distància d’un punt de coordenades (x0, y0, z0) al pla
Ax0  Ay0  Cz 0  D
d’equació Ax + By + Cz + D = 0 amb l’expressió .
A2  B 2  C 2

Solució PAU CAT TEC JUNY 2018 5.2

5.2.5
Donat el pla  : 2 x  y  z  5 :
a) Calculeu l’equació del pla paral·lel al pla π que passa pel punt P=(1, 0,−1).
b) Determineu també la distància entre el punt P i el pla π.

Solució PAU CAT TEC JUNY 2011 4.2

5.2.6
x  y  1  0
Determineu l’equació del pla perpendicular a la recta r :  que passa
x  z  2  0
pel punt (1,1,2). Quina distància hi ha d’aquest pla a l’origen de coordenades?

Solució PAU CAT TEC JUNY 2006 3.3

5.2.7
Considereu la recta r de l'espai que passa pel punt P = (1, 1, 3) i té per vector

director v  ( 1  a , a , 1 ) . Sigui π el pla que té per equació 2x +y –z = 1.
a) Determineu per a cada valor del paràmetre a la posició relativa de la recta r
respecte al pla π (paral·lela, continguda o amb un punt d'intersecció).
b) Hi ha alguna de les rectes r que sigui perpendicular al pla π ?
c) Calculeu la distància que hi ha entre el punt P i el pla π.

Solució PAU CAT TEC SET 2000 6.5 (Problema)

5.2.8
a) Sigui P un punt de l’espai, i  , un pla. Definiu el concepte de distància del
punt P al pla  .
b) Sigui P el punt de coordenades (1,1,0), i  , el pla d’equació x+y+z=5. Trobeu
la distància de P a  .

Solució PAU CAT TEC JUNY 1998 6.4


5.3 Distància entre dos plans.

Distància entre dos plans paral·lels.


És la mínima distància possible entre qualsevol punt del primer pla i qualsevol
punt del segon pla.

Si els plans són paral·lels: És la


distància entre un punt qualsevol
del primer pla al segon pla

Si els dos plans no són paral·lels


la distància serà zero.

Exercici resolt.
Calcula la distància entre els següents dos plans:

 1 : y  3z  0  2 : 2 y  6z  5  0

Solució:
Els plans són paral·lels perquè els vectors normals són proporcionals. Per tant, la
distància entre tots dos serà la distància entre un punt del primer pla al segon pla.
Prenem P  (0,3,1) com punt de  1 .

6  6(1)  5 5
dist ( 1 ,  2 )  dist ( P,  2 )    0.79
4  36 40
Determinació dels plans paral·lels a un pla donat a una distància fixa.

Exercici resolt.
Determineu l’equació dels plans paral·lels al pla  : 3x  2 y  4 z  12  0 que
equidisten 5 unitats de  .

Solució:

Els plans paral·lels a  tindran per equació  ': 3x  2 y  4 z  D  0

Obtenim un punt qualsevol del pla  fent x  y  0 i obtenim


 : 3  0  2  0  4 z  12  0  z  3  Q  (0,0,3)

3 0  2  0  4 3  D 12  D 12  D
5  dist ( ,  ' )  dist (Q,  ' )   
32  2 2  4 2 32  2 2  4 2 29
12  D 12  D  5 29  D  12  5 29
5  5 29  12  D  
29 12  D  5 29  D  12  5 29

Per tant els dos plans buscats són:

 ': 3x  2 y  4 z  12  5 29  0 i  ': 3x  2 y  4 z  12  5 29  0

5.3.1
Trobeu les equacions dels plans paral·lels a  : x  2 y  2 z  6 situats a 10
unitats de distància d’aquest.

5.3.2
Trobeu les equacions dels plans paral·lels a π: 2x –y +2z = 3 situats a 6 unitats
de distància d’aquest.
Solució PAU CAT TEC SET 2007 3.1

5.3.3
Trobeu les equacions d’un pla paral·lel al pla d’equació 2 x  2 y  z  8  0 i que
dista d’aquest sis unitats. N’hi ha més d’un, de pla, que compleixi aquestes
condicions?

Solució PAU CAT TEC JUNY 1998 3.2


5.4 Distància entre recta i pla.

Distància entre una recta i un pla.


És la mínima distància entre qualsevol punt de la recta i qualsevol punt del pla.
Si la recta no és paral·lela al pla, la distància serà zero.

Si la recta és paral·lela al pla, la


distància serà la distància entre
qualsevol punt de la recta al pla,
i aplicarem 5.2.

Exercici resolt.
A IR3, siguin la recta r que té per equació (x, y, z) = (1 + λ, λ, 1 – λ) i el pla π
d’equació 2x - y + z = -2 .
a) Determineu la posició relativa de la recta r i el pla π.
b) Calculeu la distància entre la recta r i el pla π.

Nota: Podeu calcular la distància d’un punt de coordenades (x0, y0, z0) al pla
Ax0  By0  Cz 0  D
d’equació Ax + By + Cz + D = 0 amb l’expressió
A2  B 2  C 2

PAU CAT TEC JUNY 2016 3.2

Solució:

Punt base de la recta: P  (1,0,1) , vector director de la recta: v  (1,1,1) .

Vector normal del pla: n  (2,1,1) .

a) Posició relativa de la recta i el pla:


   
v  n  1 2  1(1)  (1) 1  2  1  1  0  v  n

El punt base de la recta no pertany al pla: 2 1 - 0 +1  -2


Per tant la recta i el pla són paral·lels.

b) Distància entre la recta i el pla amb el mètode geomètric.


Primer pas: Recta s perpendicular al pla i que passa pel punt base de la recta:
Prenem com a vector director d’aquesta recta el vector normal del pla.
L’escrivim en forma paramètrica perquè serà la que necessitarem després per
trobar el seu punt de tall amb el pla:
 x  1  2k

s :  y  0  1k
 z  1  1k

Segon pas: Punt Q d’intersecció entre el pla i la recta perpendicular.

2(1  2k) - (0 - 1k) + (1  1k) = -2  2  4k  k  1  k  -2  6k  3  2 


5
k
6

 x  1  2(5 / 6)  2 / 3
  2 5 1 
s :  y  0  1(5 / 6)  5 / 6  Q   , , 
 z  1  1(5 / 6)  1 / 6  3 6 6 

Tercer pas: La distància entre la recta i el pla serà la distància entre P i Q:

dist (r ,  )  dist ( P, Q)  PQ 

 2 5 1   2 5 1   5 5 5 
PQ  Q  P   , ,   (1,0,1)   1 , , 1    , , 
 3 6 6   3 6 6   3 6 6 
 5  5  5
2 2 2
25 5
PQ           2.04
 3  6  6  6 6

b) Distància entre la recta i el pla amb la fórmula:

Abans de tot hem d’igualar l’equació del pla a zero:


2x - y + z = -2  2x - y + z  2 = 0

2  1  (1)  0  1  1  2 5
dist ( P,  )  
2  (1)  1
2 2 2
6

5.4.1
Siguin r i s les rectes de IR3 que tenen les equacions següents:
z 3 x  3 y  2 z 1
r : x 5  y 5  i s:  
2 2 3 1

a) Estudieu el paral·lelisme i la perpendicularitat entre les rectes r i s.


b) Trobeu l’equació general (és a dir, que té la forma Ax+By+Cz=D) del pla π
que conté la recta r i és paral·lel a la recta s. Calculeu la distància entre la recta s
i el pla π obtingut.
Solució PAU CAT TEC SET 2014 5.1
5.4.2
x 1 y  2
Considereu la recta r :   z  a i el pla π: 2x +y −5z =5.
3 1
a) Estudieu la posició relativa de la recta r i el pla π en funció del paràmetre a.
b) Quan a=3, calculeu la distància de la recta r al pla π.

Solució PAU CAT TEC SET 2011 2.5

5.4.3
x  3 y 1 z  2
Trobeu la distància entre la recta r :   i el pla
4 3 3
π: 3x + 4y + 7 = 0.
Solució PAU CAT TEC JUNY 2005 1.3

5.4.4
x  3 y 1 z  2
Trobeu la distància entre la recta r :   i el pla
2 3 3
π : 2x – 3y + 3z + 5 = 0.
Solució PAU CAT TEC SET 2005 3.3

5.4.5
Comproveu que la recta que passa pels punts A=(4,0,0) i B=(0,2,2) és paral·lela
al pla d'equació x –3y +5z =2, i calculeu la distància entre la recta i el pla.

Solució PAU CAT TEC SET 2002 1.3


5.5 Distància entre dues rectes.

Distància entre dues rectes.


La distància entre dues rectes és la mínima distància entre qualsevol punt de la
primera recta i qualsevol punt de la segona. Si les rectes es tallen, la distància
serà zero.

Mètode 1: Mitjançant la perpendicular comú.

Trobem la recta perpendicular


comú a r i s i els punts de tall P i
Q respectius (Vegeu apartat 4.6).

La distància entre r i s serà la


distància entre P i Q:

dist (r, s)  dist ( P, Q)

Mètode 2: Amb el producte mixt.

Si r és una recta que passa per P  ( p1 , p 2 , p3 ) i té per vector director



u  (u1 , u 2 , u 3 )
i s és una recta que passa per Q  (q1 , q 2 , q3 ) i té per vector director

v  (v1 , v 2 , v3 )
 
Si u i v són proporcionals, és a dir les rectes r i s són paral·leles, la distància
entre r i s serà la distància entre qualsevol punt de la r a s.
 
Si u i v no són proporcionals, la distància entre les dues rectes és la distància
entre els respectius punts de contacte amb la recta perpendicular comú a totes
dues, o si fem servir el producte vectorial, amb la fórmula:
 
det( PQ, u , v )
d (r , s)   
u v

Per tant, dues rectes r i s no paral·leles es tallen en un punt si i només si


 
det( PQ, u , v )  0
Exercici resolt.
Trobeu la distància entre les rectes

x  4 y 1 z  2 y  2 z 8
r:   i s : x 1  
2 1 3 2 2

Solució:
Les rectes venen donades per:
 A  (4,1,2) B  (1,2,8)
r :  i s : 
u  (2,1,3) v  (1,2,2)
AB  (3,3,10)
 3  3 10
 
det( AB, u , v )  2  1 3  6  9  40  10  12  18  39
1 2 2
i j k
   1 3 2 3 2 1 
u  v  2  1 3   , ,   4,1,3
 2 2 1 2 1  2 
1 2 2
 
u  v  42  (1) 2  32  26
 
det( AB, u , v )  39 39 3 26
d (r , s )      
u v 26 26 2
5.5.1
x3
Siguin les rectes r i s, expressades per  y  z  1 1 i (μ, –μ, μ),
2
respectivament.
a) Determineu la posició relativa de les rectes.
b) Calculeu la distància entre la recta r i la recta s.
PAU CAT TEC SET 2020 #6
5.5.2
Considereu les rectes
 x  1  3t
x  2 y 1 z 
r:   i  y  1  4t
2 1 2 z  5  t

a) Estudieu la seva posició relativa.
b) Trobeu l’equació del pla que conté s i és paral·lel a r.
c) Calculeu la distància entre r i s.
Solució PAU CAT TEC SET 2004 5.6 (Problema)

5.5.3
Considereu les dues rectes r1 i r2 de l’espai donades per les equacions següents:
x2
r1 :  z  2, y  1 r2 : ( x, y, z )  (1,0,0)  t (1,1,1) .
3
Calculeu la distància entre elles.
PAU COU
5.6 Problemes PAU amb distància, projecció ortogonal i punt simètric.

5.6.1
Siguin P= (3 – 2a, b, –4), Q= (a–1, 2 +b, 0) i R= (3, –2, –2) tres punts de l’espai
IR3.
a) Calculeu el valor dels paràmetres a i b per als quals aquests tres punts estiguin
alineats.
b) Trobeu l’equació contínua de la recta que els conté quan estan alineats.
c) Quan b=0, trobeu els valors del paràmetre a perquè la distància entre els punts
P i Q sigui la mateixa que la distància entre els punts P i R.
d) Si b= 0, calculeu el valor del paràmetre a perquè els punts P, Q i R determinin
un triangle equilàter.
Solució PAU CAT TEC JUNY 2009 3.6 (Problema)

5.6.2
Donats el punt P= (7, 5, 1), el pla π: x –2y –3z =10 i la recta

3x  2 y  2 z  7
r:
x  6 y  2 z  5

a) Trobeu la distància del punt P al pla π.


b) Trobeu la distància del punt P a la recta r.
c) Trobeu la distància de la recta r al pla π.
Solució PAU CAT TEC SET 2008 4.4

5.6.3
Trobeu les coordenades dels punts situats sobre la recta d’equació
(x,y,z) = (–1,1,1) +t·(1,2,1) que estan a distància 1 del pla 2x +2y +z = 5.
Solució PAU CAT TEC JUNY 2006 1.1

5.6.4
Donats els punts A = (1, 0, 0) i B (0, 0, 1):
x  1

a) Trobeu un punt C sobre la recta d’equació paramètrica  y  1   que faci
z  1  

que el triangle ABC sigui rectangle en C.
b) Trobeu l’àrea del triangle ABC.
Solució PAU CAT TEC SET 2005 3.4
5.6.5
Sigui π el pla d'equació x –y +2z = 3 i P el punt (1, 1, 0).
a) Calculeu la distància d de P a π .
b) Determineu l'equació de l'altre pla π' paral·lel a π que també dista d del punt
P.
c) Determineu l'equació de la recta r perpendicular a π que passa per P.
d) Calculeu la intersecció de la recta r amb el pla π.

Solució PAU CAT TEC SET 2001 4.6 (Problema)

5.6.6
x  2 y  3 z 1
Considereu a l'espai la recta r d'equacions   i la recta s
2 3 1
x  4 y 1 z  4
d’equacions  
2 3 1
a) Determineu el punt de tall de la recta r amb el pla z = 0.
b) Comproveu que les rectes r i s són paral·leles i calculeu la distància entre
elles.
c) Quina és l'equació del pla que conté les dues rectes?
d) Calculeu la distància del pla anterior a l'origen de coordenades.

Solució PAU CAT TEC JUNY 2001 5.5


6 Posició relativa.
6.1 Posició relativa entre recta i pla.
Posició relativa entre recta i pla mitjançant l’estudi del sistema.

a1 x b1 y c1 z d 1


Sigui r :  una recta i  : a3 x  b3 y  c3 z  d 3 un pla a l’espai.
a 2 x b 2 y c 2 z d 2

Considerem el sistema

a1 x b1 y c1z d 1  a1 b1 c1   a1 b1 c1 d1 


    
a 2 x  b 2 y  c 2 z  d 2 M   a2 b2 c2  M '   a2 b2 c2 d2 
a x  b y  c z  d a c3  a d 3 
 3 3 3 3  3 b3  3 b3 c3

1. SCD: Rang M = 3. Es tallen en un punt (que és la solució del sistema)

 # * * * Sistema
 
 0 # * * Compatible
 0 0 # *
  Determinat
SCD

2. SI: Rang M = 2 i Rang M’ = 3. La recta és paral·lela i externa al pla.

 # * * * Sistema
 
0 # * * Incompatible
 0 0 0 #
 
SI

3. SCI: Rang M = 2 i Rang M’ = 2 La recta està continguda al pla.

 # * * * Sistema
 
 0 # * * Compatible
 0 0 0 0
  Indeterminat
SCI
Posició relativa entre recta i pla mitjançant equacions vectorials.


Pla:  : Ax  By  Cz  D : vector normal n  ( A, B, C )

Recta r: Punt base P  ( p1 , p2 , p3 ) i vector director v  (u1 , u 2 , u3 )

 
n v ?
NO: Recta i pla es tallen en un punt

SI

El punt P pertany a  ? NO: Recta paral·lela externa al pla.

SI

Recta continguda al pla


6.1.1
Determineu la posició relativa del pla  : 3x  2 y  z  3 i la recta

x 1 y
r:   z 3
3 2

6.1.2
Determineu la posició relativa entre la recta i el pla per als diferents valors del
paràmetre k:
x  2 y  z  1
r:  : k x  y  2z  4
2 x  y  z  2

6.1.3
Determineu la posició relativa entre la recta i el pla per als diferents valors del
paràmetre t:
t x  y  z  1
r:  : x  y t z t
x  t y  z  1

6.1.4
Considereu el pla π: x + y + z = 1 i la recta r que passa pels punts P = (0, 0, 6) i
Q = (1, 2, 3).
a) Estudieu la posició relativa de la recta r i el pla π.
b) Calculeu la distància entre la recta r i el pla π.

Nota: Podeu calcular la distància d’un punt de coordenades (x0, y0, z0) al pla
Ax0  By0  Cz 0  D
d’equació Ax + By + Cz + D = 0 amb l’expressió .
A2  B 2  C 2

Solució PAU CAT TEC SET 2017 2.1

6.1.5
x 1 zm
Donats el pla π: x +2y –z = 3 i la recta r : y ,
2 4
a) Comproveu que el vector característic (o normal) de π i el vector director de r
són perpendiculars.
b) Estudieu la posició relativa de π i r en funció del paràmetre m.

Solució PAU CAT TEC SET 2013 1.3


6.1.6
a x  y  2
Donats el pla π: 5x +y +3z = 4 i la recta r :  , estudieu-ne la posició
2 y  z  3
relativa en funció del paràmetre a.

Solució PAU CAT TEC SET 2010 2.2

6.1.7
Sigui  el pla de l’espai que passa pel punt (0,0,3) i que conté els vectors
 
u  (1,2,5) i v  (2,1,3) . Sigui r la recta d’equacions:
4 x  y  z  0

2 x  y  z  0

a) Escriviu l’equació cartesiana del pla  (equació de la forma ax+by+cz=d).


b) Estudieu la posició relativa de r respecte a  (heu de dir si r és paral·lela a  ,
si està continguda en  o bé si talla  ).
PAU CAT TEC JUNY 1998 6.3

6.1.8
Estudieu, segons els diferents valors que pot tenir el paràmetre m, les posicions
relatives del pla  i de la recta r que es donen a continuació:

3x  y  1
 : mx  3 y  2 z  1 r:
2 x  y  m z  1
PAU CAT TEC JUNY 1997 3A.4
6.2 Posició relativa entre dos plans.

Posició relativa entre dos plans mitjançant l’estudi de sistema.

Un pla en l’espai queda determinat algèbricament amb una equació lineal de tres
variables:
 : ax  by  cz  d

Per tant, l’estudi de la posició relativa de dos plans equival a discutir el sistema
format per les seves equacions:

 1 : a1 x  b1 y  c1 z  d1   a1 b1 c1 d1 
 o en forma matricial :  
 2 : a2 x  b2 y  c2 z  d 2   a2 b2 c2 d 2 

1. SCI : Rang M = Rang Ma = 2. Els dos plans es tallen en una recta.

 # * * *
 
 0 # * *

2. SI : Rang M = 1, Rang Ma = 2. Els plans són paral·lels i no coincidents.

 # * * *
 
 0 0 0 # 

3. SI : Rang M = Rang Ma = 1. Els plans són coincidents.

 # * * *
 
 0 0 0 0
Posició relativa entre dos plans mitjançant equacions vectorials.

Pla  1 : A1 x  B1 y  C1 z  D1 : vector normal n  ( A1 , B1 , C1 )

Pla  2 : A2 x  B2 y  C2 z  D2 : vector normal m  ( A2 , B2 , C2 )

 
nkm ?
NO: Els dos plans es tallen en una recta

SI

(a1, b1, c1, d1 )  k (a1, b1, c1, d1 ) ? NO: Dos plans paral·lels i no
coincidents
(és a dir, mirem si són equacions
equivalents)

SI

Els dos plans són coincidents


6.2.1
Estudia la posició relativa dels dos plans. En el cas que es tallin en una recta,
determina la seva equació vectorial.

 x  y  5z  4 x  y  5z  4 x  y  5z  4  0
a)  b)  c) 
3x  y  15 z  1 3(5 z  y  x)  1 5z  4  x  y

6.2.2
Determineu m i n de forma què els plans  1 : 6 x  m y  4 z  9  0 i
 2 : 9 x  3 y  n z  n  0 siguin paral·lels.

6.2.3
Considereu els plans d'equacions:

π1: x + 2y – z = 3 i π2 : ax + (a – 2)y + 2z = 4.

a) Hi ha algun valor del paràmetre a per al qual la intersecció dels plans π1 i π2


no és una recta?
b) Calculeu un vector director de la recta que s'obté quan es fa la intersecció de
π1 i π2 per al valor del paràmetre a=0.

Solució PAU CAT TEC JUNY 2002 3.3

6.2.4
Discutiu la posició relativa dels plans

 1 : ax  y  az  0 i  2 : (a  3) x  (1 / a) y  z  1

segons els valors de a ( a  0 ).


PAU COU
6.3 Posició relativa entre dues rectes.

Posició relativa entre dues rectes mitjançant l’estudi del sistema.


a x b1 y c1 z d 1 a 3 x b 3 y c 3 z d 3
r: 1 s:
a 2 x b 2 y c 2 z d 2 a 4 x b 4 y c 4 z d 4

Considerem el sistema:
a 1 x  b 1 y  c 1 z  d 1  a1 b1 c1 
a x  b y  c z  d  
 2 a b c2 
M  2 2
2 2 2

a 3 x  b 3 y  c 3 z  d 3 a b c3 
 3 3 
a 4 x b 4 y c 4 z d 4 a b c4 
 4 4
 a1 b1 c1 d1 
 
 a 2 b2 c 2 d 2 
M ' 
a b3 c3 d 3 
 3 
a b c d 
 4 4 4 4

1. SCD. Rang M = 3, Rang M’ = 3. Les dues rectes es tallen en un únic punt.

2. SCI. Rang M = 2, Rang M’ = 2. Les dues rectes són coincidents.

3. SI. Rang M = 2, Rang M’ = 3. Les dues rectes són paral·leles i no coincidents.

4. SI. Rang M = 3, Rang M’ = 4. Les dues rectes es creuen.


Posició relativa entre dues rectes mitjançant equacions vectorials.

Recta r: Punt base P  ( p1 , p2 , p3 ) i vector director u  (u1 , u2 , u3 )

Recta s: Punt base Q  (q1 , q 2 , q3 ) i vector director v  (v1 , v 2 , v3 )

 
det(u , v , PQ)  0 ?
NO: Les dues rectes es creuen.

SI

 
u kv ? NO: Les dues rectes es tallen en un únic
punt.

SI

El punt P pertany NO: Les dues rectes són paral·leles i no


a la recta r ? coincidents.

SI

Les dues rectes són coincidents.


6.3.1
Determineu la posició relativa de les rectes

2 x  z  9 x  y  1
r: i s:
y  2  x  2 y  2 z  0

6.3.2
Determineu la posició relativa de les rectes

x  2 y  z  1 3x  2 y  5
r: i s:
2 x  y  3 x  5 y  2z  0

6.3.3
Determineu la posició relativa de les rectes

x 1 z 3 z
r: y i s : x  3  y 
2 3 4

6.3.4
Estudieu la posició relativa de les rectes, en funció del paràmetre t:

t x  y  z  1 x  y  t z  1
r: i s:
x  t y  z  1 x  y  z  t
Posició relativa de dues rectes que depenen d’un paràmetre.

Exercici resolt.
Estudieu la posició relativa de les rectes

x  y  0 x  ay  1
r: s:
a y  z  0 y  z  a

en funció del paràmetre a


PAU COU

Solució:

i j k

Vector director de r: v  1  1 0  (1,1, a )
0 a 1

Punt base de r: (Prenent y  0 )  P  (0,0,0)

i j k

Vector director de s: w  1  a 0  (a,1,1)
0 1 1

Punt base de s: (Prenent y  0 ) Q  (1,0,a)

PQ  Q  P  (1,0,a)
1 a 1
1 1 0  0  a 1
a 1 a

Si a 1  Les dues rectes es creuen.



v  (1,1,1)
Si a  1     Els vectors directors són proporcionals, per tant són
w  (1,1,1)
paral·leles. I com que P  (0,0,0) no pertany a la recta s:

0  a0  1
s:
0  0  1

Les dues rectes són paral·leles i no coincidents.


6.3.3
Siguin r i s dues rectes de l’espai les equacions respectives de les quals, que
depenen d’un paràmetre real b, són les següents:

b x  y  3z  1 x y  b z 1
r: s:  
x  2 y  5z  1 1 b 1 1

a) Trobeu el punt de tall de la recta r amb el pla d’equació x= 0 i el punt de tall


de la recta s amb aquest mateix pla.
b) Calculeu un vector director per a cada una de les dues rectes.
c) Estudieu la posició relativa de les dues rectes en funció del paràmetre b.

Solució PAU CAT TEC JUNY 2009 4.6 (Problema)

6.3.4
x  a y z 1 x  2 y b z 4
Les rectes r1 :   i r2 :   són coplanàries (és a
2 1 4 1 2 1
dir, estan incloses en un mateix pla).
a) Expliqueu, raonadament, quina és la posició relativa d’aquestes rectes.
b) Trobeu la relació que hi ha entre els paràmetres a i b.
c) Trobeu els valors de a i b si el pla que les conté passa pel punt P= (2, 4, 6).

Solució PAU CAT TEC SET 2008 4.6 (Problema)

6.3.5
Estudieu la posició relativa de les dues rectes r i s de l'espai donades per les
equacions següents:

2 x  z  9 x   y
r: s:
y 1 2 y  z  3x  5
PAU CAT TEC JUNY 1997 2B.3

6.3.6
x  2 y  a
Considereu la recta r de IR3 donada per  i la recta r’ que passa pels
z  y  1
punts (1,0,1) i (3,1,4) . Calculeu el valor d’a sabent que r i r’ es tallen. Calculeu
també el punt d’intersecció de r i r’.
PAU COU
6.3.7
z 1
Considereu la recta de IR3 donada per x  1  y  i la recta donada per
2
x4
 y  2  z  4 . Digueu si aquestes dues rectes es tallen en un punt, són
2
paral·leles o bé es creuen (dues rectes de l’espai es creuen quan no es tallen ni
són paral·leles).

PAU COU

6.3.8
Considereu, a IR3 , les dues rectes següents :
2 x  y  z  1 ax  z  0
 
x  z  4 x  y  0

on a és un nombre real. Digueu si existeix algun valor de a per al qual les rectes
es tallin.

PAU COU

6.3.9
Donades les rectes
x y kx  y  2
r1 :  z r2 : 
k 1 x  z  1

determineu el valor de k per tal que les rectes siguin paral·leles.


PAU COU

6.3.10
Donades les rectes d’equacions
ay  2 z  a  4 3 y  az  3  2a 
 
2x  3 y  3  ax  3 y  3 

busqueu els valors de a pels quals les dues rectes es tallen.

PAU COU
6.4 Posició relativa de tres plans.

Posició relativa de tres plans mitjançant l’estudi del sistema.

L’estudi de la posició relativa de tres plans equival a discutir el sistema format


per les seves equacions:
 1 : a1 x  b1 y  c1 z  d1   a1 b1 c1 d1 
  
 2 : a 2 x  b2 y  c 2 z  d 2  o en forma matricial :  a 2 b2 c 2 d 2 
 3 : a3 x  b3 y  c3 z  d 3  a b c d 
 3 3 3 3

1. SCD: Rang M = Rang Ma = 3. Els tres plans es tallen en aquest punt formant
un tríedre.

 # * * *
 
 0 # * *
 0 0 # *
 

2. SCI: Dos casos:

2.1 SCI i Rang M = Rang Ma = 2 Els tres plans es tallen en una recta.

 # * * *
  (Una única fila de zeros)
 0 # * *
 0 0 0 0
 

2.2 SCI i Rang M = Rang Ma = 1. Tots tres plans són coincidents.

 # * * *
  (Dues files de zeros)
 0 0 0 0
 0 0 0 0
 
3. SI: Els tres plans no tenen cap punt en comú. Hem d’estudiant el paral·lelisme
entre els plans:

3.1 Els tres plans són paral·lels:

3.1.1 Dues equacions són proporcionals : Dos plans coincidents i


paral·lels (però no coincidents) al tercer.

3.1.2 No hi ha equacions proporcionals : Tres plans paral·lels i no


coincidents.

3.2 Una parella de plans paral·lels :

3.3 Cap parella de plans paral·lels: Els plans es tallen dos a dos amb
rectes paral·leles. Tenim el que col·loquialment s’anomena « tenda de
campanya »
Exercici resolt.
Determina la posició relativa dels tres plans

1 : x  y  z  1
 2 : 5x  y  z  0
 3 : 3x  y  z  1

Solució:

El sistema d’equacions dels punts d’intersecció és:

x  y  z  1 1 1 1 1
  
5 x  y  z  0 , en forma matricial: A'   5  1 1 0 
3 x  y  z  1 3 1 1 1
  

1 1 1 
 
A   5  1 1   A  16  0
 3 1  1
 

Per tant, el sistema és compatible determinat, i.e. els tres plans es tallen en un
únic punt.
Exercici resolt.
Determina la posició relativa dels tres plans

1 : 2x  y  z  3
 2 : x  y  2z  1
 3 : 3 y  3z  2

Solució:
El sistema d’equacions dels punts d’intersecció és:

2 x  y  z  3  2 1 1 3
  
 x  y  2 z  1 , en forma matricial: A'   1  1 2 1 
 3 y  3 z  2  0  3 3 2
  

2 1 1
 
A   1 1 2  A  0
 0  3 3
 

Agafant les 2 primeres files i columnes, obtenim un determinant no nul:

2 1
 2  1  3 , per tant, Rang(A) = 2
1 1

Rang(A’) = 3, només cal substituir la primera columna per la quarta :

3 1 1
1 1 2  9  0
2 3 3
Per tant, el sistema és incompatible. Els tres plans no tenen punts en comú.

2 x  y  z  3
 1 i  2 es tallen en una recta, ja que el sistema associat  és
x  y  2 z  1
compatible:
 2 1 1 3
La matriu associada   té rang 3, només cal agafar les dues
 1  1 2 1
2 1
primeres columnes :  2  1  3
1 1

De la mateixa manera comprovem que també  1 i  3 es tallen en una recta, i  2 i


 3 es tallen en una recta, per tant, estem davant d’un cas de tres plans formant un
prisma triangular.
6.4.1
Estudieu la posició relativa dels plans següents, en funció del paràmetre t:
1 : x  t y  t z  1
2 : t x  y  t z  t
3 : t x  t y  z  1

6.4.2
 1 0 a  1
 
Considereu la matriu A   1 a 1  , en què a és un paràmetre real.
 4 3a 1 

a) Trobeu els valors del paràmetre a per als quals la matriu és invertible.
b) Discutiu la posició relativa dels plans π1: x + (a – 1)z = 0, π2: x + ay + z = 1 i
π3: 4x + 3ay + z = 3 en funció dels valors del paràmetre a.

PAU CAT TEC SET 2019 5.2 (Solució: "Compendium Tec", Pàg. 543)

6.4.3
Siguin els plans de IR3 π1: –y +z = 2, π2: –2x + y + z = 1 i π3: 2x – 2z = –1.
a) Calculeu la posició relativa dels tres plans.
b) Comproveu que el pla π3 és paral·lel a la recta definida per la intersecció dels
plans π1 i π2.

Solució PAU CAT TEC SET 2015 5.4

6.4.4
Digueu per a quin valor del paràmetre m els plans
 1 : x  y  mz  1 ,  2 : x  y  z  m i  3 : my  2 z  3

tenen com a intersecció una recta.


Solució PAU CAT TEC JUNY 2012 3.1

6.4.5
A l’espai es consideren els tres plans d’equacions:
 1 : x  2y  z  1 ,  2 : px  y  pz  1 i  3 : px  y  2z  1 , on p és un paràmetre
real.
a) Esbrineu per a quins valors de p els tres plans es tallen en un únic punt.
Trobeu aquest punt quan p=1.
b) Hi ha algun valor de p que faci que la intersecció comuna sigui una recta? Si
és així, escriviu l’equació vectorial d’aquesta recta.
c) Trobeu quina és la posició relativa dels tres plans quan p=1/2

Solució PAU CAT TEC JUNY 2007 2.6 (Problema)


6.4.6
Discutiu el sistema següent en funció del paràmetre p.
 x  2y  z  5

2 x  p y  2 z  10
 px  6 y  3z  12

Doneu la interpretació geomètrica del sistema en cada cas i resoleu-lo quan sigui
compatible.

Solució PAU CAT TEC JUNY 2007 1.6

6.4.7
Donada la matriu següent dependent d’un paràmetre m:
1 1 2 
 
A   2 m 2m 
m 2 2  m
 
a) Estudieu-ne el rang segons els valors de m.
b) Digueu quina és la posició relativa dels plans π1:x +y +2z = 2,
π2:2x +my +2mz =2+m i π3: mx +2y +(2+m)z= 0, segons els valors de m.

Solució PAU CAT TEC SET 2007 3.2

6.4.8
Donat el sistema
 yz2

  2 x  y  z  1
(2  2m) x  (2m  2) z  m  1

on m és un paràmetre:
a) discutiu el sistema segons els valors de m;
b) resoleu els casos compatibles;
c) en cada un dels casos de la discussió de l’apartat a), feu una interpretació
geomètrica del sistema.

Solució PAU CAT TEC JUNY 2004 3.5

6.4.9
Considerem els punts de l’espai A(1, 1, 0), B(0, 1, 2) i C(–1, 2, 1). Ens diuen que
aquests tres punts formen part del conjunt de solucions d’un sistema de tres
equacions lineals amb tres incògnites. Es demana:
a) aquests punts, estan alineats?
b) podem saber el rang de la matriu del sistema d’equacions?

Solució PAU CAT TEC JUNY 2004 3.4


6.4.10
Se sap que el sistema d'equacions
x  y  az  2 

2 x  y  8z  1 
 x  2 y  10 z  5
té més d'una solució.
Calculeu a i digueu quina és la interpretació geomètrica que té el conjunt de
totes les solucions d'aquest sistema.

Solució PAU CAT TEC JUNY 2000 3.3

6.4.11
Discutiu, segons els valors del paràmetre a, el sistema següent:
ax  y  z  4

 x  ay  z  1
x  y  z  a  2

Doneu en cada cas la interpretació geomètrica.

PAU COU

6.4.12
Estudieu en funció del paràmetre a el sistema següent:
2 x  a y  9

 5 y  a z   a
4 x  5 y  5a z  13

Doneu en cada cas la interpretació geomètrica.

PAU COU
7 Angles.
7.1 Angle entre dues rectes.
Angle entre dues rectes.

És l’angle que formen els seus vectors directors, amb valor absolut:

 
u v
cos   
uv

Exemple resolt.
Determina l’angle entre les rectes r i s d’equacions
 x  2 y  1
r : x 1  y  1  z , s : 
y  z 1

Solució:
En l’apartat 2.3 ja vam veure que aquestes rectes es tallen al punt P  ( 3 , 2 ,  1) .

Determinem el vector director de la recta r:


x 1 y  0 z 1 
x 1  y  1  z     v  (1,1,1)
1 1 1
Determinem el vector director de la recta s:
i j k
 x  2 y  1   2 0 1 0 1 2
s:  w  1  2 0   , ,   (2,1,1)

 y  z  1
0 1 1 
1 1 0 1 0 1 
Per tant:
 
v  w  1(2)  1(1)  (1)1  2  1  1  4

v  12  12  (1) 2  3

w  (2)2  (1)2  12  6
 
 v w   4 
  arccos     arccos   19.47 º
vw  3 6 
7.1.1
Determina l’angle entre les dues rectes (en graus).
3 z 4 x 1 z
r : 1  x  6  y  s:  y 1 
2 3 2
7.1.2
Determina l’angle entre les dues rectes (en graus).
7 x 8 z  x 9 y
r:  2 y  s:   3  z
2 2 3 5
Determinació d’una recta coneixent l’angle.

Exercici resolt.
x  z
Considereu la recta r de R3 d’equacions  i la recta r’ d’equacions
y  1
x 1 z 1
 y 1 
2 a

on a és una constant. Aquestes dues rectes es tallen en el punt (1,1,1) ja que


aquest punt compleix les equacions de les dues rectes. Determineu els valors de
a que fan que r i r’ es tallin en aquest punt formant un angle de 30 graus.
PAU CAT COU
Solució:
Passem les equacions de r a forma paramètrica per a extreure un vector director:

 x  1z  0  x  1 0
x  z       
   y  0z 1   y    0z   1 
y  1  z  1z  0  z  1 0
      

Un vector director de r serà doncs v  (1,0,1)
Un vector director de r’ es pot determinar agafant els denominadors de la seva

equació general: w  (2,1, a)
 
v w
Així doncs, hem de resoldre l’equació cos(30)   
vw
3
cos(30) 
2
 
v  w  1  2  0 1  1  a  a  2

v  12  0 2  12  2

w  2 2  12  a 2  a 2  5

 
v w 3 a2
cos(30)       3 2 a2  5  2 a  2 
vw 2 2 a 52

 
6 a2  5  2 a  2

Elevant al quadrat tots dos costats

 6a   2 a  2 
2
5  4(a  2) 2 
2 2

   
6 a 2  5  4(a  2) 2  3 a 2  5  2(a  2) 2 
3a 2
 5  2(a 2
 4a  4)  3a  15  2a 2  8a  8 
2

7
0  2a 2  8a  8  3a 2  15  0  a 2  8a  7  a  
1

Efectivament, per a a  7 , w  (2,1,7) i
 
v w 9 9 9 3 3 3 3
         cos(30)
vw 2  54 2  23 3
23 3 2 3 23 2

i per a a  1 , w  (2,1,1) i

 
v w 3 3 3 3 3
       cos(30)
vw 2 3 2 3 23 2

Exercici resolt.
Considereu la recta r de l’espai donada per les equacions
(1  a) x  y  z  0

x  z  0

on a és un paràmetre, i la recta r’ que té com a vector director 1, 2 ,1 i que
passa per l’origen formant un angle de 45 graus. Determina els possibles valors
del paràmetre a.

PAU CAT COU


Solució:
Passem les equacions de r a forma paramètrica per a extreure un vector director:
x  z  0  x  z 

(1  a) x  y  z  0  (1  a) x  z  y 
(1  a)( z )  z  y   z  az  z  y  az  y
x   z  x   1  0
      
 y  az   y     a  z   0 
z  1  z   1  0
      

Per tant un vector director de la recta és (1,a,1) o equivalentment v  (1, a,1) .



Un vector director de la recta r’ ja ens ve donat directament: w  1, 2 ,1 
 
v w
Així doncs, hem de resoldre l’equació cos(45)   
vw
1
cos(45) 
2
 
v  w  1  1  a  2  (1)(1)  1  2a  1  2a  2 

v  12  a 2  (1) 2  a 2  2

w  12  ( 2 ) 2  (1) 2  1  2  1  4  2

  2a  2
v w 1
cos(45)       2 a 2  2  2  2a  2
vw 2 2 a 2
2
Elevem al quadrat tots dos costats de l’equació:
2   2
2
a2  2  2a  2   4a  2  2
2
2
2a  2 
2

 
4a 2  8  2 2a 2  4 2 a  4  4a 2  8  4a 2  8 2 a  8
0
4a 2  8 2 a  8  4a 2  8  0  8 2 a  0  a  0
8 2
 
 v w 2 2 1
Efectivament, per a a=0, v  (1,0,1) i       cos(45)
vw 2 2 2 2 2

7.1.3
En un sistema de coordenades rectangulars tenim la família de rectes r:
x  3 y 1 z  2 x  y  z  1
  i la recta s : 
a 2 3  x  y  z  1

a) Determineu les dues rectes de la família r que fan un angle de 60º amb la recta
s.
b) Determineu l’angle d’aquestes dues rectes de la família r.
PAU COU

7.1.4

4 x  2 y  2 z  0
Considereu la recta r donada per  i la recta r’ de vector

 2 x  y  0
director (1, a,1) que passa per l’origen. Determineu per a quins valors de a, r i
r’ es tallen a l’origen formant un angle de 45 graus.
PAU COU

7.1.5
Considereu a l’espai  3 les dues rectes següents:
x  y  0
( x, y, z )  ( 2 , 2 ,2)   (1, a,1) i  , on a és un nombre real.
 2x  z  0
Comproveu que aquestes dues rectes es tallen per a qualsevol valor de a i
determineu a perquè formin un angle de 60 graus.
PAU COU

7.1.6
En un sistema de coordenades rectangulars tenim la recta
x  3 y 1 z  2 x y  z 1 
r:   i la recta s : .
a 2 3 x  y  z  1
Determineu els dos valors d’a per als què les dues rectes fan un angle de 60º.
PAU COU
7.2 Angle entre dos plans.

Angle entre dos plans.


L’angle que formen els plans  1 : Ax  By  Cz  D i  2 : A' x  B' y  C ' z  D'
és l’angle que formen els seus vectors ortogonals
 
n1  ( A, B, C ) i n2  ( A' , B' , C ' )

 
n1  n2
cos   
n1  n2

7.2.1
Determina l’angle entre els dos plans (en graus)

 1 : 8x  11y  7 z  18  2 : 5x  2 y  6 z  8

7.2.2
Determina l’angle entre els dos plans (en graus)

 1 : 3x  y  20 z  6  2 : 2 x  13 y  14 z  4

7.2.3
Considereu els plans π1: 5x – y –7z = 1 i π2: 2x + 3y + z = 5.
a) Determineu l’equació general (és a dir, la que té la forma Ax + By + Cz = D)
del pla que passa per l’origen de coordenades i és perpendicular als plans π1 i π2.
b) Calculeu l’angle que formen els plans π1 i π2.

Solució PAU CAT TEC JUNY 2017 1.2


7.3 Angle entre recta i pla.

Angle entre recta i pla.


L’angle que formen una recta i un pla és el complementari de l’angle que formen
un vector normal al pla i un vector director de la recta:

 
n u
cos    
nu

  90º 

7.3.1
Calculeu l’equació general (és a dir, de la forma Ax+By+Cz+D=0) dels plans
y  2
que contenen la recta r :  i que formen un angle de 45° amb el pla z=0.
z  1
Solució PAU CAT TEC JUNY 2011 4.5

7.3.2
2 x  5 y  z  3  0
Considereu la recta r :  i el pla π: 2x –y +az +2 = 0 on a és un
x  3 y  z  2  0
paràmetre.
a) Trobeu un vector director de la recta i un vector perpendicular al pla.
b) Quin ha de ser el valor de a per tal que la recta i el pla siguin paral·lels?
c) Esbrineu si existeixen valors de a per als quals la recta i el pla siguin
perpendiculars. En cas afirmatiu, calculeu-los.
d) Esbrineu si existeixen valors de a per als quals la recta i el pla formin un angle
de 30º. En cas afirmatiu, calculeu-los.
Solució PAU CAT TEC JUNY 2006 3.6

7.3.3
Una recta r passa pel punt A=(3,0,2) i té la direcció del vector (–1,1,4).
a) Trobeu quin angle forma r amb el pla horitzontal.
b) Comproveu que no passa pel punt B=(1,3,10).
c) Trobeu l’equació de la recta que passa per A i B.

Solució PAU CAT TEC JUNY 2006 1.5 (Problema)


7.3.4
Calculeu l’angle que forma el pla x  2 y  z  1 amb la recta determinada per les
x  t

equacions  y  1  t
z  2

Solució PAU CAT TEC JUNY 2002 2.4
7.3.5
Considereu la recta
2 x  5 y  z  3  0

x  3y  z  2  0 

i el pla 2 x  y  a z  2  0 , on a és un paràmetre.
a) Per a quin valor de a la recta i el pla són paral·lels? Quina serà llavors la
distància entre el punt P=(1,0,-1) de la recta i el pla?
b) Existeix algun valor de a per al qual la recta i el pla siguin perpendiculars?
c) Determineu el valor de a perquè la recta i el pla formin un angle de 30º.

Solució PAU CAT TEC SET 2000 2.6

7.3.6
x  y  z  2
Considereu la recta de  3 donada per  i el pla d’equació
x  z 1 
x  y  3 . Digueu quin angle formen aquesta recta amb aquest pla.

PAU COU

7.3.7
Trobeu els valors de a que fan que el vector perpendicular al pla z  0 i la recta
2 x  ay  z  5
de l’espai d’equacions  formin un angle de 30 graus.
3 x  2 ay  2 z  5

PAU COU
8 Problemes PAU amb àrea i volum.
8.1 Problemes PAU amb àrees.

Els problemes PAU d’àrea fan servir el “Producte vectorial” introduït a l’apartat
1.6

8.1.1
Siguin P, Q i R els punts d’intersecció del pla d’equació x + 4y + 2z = 4 amb els
tres eixos de coordenades OX, OY i OZ, respectivament.
a) Calculeu els punts P, Q i R, i el perímetre del triangle de vèrtexs P, Q i R.
b) Calculeu l’àrea del triangle de vèrtexs P, Q i R.

Nota: Per a calcular l’àrea del triangle definit pels vectors v i w podeu fer servir
1
l’expressió S  v  w , en què v  w és el producte vectorial dels vectors v i
2
w.

PAU CAT TEC SET 2019 5.5 (Solució: "Compendium Tec", Pàg. 548)

8.1.2
Un triangle d’àrea 3/2 té dos dels vèrtexs als punts P=(0, 0, 0) i Q=(2, 0, 1). El
tercer vèrtex, R, és un punt de la recta
x  y  z  0
r:
y 1
i té la primera coordenada no nul·la. Calculeu les coordenades del vèrtex R.

Solució PAU CAT TEC JUNY 2013 3.4

8.1.3
Donats els punts de l'espai A=(2, 1, 0), B=(0, 2, 0), C=(–3, 0, 0) i D=(0, –1, 0)
a) Són coplanaris? Formen un paral·lelogram?
b) Calculeu l'àrea del polígon ABCD.
c) Calculeu el punt simètric del punt E= (1, 1, 2) respecte del pla que determinen
A, B i C.
d) Calculeu la distància entre la recta que passa per E i A i la recta que passa per
B i C.

PAU CAT TEC SET 1999 2.6 (Problema)


8.2 Problemes PAU amb volum.

Els problemes PAU de volum fan servir el “Producte mixt” introduït a l’apartat
1.7

8.2.1
Considereu els punts de l’espai tridimensional A = (1, 1, 0), B = (3, 5, 0) i
z
C = (1, 0, 0) i la recta r : x  y  1  .
2
a) Trobeu el punt d’intersecció de la recta r amb el pla que passa pels punts A, B
i C.
b) Trobeu els punts P de la recta r per als quals el tetraedre de vèrtexs P, A, B i C
té un volum de 2u3.

Nota: El volum d’un tetraedre de vèrtexs P, Q, R i S es pot calcular amb


l’expressió
1
det( PQ, PR, PS )
6
Solució PAU CAT TEC SET 2018 3.5

8.2.2
Donats els vectors u=(2, –1, 0), v=(–1, 3, 4) i w=(0, 3a–1, 4a),
a) Calculeu els valors del paràmetre a perquè els vectors u, v i w siguin
linealment dependents.
b) Calculeu els valors del paràmetre a perquè un tetràedre d’arestes u, v i w
tingui un volum de 2/3 d’unitats cúbiques.
Solució PAU CAT TEC SET 2014 5.5

8.2.3
Donats els punts P= (1, –1, 2), Q= (2, 0, 1) i R= (3, 2, –1),
a) Trobeu l’equació cartesiana (és a dir, de la forma Ax+By+Cz+D= 0) del pla
que determinen.
x  5 y 1 z  5
b) Trobeu un punt S pertanyent a la recta r :   , de manera que
2 1 3
el tetràedre de vèrtexs P, Q, R I S tingui un volum igual a 1/2.

Solució PAU CAT TEC JUNY 2013 5.3

8.2.4
Una piràmide de base quadrada té el vèrtex en el pla d’equació z = 3. Tres dels
vèrtexs de la base són els punts del pla OXY: A = (1, 0, 0), B = (1, 1, 0) i
C = (0, 1, 0).
a) Feu un gràfic dels elements del problema. Quines són les coordenades del
quart vèrtex de la base, D?
 àrea base  altura 
b) Quin és el volum de la piràmide? Volum  
 3
c) Si el vèrtex de la piràmide és el punt V = (a, b, 3), quina és l’equació de la
recta que conté l’altura sobre la base?
Solució PAU CAT TEC JUNY 2005 4.6 (Problema)
8.2.5
Tenim quatre punts a l’espai: A(0, 0, 0); B(0, 0, 2); C(0, 2, 0) i D(2, 0, 0).
Es demana:
a) representeu gràficament els quatre punts;
b) calculeu el volum del tetràedre (piràmide de base triangular) ABCD;
c) trobeu l’equació del pla que passa per B, C i D;
d) calculeu la distància de l’origen al pla de l’apartat anterior.

Solució PAU CAT TEC JUNY 2004 3.6

8.2.6
Donat el pla p d'equació x+ 4y+z= 8 i sent A, B i C els punts d'intersecció
d'aquest pla amb els eixos de coordenades OX, OY i OZ, respectivament:
a) Determineu les coordenades dels punts A, B i C.
b) Determineu les equacions de la recta perpendicular al pla p que passa per
l'origen de coordenades.
c) Calculeu el volum del tetràedre determinat per OABC, on O és l'origen de
coordenades.
d) Calculeu la distància de l'origen de coordenades al pla p. Determineu l'àrea
del triangle ABC (podeu utilitzar el volum calculat en l'apartat anterior)

PAU CAT TEC SET 1998 5.6


9 Recopilacions d’exercicis.
9.1 Exercicis de repàs per al primer parcial.

9.1.1
x  y  z  1
Sigui la recta r :  i el pla x  m z  0 , on m és un paràmetre real.
x  y  z  0
Trobeu:
a) Un vector director de la recta r.
b) El valor de m per al qual la recta i el pla són paral·lels.

PAU VALENCIA TEC SET 2012

9.1.2
Donades les rectes
x4 y4 y z
r:   z4 i s:x  
3 2 2 3

es demana calcular raonadament:


a) Les coordenades del punt P d’intersecció entre les rectes r i s.
b) L’angle que formen les dues rectes.
c) L’equació implícita Ax  By  Cz  D  0 del pla que conté les dues rectes.

PAU VALENCIA TEC SET 2010

9.1.3
Siguin A,B i C els punts d’intersecció del pla d’equació x  4 y  2 z  4  0 amb
els tres eixos OX, OY i OZ, respectivament. Es demana calcular raonadament:
a) L’àrea del triangle ABC.
b) El perímetre del triangle ABC.
c) Els tres angles interiors del triangle ABC.
PAU VALÈNCIA TEC JUNY 2009

9.1.4
x  1
x  2 y  z  3  0 
Donades les rectes r :  i s :  y  2 , es demana:
3x  y  z  1  0 z    2

a) La recta paral·lela a r que passa pel punt (0,1,0)
b) El pla  que conté la recta r i és paral·lel a s.
PAU VALÈNCIA TEC JUNY 2016
9.1.5
 x  1  2  x  1
 
Donades les rectes r :  y   i s :  y  1   , on  i  són paràmetres
z  2    z  1  2 
 
reals, calculeu raonadament:
a) Les coordenades del punt de tall de r i s.
b) L’equació del pla que conté aquestes dues rectes.

PAU VALÈNCIA TEC JUNY 2012

9.1.6
x  

Donades les rectes r :  y  1   i s : x  1  y  z  3 , calculeu raonadament:
z  3

a) Un vector director de cada recta.
b) El punt d’intersecció de les rectes.
c) L’equació del pla que conté a aquestes rectes.
PAU VALÈNCIA TEC JUNY 2011

9.1.7
x  1  
x  1 y 1 z 
Donades les rectes r :   i s :  y   , i el punt P  (0,3,2) ,
3 1 2 z  0

calculeu:
a) Les equacions de la recta que passa pel punt P i és paral·lela a la recta r.
b) L’equació del pla que conté la recta r i és paral·lel a la recta s.

PAU VALÈNCIA TEC JUNY 2015

9.1.8
x  y  0 x  y  8
Donades les rectes r :  i s: , determineu:
 z  10  x  y  z  13
a) Un vector director de cada recta.
b) L’equació del pla que conté la recta s i és paral·lel a la recta r.

PAU VALÈNCIA TEC JUNY 2014


9.2 Exercicis de repàs de perpendicularitat i paral·lelisme.

9.2.1
5x  y  z  0
Considerem la recta r :  i el pla  : a x  6 y  4 z  5 .
x  y  z  4
Es demana calcular:
a) El valor de a per al què la recta i el pla siguin paral·lels.
b) El valor de a per al què la recta i el pla siguin perpendiculars.

9.1.2
Determineu si el pla 3x  2 y  z  1 és perpendicular a la recta
 x  3 y  3z
r:
 y  2 z  1

9.2.3
Per a quin valor d’a són paral·leles les rectes:
5 x  y  2a z  7  0
4 x  5 y  2 z  3  0 
r: s: a
x  3 y  4z  5  0 10 x  9 y  2 z  9  0

9.2.4
a) Determina el pla que passa pel punt de coordenades (1,1,1) i talla
x 1 y z 1
perpendicularment a la recta  
2 1 1
b) Determina el punt de tall entre la recta i el pla.

9.2.5
x  y  z  0 x  y  z  0
Donades les rectes r :  i s: ,
x  y  z  1 a x  b z  0
trobeu la relació que ha d’haver entre a i b de forma que:
a) Siguin rectes paral·leles.
b) Siguin rectes perpendiculars.

9.2.6
Determineu els valors d’a per als què els plans 2 x  ay  3z  1  0 i
5x  y  3z  2  0 siguin perpendiculars.

9.2.7
Donats els plans x  y  z  3 i x  y  a z  0 , trobeu els valors de a de forma
que:
a) Els plans siguin perpendiculars.
b) Els plans siguin paral·lels.
9.3 Exercicis de repàs de perpendicularitat, angle i distància.

9.3.1
Sigui  el pla d’equació  : 3x  2 y  4 z  12  0 . Es demana calcular
raonadament:
a) Les equacions dels dos plans paral·lels a  que disten 5 unitats de  .
b) Els tres punts A, B i C, intersecció del pla  amb cadascun dels tres eixos de
coordenades.
c) Els tres angles del triangle ABC.

9.3.2
Donats els punts P  (3,1,4) i Q  (1,0,1) i el pla  d’equació
 : x  2 y  2 z  5  0 , es demana calcular raonadament:
a) L’equació de la recta r que passa pel punt P i és perpendicular al pla  .
b) L’equació del pla  ' que passa pels punts P i Q i és perpendicular al pla  .

9.3.3
Donats els plans  1 : x  y  z  3 i  2 : x  y  a z  0 , es demana calcular
raonadament:
a) El valor de a per al què els plans  1 i  2 siguin perpendiculars, i per a aquest
valor de a, obtenir les equacions paramètriques de la recta intersecció d’aquests
dos plans.
b) El valor de a per al què els plans  1 i  2 siguin paral·lels, i per a aquest valor
de a, obtenir la distància entre tots dos.

9.3.4
x  y  z  1
En l’espai tenim la recta r :  i el pla  : x  m z  0 , on m és un
x  y  z  0
paràmetre real. Obtenir raonadament:
a) El vector director de la recta r.
b) El valor de m per al què la recta r i el pla  són perpendiculars.
c) El valor de m per al què la recta r i el pla  són paral·lels.
d) La distància entre r i  en el cas anterior.

9.3.5
Donades les rectes r i s d’equacions
x4 y4 y z
r:   z4 i s:x  
3 2 2 3
es demana calcular raonadament:
a) Les coordenades del punt P d’intersecció entre ambdues rectes.
b) L’angle que formen les rectes r i s.
c) L’equació implícita Ax  By  Cz  D  0 del pla  que conté a totes dues.
9.3.6
Es demana calcular raonadament:
a) L’equació del pla  que passa pels punts O  (0,0,0) , A  (6,3,0) i
B  (3,0,1) .
b) L’equació de la recta r que passa pel punt P  (8,7,2) i és perpendicular al
pla  .
c) El punt Q del pla  la distància del qual al punt P és menor que la distància
de qualsevol altre punt del pla  al punt P.

9.3.7
Sigui r la recta intersecció dels plans y  z  0 i x  2 y  1  0 , i sigui s la recta
x
d’equació s :  y  1   z  3 . Es demana:
2
a) Obtenir, raonadament, les equacions paramètriques de r i s.
b) La posició relativa entre totes dues rectes.
c) Calcular la distància entre les rectes r i s.

9.3.8
Donats els punts A  (2,1,1) i B  (1,0,1) , i la recta r d’equació
z2
r : x 5  y  , trobeu:
2
a) El punt C de r que equidista de A i B.
b) L’àrea del triangle ABC.
9.4 Llista d’exercicis de rectes en l’espai.

9.4.1
Determina el valor de m per a què els punts A=(m, 0, 1), B=(0, 1, 2), C=(1, 2, 3)
i D=(7, 2, 1) siguin coplanaris.

9.4.2
Donats els punts A=(m, 2,−3), B=(2, m, 1) y C=(5, 3,−2), determineu el valor de
m per al què els punts estiguin alineats i trobeu l’equació contínua de la recta
que passa per tots tres punts.

9.4.3
x  4 x  3 z 1
Escriu l’equació cartesiana de la recta  
0 0 2

9.4.4
Determina un vector director i les equacions paramètriques de la recta
x  y  0

y  z  2

9.4.5
x y 1
Escriu l’equació cartesiana de la recta  z
2 1

9.4.6
3x  y  z  1  0
Escriu la recta r :  en forma paramètrica i contínua.
x  y  z  3  0

9.4.7
Determina l’equació contínua de la recta que passa pel punt P  (1,1,1) i és
paral·lela a la recta que passa pels punts A  (2,0,1) i B  (1,2,3) .

9.4.8
Donats els punts A=(1, 3,2), B=(2, 5, 1) i C=(3, 0, -4), determineu l’equació
paramètrica de la recta que passa per C i és paral·lela a la recta determinada pels
punts A i B.
9.4.9
Determina l’angle que formen les rectes:
x2 x 1 y
r:  y 1  z s:  z
2 1 2

9.4.10
Determina l’angle que formen les rectes:
2 x  3 y  z  1  0 3x  y  z  3  0
r: s:
x  y  2 z  2  0 2 x  y  3z  1  0

9.4.11
Determina l’angle que formen les rectes:
x 1 y 1 z  2 x  y  z  0
r:   s:
1 2 3 2 x  y  3z  1  0

9.4.12
Determina el volum del tetraedre de vèrtexs A=(3, 2, 1), B=(1, 2, 4), C=(4, 0, 3)
i D=(1, 1, 7).

9.4.13
Estudia la posició relativa de les següents rectes:

 x  3  2  x  1  6 
 
r : y  1  s :  y  3  3
z  5 z  5
 

9.4.14
Estudia la posició relativa de les següents rectes:

x   x  3
 
r : y   s : y  3
z  0 z  
 

9.4.15
Estudia la posició relativa de les següents rectes:

x  3    x  2 
 
r :  y  2   s :  y  3  2
z  1  z  1
 
9.4.16
Calcula a i b para que els punts A=(1, 2, –1), B=(3, 0, –2) y C=(4, a, b) estiguin
alineats.

9.4.17
Estudia la posició relativa de les següents rectes i troba el punt de tall quan sigui
possible:

x 1 y  2 z 1 x  2 y 3 z 2
a) r:   r:  
3 2 4 1 2 3

x 1 y 1 z  2 x 4 y 4 z 5
b) r:   s:  
1 2 1 4 1 2

x z 1 x  2 y  1  0
c) r:  y 1  s:
2 3 3 y  z  1  0

 x  3  4
x 1 y z 
d) r:   s :  y  3  6
2 3 4  z  4  8

9.4.18
Determina el valor de a per al què les rectes r i s es tallen, i troba el punt de tall:
2x  1 y  3 z  2
r:x  y  za s:  
3 2 0

9.4.19
Determina els valors de m i n per als què les rectes r i s siguin paral·leles:

 x  5  4
 x y 1 z  3
r : y  3   s:  
 z   m 3 n

9.4.20
Estudia la posició relativa de la recta r determinada pels punts A  (1,1,1) i
x  2z  1  0
B  (3,1,2) i la recta s : 
y  2  0

9.4.21
Demostra que les rectes r : (1,0,5)  (4,2,6) i s : (3,1,8)  (2,1,3)
són la mateixa recta.
9.4.22
Determina el valor de a per a què les rectes r i s siguin coplanàries:
x  2 y  0 x  y  1
r: s:
y  z  2 x  2 z  a

9.4.23
Determina m per a què les rectes
x  3  
 4 x  5 y  7  0
r :  y  1  2 s:
z  2   3 y  4 z  7  m  0

siguin coplanàries.

9.4.24
Estudiar, segons els valors del paràmetre a, la posició relativa de les rectes r i s:
 x  ( a  2) 
 ax y2 za
r : y  1 s:  3 
z  a 1 a a 1

9.4.25
Determina els valors de a i b per als què les rectes siguin paral·leles:
2 x  a y  z  1
r : 4x  2 y  6  z s:
2 x  3 y  b z  3

9.4.26
Trobeu els valors de m i n per als què les rectes r i s siguin paral·leles :
 x  5  5
 x y 1 z  3
r : y  3   s:  
 z   m 3 n

9.4.27
Comproveu que les rectes
y  0 x  0
r: i s:
x  z  0 y  4
es creuen i trobeu la distància entre ambdues.

9.4.28
Determineu les coordenades del punt A ' , simètric de A  (2,0,3) respecte a la
x 1 y  2 z 1
recta r :  
1 1 2
9.5 Llista de repàs general.

9.5.1
Considereu les dues rectes r i s de IR3 donades per les equacions següents:

 x  y  az  1 3bx  2 y  3z  5
r: s:
2 x  ay  2 z  2 x  z  2

Determineu els valors d’a i b sabent que són paral·leles.

9.5.2
x  y  z  2
Considereu la recta de IR3 donada per  i el pla d’equació
x  z  1
x  y  3.
a) Determineu el seu punt de tall.
b) Digueu quin angle formen aquesta recta amb aquest pla.

9.5.3
Busqueu el punt simètric de (1,2,3) respecte del pla x  3 y  2 z  4  0

9.5.4
x 1 z 1
Busqueu l’equació del pla que conté la recta y i que passa pel
2 2
punt (-1,2,1).

9.5.5
Considereu la recta que passa pel punt (1,2,0) i que té (1,1,1) com a vector
director, i la recta que passa per (0,0,1) i que té (1,2,1) com a vector director.
Escriviu les equacions paramètriques de la recta que talla les dues anteriors i que
és perpendicular a cadascuna.
9.6 Llista de repàs general.

9.6.1
Troba l’equació del pla que conté la recta x  2  y  3  z i és paral·lel a la
recta

 x  3  2

r : y  2  
 z  2  4

9.6.2
Troba el punt del pla  : 3x  4 y  2 z  32  0 més proper al punt P  (1,2,4) .

9.6.3
Estudia la posició relativa dels plans

1 : x  2 y  z  1
 2 : mx  y  mz  1
 3 : mx  y  2 z  1
en funció de m.

9.6.4
Es considera el punt P  (5,2,1) i la recta r : x  y  2  z  3 . Troba el punt
simètric de P respecte de r.

9.6.5
Siguin els punts A  (3,4,0) , B  (3,6,3) i C  (1,2,1) els vèrtexs d’un
triangle.
a) Calcula l’equació del pla  que els conté.
b) Troba l’equació contínua de r perpendicular a  que passa per l’origen de
coordenades.

9.6.6
Donades les rectes
x  3  
 x 4 y 3 z 5
r :  y  1 , s:  
 z  4  2 3 1 4

Calcula les equacions paramètriques de la recta perpendicular comú a r i s.

9.6.7
y
Determina el valor de m per al què la recta r : x  3   z  3 i el pla
m
 : x  y  mz  24  0 formin un angle de 30º.
10 Apèndix.
10.1 Taula Paral·lelisme-Perpendicularitat-Angle.

Angle Perpendicularitat Paral·lelisme


 
Entre dos vectors: u v        
cos    u  v u v  0 u // v  u  k v
u v
Entre dues rectes:

     
x  p1 y  p 2 z  p3 u v r  s  u v  0 r // s  u  k v
r1 :   cos   
u1 u2 u3 uv

u  (u1 , u 2 , u 3 )

x  q1 y  q 2 z  q3
r2 :  
v1 v2 v3

v  (v1 , v 2 , v3 )
Entre dos plans:

   
n1  n2 1   2   1 //  2  n1  k n2
cos     
n1  n2 n1  n2  0

 1 : Ax  By  Cz  D
 2 : A' x  B' y  C ' z  D'
 
n1  ( A, B, C ) , n2  ( A' , B' , C ' )
Entre recta i pla:

 
 
n u  // r  n  v  0
cos      r
nu I, a més a més, hem de
 
n kv comprovar que el punt
base de la recta no
 1 : Ax  By  Cz  D   90º  pertany al pla
x  p1 y  p 2 z  p3
r:  
v1 v2 v3

v  (v1 , v 2 , v3 )
10.2 Que l’immens poder de la Projecció ortogonal t’acompanyi!
Si et donen un punt P i una recta r... Si et donen un punt P i un pla  ...

... determina el pla perpendicular a r que ... determina la recta perpendicular a 


   
passa per P, agafant n  v que passa per P, agafant v  n

... i troba el punt d’intersecció Q entre la ... i troba el punt d’intersecció Q entre la
recta i el pla. recta i el pla.

Aquest punt és la Projecció ortogonal de Aquest punt és la Projecció ortogonal de


P en la recta r P en el pla 
La distància entre el punt P i la recta r serà La distància entre el punt P i el pla  serà
dist ( P, r )  dist ( P, Q)  PQ dist ( P,  )  dist ( P, Q)  PQ

El punt simètric de P respecte a r serà El punt simètric de P respecte a  serà


P'  P  2PQ P'  P  2PQ
10.3 Algunes demostracions.

Demostració 1.3.3.
   
u v  u  v
 
Ho demostrarem per al cas bidimensional. Si u  (u1 , u2 ) i v  (v1 , v2 ) ,
 
u  v  (u1 , u 2 )  (v1 , v2 )  (u1  v1 , u 2  v2 )  (u1  v1 ) 2  (u 2  v2 ) 2 

 u1  2u1v1  v1  u 2  2u 2 v2  v2
2 2 2 2

 
u  v  (u1 , u2 )  (v1 , v2 )  u1  u2  v1  v2
2 2 2 2

Hem de demostrar, doncs, que


u1  2u1v1  v1  u2  2u2v2  v2  u1  u2  v1  v2
2 2 2 2 2 2 2 2

Si elevem al quadrat els dos costats de la desigualtat:


2
 u 2  2u v  v 2  u 2  2u v  v 2   u 2  2u v  v 2  u 2  2u v  v 2
 1 1 1 1 2 2 2 2
 1 1 1 1 2 2 2 2

2
 u 2  u 2  v 2  v 2   u 2  u 2  v 2  v 2  2 u 2  u 2 v 2  v 2
 1 2 1 2
 1 2 1 2 1 2 1 2

Simplificant u1  v1  u2  v2 a esquerra i dreta, hauríem de comprovar que la resta


2 2 2 2

2u1v1  2u2v2  2 u1  u2 v1  v2
2 2 2 2
compleix:
Si elevem de nou al quadrat:
2u1v1  2u2v2 2  2u1v1 2  2u2v2 2  2  2u1v1  2u2v2  4u12v12  4u22v22  8u1v1u2v2
    
2
 2 u 2  u 2 v 2  v 2   4 u 2  u 2 v 2  v 2  4 u 2v 2  u 2v 2  u 2v 2  u 2v 2
 1 2 1 2
 1 2 1 2 1 1 1 2 2 1 2 2

Simplificant 4u1 v1  4u2 v2 a esquerra i dreta, s’ha de demostrar que


2 2 2 2

8u1v1u2v2  4u1 v2  4u2 v2


2 2 2 2

Simplificant: 2u1v1u2v2  u1 v2  u2 v2
2 2 2 2

Posant el primer membre en el segon: 0  u1 v2  u2 v2  2u1v1u2v2


2 2 2 2

Finalment veiem que la part de la dreta s’adapta a la igualtat a 2  2ab  b 2  (a  b) 2


És a dir, que 0  u1v2  u2 v2 
2

I aquesta última expressió és correcta, ja que qualsevol nombre al quadrat és més gran o
   
igual a 0. Per tant, l’expressió inicial també és certa: u  v  u  v

Demostració de 1.5.2.
   
u v  u  v
Aquí utilitzarem que A  B , amb B  0  A2  B 2
   
u  v  u1v1  u2v2 , u  v  u1  u2 v1  v2
2 2 2 2

Elevant al quadrat:
  
2
u1v1  u2v2 2   u1  u2
2
v1  v2   u1  u2 v1  v2
2 22 2 2 2 2

 
u1
2
 u2
2

v1  v2
2 2
  u1 v1  u1 v2  u2 v1  u2 v2
2 2 2 2 2 2 2 2

u1v1  u2v2  2
 u v  u2 v2  2u1v1u2v2
2 2
1 1
2 2
Eliminem els termes comuns u1 v1  u2 v2 , i ens queda
2 2 2 2

2u1v1u2v2  u1 v2  u2 v1  u1 v2  u2 v1  2u1v1u2v2  0
2 2 2 2 2 2 2 2

Però això és evident, ja que:


u1 v2  u2 v1  2u1v1u2v2  u1v1  u2v2 
2 2 2 2 2

i un quadrat mai és negatiu.


Solucions.
 
1.1.1 u  v  (4,1,5)
 
1.1.2 u  v  (1,2,4)
1.1.3 (2.6 , -3.9 , -6.5 )
1.1.4 (0.7 , 2.8 , -1.4 )
1.2.1
a) b)

c)

1.2.2 a) (-1,0,1) b) (-4,2,3) c) (2,2,-1) d) (-4,-2,1)


1.2.3 No.
1.2.4 Sí.
1.2.5 x  5 .
a  1  0

1.2.6 AB  k AC  1  k  k  1, a  1, b  2
b  1  k

1.2.7
1.2.8 a  2 / 5, c  2
1.2.9 a=-2, b=0 o a=5, b=7
1.3.1 8.124
1.3.2 23.664
1.3.3 AB  BC  CD  DA  7 , i per tant és un rombe.
1.3.4 AB  (2   ) 2  (  2) 2  ( ) 2  32  8
AC  02  (2) 2  22  8

BC  (  2) 2  2  (  2) 2  32  8
i per tant els costats AB i BC són iguals.
b) 32  8  8    0
1.3.5 P  ( 0, y , 0), (1  0) 2  (2  y) 2  (3  0) 2  10  y  2  P  ( 0 , 2 , 0 )
1.3.6 P ( x,0,0)
PA  PB  PA  PB  ( x  3) 2  (0  2) 2  (0  2) 2 
49  49 
 ( x  5) 2  (0  5) 2  (0  4) 2  x  P ,0,0 
4  4 
 1 
1.3.8 P   9, ,0 
 2 
1.5.1 a) k=-2 b) No té solució.
1.5.2 52.589°
1.5.3 30.894
1.5.4 a=0
1.5.5 y  2 , z  2 ; y  2 / 3 , z   2 / 3
1.5.6 4. a  2 , a  4 / 5
1.5.10 PQ  1  1  16  3 2 , QR  9  9  0  3 2 , PR  16  4  16  6
i per tant PQ  QR i és isòsceles. PQ 2  QR 2  18  18  36  PR 2  es
verifica el Teorema de Pitàgores, i per tant és un triangle rectangle en el vèrtex
Q.
 
1.6.1 a  b  ( 20,8,12)
 
1.6.2 a b  (8,13,10)
   
1.6.3 a  b  (4,6,1) , b  a  (4,6,1)
1.6.4 ( 3, 3, 3 ) o ( 3, 3, 3 )
1.6.5 a) 19 b) (-8,-11,-1) c) 95º 46’ 5’’
1.6.6 7.28
1.6.7 21.564
1.6.8 D  (0,1,0) , perímetre = 4 6 , àrea = 2 3 .
1.6.9 AB  (0,1,0) , AC  (0,6, a  1)
i j k
AB  AC  0 1
1
0  (a  1,0,0)  Àrea  AB  AC 
1
a  12  1 a  1
2 2 2
0 6 a 1
1 3 a  1  3  a  4
a 1   a 1  3  
2 2  (a  1)  3  a  1  3  a  2
Les solucions son a  4 , a  2
1.7.1 16
1.7.2 58
1.7.3 Vol  Det   1  1
1.7.4 F  (0,0,1) , G  (3,0,3) , H  (5,0,4) , volum = 4.
1.7.5 a) Són coplanaris. b) No són coplanaris.
1.7.6 a) x  4 , b) x  1
1.7.7 C  ( 1 , 0 , 2 ) i D  ( 0 , 1 , 2 )
1.7.8 91
1.7.9 5/6
1.7.10 a)   0,   6 , b)   3
1.7.11
AB  (a  1, b,0)  a 1 b 0

AC  (a  1,0, b)  a  1 0 b  0  (a  1)ab  b(a  1)b  c   0 
 1 a b
AD  (1, a, b) 

a  0

 a b  ab  b a  0  ab(a  b  1)  0  b  0
2 2

a  b  1

Si a = 0 els punts són: (1, 0, 0), (0, b, 0), (0, 0, b) y (0, 0, b); i per tant els dos
últims coincideixen.
Si b = 0 los punts són: (1, 0, 0), (a, 0, 0), (a, 0, 0) y (0, a, 0); i també hi
coincideixen dos.
Per tant, per a que els punts siguin diferents i coplanaris és necessari que
a  b  1 , amb a i b diferents de zero.
1.7.14
1.7.15 Volum=5/6
1.7.16
AB  (a  1,1,1) a 1 1 1

 4
AC  (0,2,5)  0 2 5  0  21a  28  0  a 
3
AD  (1,3,3)  1 3 3


1 1 a  10 / 3
V  det( AB, AC, AD)  7  21a  28  7  
6 6 a  2 / 3
1.8.1 1. a)  21 , 33 , 6  , 72, 72 b) (  31 , 15 ,  35 ) , 27, 27
c) (  18 ,  15 ,  10 ) , -89, -89
1.11.1 b= 5, c= -6, e= 5
1.11.2 (1,18 / 5,6) , (4,21 / 5,6) , (7,24 / 5,6) , (10,27 / 5,6)
1 3 7
1.11.3 a) b=-4, c=1, d=10 b)  , ,  c) (3,4 / 3,6) i (4,5 / 3,8)
2 2 2
1.11.4 a  2, b  0 o a  5, b  7
1.11.5 No existeix cap k.
1.11.6 k=18
1.11.7 z  2 , y  2 o z   2 / 3, y  2 / 3
 
1.11.8 w  ( 3 , 3 , 3 ) i w  (  3 ,  3 , 3 )
1.11.9 a) 11 , 13 b) arccos( 11 / 13 ) c) z= -3
1.11.10 a) 3 20 b) 12
3 10
1.11.11
2
1.11.12 a) z=0 b) 854
1.11.13 a) -49 b) 13 i 65 c) 117º52’ d) x  3 / 4
 1 1 1    1 1 1 
1.11.14 Hi ha dues solucions: u   , ,  i u   , , 
2 2 2  2 2 2
1.11.15 Hi ha dues solucions: m  1 i m  35 /13
1.11.16   11 o    11
3 1
1.11.17 a  , b  , c  5 , 90º
5 5
1.11.19 a) a  2 b) a  1
1.11.20 c) 24
 2 4 1 
1.11.21  , , 
 21 21 21 
1.11.22 a  0 o a  1
1.11.23 b) Àrea  1
19
1.11.24 a) b) 7/6
2
1.11.25 a) D=(0,2,2) b) 2 2
1.11.26 a) 2 17 b) 50 / 3
1.11.27 a) a  4 / 3 b) a  10 / 3 o a  2 / 3
1.11.28 a) AB  CD  24 , BC  AD  42 , els seus costats oposats són iguals,
per tant és un paral·lelogram. Àrea= 2 251
2.1.3 (-10, 2, -4), (15, -3, 6), (-2.5, 0.5, -1)
2.1.4 (8, 28, 20), (-2, 7, -5), (2/3, -7/3, 5/3)
2.1.5 Det  9 per tant són lin.ind.
2.1.6 k = -13/4
 3 1 5
2.1.7 v  a b c
2 2 2
1 1 4
2.1.9 5. a) 2 1 3  7 per tant són lin.ind. b) 5  7a  3b  0
1 0 0
6. k=2
 3  3  
7. a) Si a  1 lin. dep. en cas contrari lin. ind. b) c  u  v  3w
2 2
x  11 y  10 z  6
2.2.1  
3 27  11
17  x 20  y z  13
2.2.2  
23 21 11
2.3.1 ( 15 , 8 , -6 )
2.3.2 ( 0 , -5 , 7 )
2.4.1 a) 2 x  8 y  5z  18 b) x  2 y  2 z  5 c) 7 x  5 y  z  10
d)  x  9 y  11z  15
2.4.2 x  2 y  3z  0
2.4.3 2 x  4 y  z  7  0
2.4.4 4 x  12 y  7 z  1  0
Problemes.
1. a) 6 x  3 y  2 z  6 2. a) a  3 , b) 2 x  5 y  z  11
3. b) 2 x  y  2 z  5  0 4. a)  x  2 y  z  1  0 b) k  1
5. x  y  z  0 6. 2 y  z  1 7.
2.4.11 2x  z  1
2.5.1 ( 1 , 1 , -1 )
2.5.2 ( -1/5 , -8/5 , 17/25 )
2.7.1 No són coplanàries.
2.7.2 Sí són coplanàries.
2.7.3 k  1
2.7.4 3 p  q  12  0
2.7.5 a  17 / 2 .
2.7.6 a) a=2, b=4 b) 2a  b  8  0
 14 7  30
2.8.5 1. P  1, ,  , d 
 5 5 5
x 1 y  5 z  3
3.1.1   (o equivalent)
2 3 4
x 1 y  4 z  3
3.1.2   (o equivalent)
8 3 6
3.1.3 1,  7, 0
3.1.4  4,  1, 0
3.1.7 El punt no es troba sobre la recta paral·lela.
3.1.8 k  1 .
3.1.9 No es tallen per a cap valor de a. Són paral·leles per a a  1 .
3.1.10 a  0, b  5 / 3 , a  1, b  1 .
3.2.1 x  15  y  4 z
3.2.2 2 x  5 y  4( z  5)
6 8 3
3.2.4 a) Veure teoria. b) a  2 , dist  i a  1 , dist 
12 3
3.3.2   1,   4
3.5.2 a  0 . El pla és x  y  z  2
3.5.3 4 x  y  5z  5
3.6.1 x  2   y  z
4.4.1 (-2/19, -2/19, -83/19)
4.4.2 (8/7, 10/7, -9/7)
4.4.3 (-151/22, -19/22, -21/11)
4.4.4 (-124/21, 67/21, -92/21)
4.4.5 (-39/19, -22/19, 16/19)
4.4.6 (8/7, -79/7, -38/7).
4.4.7 (-1,0,-2)
 3 3 3
4.4.10 P   , , , d 1

 3 3 3 
 15 9 42 
4.4.11  , , 
 23 23 23 
 15 9 42 
4.4.12  , , 
 23 23 23 
4.5.2 1. (5/21, 64/21, -73/21) 2. (-51/14, -9/7, 57/14) 3. (39/38, -159/38, -23/19)
4. (7/3, -10/3, 4/3) 5. (-8/27, -127/27, -40/27) 6. (-2/13, 45/13, 5/13)
7. (10/3, -16/15, -92/15)
1 1 1 2
4.6.1 a) ( x, y, z )   , ,   (1,1,0) , dist 
4 4 4 2
x 3 y 5
b)   z 4, dist  3 .
2 2
x  6  k x  2  
x 1 y 1 z  2  
c)   d)  y  k e)  y  
2 2 1 z  3  z  1
 
4.6.2 (1,1,-1) i (2,0,1)
4.6.3 P  (17 / 9, 13 / 9, 1) , P' (2, 11/ 9, 11/ 9)
x  3  

4.6.4  y  4
z  2  

4.6.5 A  (3,4,6) , B  (3,5,5)
5.1.1 2
11 3
5.2.4
3
5.3.1 7.121
5.3.2 3.818
5.3.1  1 : x  2 y  2 z  24  0 ,  2 : x  2 y  2 z  36  0
3 14
5.5.2
14
5.7.5 ( 1, 1, 0 ) i ( 2, 3, -1 )
6.1.1 La recta i el pla es tallen al punt (5 / 2 , 1 ,  5 / 2)
 1 4  k 12  3k 
6.1.2 Si k  5 la recta i el pla es tallen al punt  , , 
5k 5k 5k 
Si k  5 la recta i el pla són paral·lels.
6.1.3 Si t=1 la recta no està ben definida, és un pla. Si t=-2 és una recta continguda
en el pla i si t  1,2 es tallen.
6.1.7 a) x  7 y  3z  9 b) Recta i pla es tallen en un punt.
6.2.1 a) Dos plans secants en la recta
 x   3/ 4    5
     
 y     13 / 4     15 
z  0   2 
     
b) Dos plans paral·lels i no coincidents.
c) Dos plans coincidents.
6.2.2 m  2 i n  6.
6.2.5 Si a  2 són paral·lels. Si a  2 es tallen en una recta.
6.3.1 Es creuen.
6.3.2 Es tallen al punt (1,1,2)
6.3.3 Es creuen.
6.3.4 Si t  1no són rectes sinó plans coincidents, si t  3 es tallen, si t  1,3 es
creuen.
6.3.6 a=1, P=(1,0,1)
6.3.7 (2,1,3)
6.3.8 Les rectes no es tallen per cap valor de a.
6.3.9 k  1
6.3.10 a  2 (per a a   6 les rectes són coincidents)
6.4.1 Si t=1 els tres plans són coincidents. Si t=-1/2 es tallen dos a dos segons rectes
paral·leles, si t  1,1 / 2 es tallen en un punt.
6.4.10 Si a  1 el sistema és incompatible (Prisma triangular). Si a  1 el sistema
és compatible indeterminat amb un grau de llibertat (tres plans que es tallen en
una recta). Si a  1,1 el sistema és compatible determinat (tres plans que es
tallen en un punt).
6.4.11 Per a  0 o a  15 incompatible (prisma triangular). Per a qualsevol altre
valor de a, compatible determinat (tres plans amb un punt d’intersecció).
7.1.1 49.107 °
7.1.2 42.906 °
7.1.3 a) Les rectes corresponen als valors a  4  21 i a  4  21
2 151
b) L’angle té cos 
151
7.1.4 a=0

7.1.5 w  (1, a,1)

v  (1,1, 2 )
 
1   wv (1, a,1)(1,1, 2 )
 cos(60º )  cos(w, v )     
2 wv 1  a  1 1  (1)  2
2 2 2 2
 
2 2

1 a  2

2  a2 4
1 1 a  2 1 a  2
 1  2  a2  1  a  2
2 2a 42
2a 2

1  2  2.414

 2  a   (2.414  a)
2
2
 2  a 2  5.827  4.828a  a 2 
2

 2  5.827  4.828a  a  0.793


1 2 2
Valor exacte: a 
22 2
4. a  4  21 i a  4  21
7.2.1 32.513°
7.2.2 42.45°
7.2.3
7.2.4
1 2 2
7.2.5 a) Es tallen. b) a 
22 2
7.3.6 30º
7.3.7 3, 3

9.1.1 a) v  (1,0,1) , m  1
9.1.2 a) P=(1,2,3) b) 44.4153º c) x  2 y  z  0
9.1.3 a) A=(4,0,0), B=(0,1,0), C=(0,0,-2), A  21 . b) 10.83
c) Aˆ  29.805º , Bˆ  83,773º , Cˆ  66,422 º
x  

9.1.4 a)  y  1  4  b) 10 x  y  2 z  6  0
z  7

9.1.5 a) (-1,-1,3), b)  x  4 y  2 z  3  0
 
9.1.6 a) v  (1,1,0) , w  (1,1,1) b) (1,0,3) c) x  y  2 z  5  0
x y 3 z  2
9.1.7 a)   b) x  y  z  0
3 1 2
 
9.1.8 a) v  (1,1,0) , w  (1,1,0) b) z  5
9.2.1 a) a = 26, b) a = -2
9.2.2 Sí és perpendicular.
9.2.3 Són paral·leles i no coincidents per a  4 .
 1 1
9.2.4 2 x  y  z  4 ,  2, , 
 2 2 
9.2.5 Són paral·leles si a  b , són perpendiculars si a  b
9.2.6 a  19
9.2.7 a) a  2 , b) a  1
9.3.1 a)  1 : 3x  2 y  4 z  12  5 29  0 ,  2 : 3x  2 y  4 z  12  5 29  0
b) A  (4,0,0) , B  (0,6,0) , C  (0,0,3)
c) A  63.6560º , B  41.9088º , C  74.4352º
x  3 y 1 z  4
9.3.2 a)   b)  ': 8x  y  3z  11  0
1 2 2
x  

9.3.3 a) a  2 y  2   b) a  1 dist  3
z  1

 1
9.3.4 a) v  (1,0,1) b) m  1 c) m  1 d) d   0.3536
2 2
9.3.5 a) P  (1,2,3) b) 44.4153º c) x  2 y  z  0
x 8 y 7 z  2
9.3.6 a)  3x  6 y  9 z  0 b)   c) Q  (6,3,4)
1 2 3
 x  1  2  x  2
 
9.3.7 a) r :  y   s :  y  1   b) Són dues rectes paral·leles
z   z  3  
 
5
c) dist=
3
 9 1  66
9.3.8 a) C   , ,1 b)
2 2  2
9.4.1 m = -1
x6 y2 z 3
9.4.2 m  6 , r :  
1 1 1
x  4  0
9.4.3 
y  3  0
 x  
 
9.4.4 v  (1,1,1) ,  y   (la solució no és única)
z  2  

x  2 z  0
9.4.5 
x  2 y  2  0
x  
 x y 1 z  2
9.4.6 y  1   
 z  2  2  1 1 2

x 1 y 1 z 1
9.4.7  
3 2 2
x  3  

9.4.8 r :  y  2
 z  4  

9.4.9 80.41º
9.4.10 48.70º
9.4.11 68.48º
9.4.12 5/6
9.4.13 Les rectes coincideixen.
9.4.14 Les rectes es tallen en un punt.
9.4.15 Les rectes són paral·leles.
5
9.4.16 a  1 , b 
2
9.4.17 a) Es creuen. b) Es tallen en el punt (0,3,3) c) Són paral·leles i no coincidents.
d) Són la mateixa recta.
9.4.18 a=3, (1,1,2)
9.4.19 m = 12, n = -3
9.4.20 Són paral·leles.
9.4.21
9.4.22 a  4 / 3
9.4.23 m  6
9.4.24 a  2  la recta r no està definida, a 1  es tallen en un punt
a  1,  2  es creuen.
1
9.4.25 b , a  5
5
9.4.26 m  5 , n  1
9.4.27 2
9.4.28 A'  (1,5,1)
9.5.1 a=0, b=5/3 i a=1,b=0
9.5.2 a) (2,1,-1) b) 30 graus.
9.5.3 (2,1,1)
9.5.4 2 x  2 y  3z  1  0
x  3  

9.5.5 y  4
z  2  

9.6.1  : 3x  2 y  z  0
9.6.2 Q  (4,6,2)
1
9.6.3 Si m  ,2  els plans es tallen en un punt.
2
Si m  2  els plans  2 ,  3 són coincidents i  1 els talla.
1
Si m    1 ,  2 són plans paral·lels i  3 els talla.
2
9.6.4 Q  (1,8,1)
y z
9.6.5 a)  : x  2z  3  0 b) x  
0 2
x  8 `y  1
9.6.6 t:   z  14
2 2
9.6.7 m0 i m4
Bibliografia.
Aquest document forma part de la col·lecció "Matemàtiques per a Segon de Batxillerat":

Àlgebra Lineal 2n Batxillerat


http://www.toomates.net/biblioteca/AlgebraLineal.pdf

Geometria Lineal 2n Batxillerat


http://www.toomates.net/biblioteca/GeometriaLineal.pdf

Càlcul infinitesimal 2n Batxillerat


http://www.toomates.net/biblioteca/Calcul.pdf

Programació Lineal 2n Batxillerat


http://www.toomates.net/biblioteca/ProgramacioLineal.pdf

Compendium PAU TEC: Totes les Proves de la Selectivitat de Catalunya 1998-2019


http://www.toomates.net/biblioteca/Pautec.pdf

Compendium PAU CCSS: Totes les Proves de la Selectivitat de Catalunya 1998-2019


http://www.toomates.net/biblioteca/Pauccss.pdf

Links d'interès.
Compendium EBAU Zaragoza Matemáticas II
http://www.matematicasentumundo.es/EVAU_Zgz_MII.pdf

Compendium EBAU Matemáticas aplicadas a Ciencias Sociales II


http://www.matematicasentumundo.es/EVAU_Zgz_MCCSSII.pdf

También podría gustarte